You are on page 1of 258

MATHEMATICS

Limits
About Me
Mentor of AIR 1, 12, 18, 24 and thousands of IIT Aspirants

14+ years of teaching experience (2007-Present)

Ex-Maths Faculty at top-notch IIT-JEE Coaching Institutes: Career Point,


Bansal Classes, Etoos & Vibrant, Kota

Star Maths Educator on Unacademy Plus. Each course starts from Basic to
JEE Advance level.

Renowned for providing vast, clear cut and well-organized study material
with top-quality questions.

Easy Access to Handwritten Notes, Sheets, Workbooks, DPP’s, PYQ’s


& Question Bank with Solutions.

MANOJ CHAUHAN
IIT-DELHI Alumni

"Trusting MC sir throughout my IIT JEE Preparation was the best decision I made in my life.

Attending sir's classes regularly, revising key concepts and class notes, solving sheets and PYQ's

are of utmost importance. Consistency and hard work will never stop aspirants get the desired

results. MC sir is not only the best Maths teacher but also an excellent mentor. His esteemed

guidance helped me to boost my confidence in performing well in the competitive exam."

"My parents are very much happy with my achievement. Regarding my journey, I attended all

classes of MC sir & used to complete homework on time. MC sir has always been my favorite

Maths teacher. I never had a phobia for Mathematics, credits to MC sir for making me fall in

love with the subject."


"Time is the most important aspect in deciding your rank. My primary focus was working on my

speed of solving each question & that too with the Best Approach. Attending MC sir’s classes

on a regular basis made me realize that a question can actually be solved in many ways & in

the least possible time. Solving DPP’s and Sheets helped me analyze which method I should

follow during the exam. I would like to dedicate my achievement in the all India level exam to

MC sir. His consistent support and mentorship proved to be the turning point in my life."

“Maths is my favorite subject, I used to dedicate most of my time in solving Maths questions.

Right after completing each lecture, I never missed revising each concept of the topic taught.’’

“Maths is all about practice, practice and practice. I attempted mock tests daily & used to

analyze previous year marks vs rank statistics & PYQ’s to predict where I stand, which topics to

improve and which concept holds prime importance. Solving PYQ’s helped me get an insight

about the difficulty level of the exam.''


Contents
S.No. Topic Page No.
1. Limits Key Concept 1–2

2. Limits Lecture Notes 3 – 49

3. Limits Solved Example, GNBERMAN 50– 93

CB-1,2,3,Exercise -I to V

4. Answer key 94 – 98

5. Revision Planner 99

6. Limits Concept building Solution (1 to 3) 100 – 118

7. Limits Solved Exercise - I 119 – 133

8. Limits Solved Exercise - II 134 – 156

9. Limits Solved Exercise - III 157 – 206

10. Limits Solved Exercise - IV 207 – 232

11. Limits Solved Exercise - V 233 – 240

12. Brahmastra (Final Revision) 241– 250


KEY CONCEPTS
THINGS TO REMEMBER :
1. Limit of a function f(x) is said to exist as, xa when
Lim f (x) = Lim f (x) = finite quantity..
x a  x a 
2. FUNDAMENTAL THEOREMS ON LIMITS:
Let Lim f (x) = l & Lim g (x) = m. If l & m exists then :
x a x a
REMEMBER
(i) Lim f (x) ± g (x) = l ± m (ii) Lim f(x). g(x) = l. m
x a x a
Limit  x  a
f (x)  x a
(iii) Lim  , provided m  0
x a g(g ) m
(iv) Lim k f(x) = k Lim f(x) ; where k is a constant.
x a x a

 
(v) Lim f [g(x)] = f  Lim g( x )  = f (m) ; provided f is continuous at x = m.
x a  x a 
 
For example Lim l n (f(x) = ln Lim f ( x ) = l n l (l > 0).
x a  x a 
3. STANDARD LIMITS :
sin x tan x tan 1 x sin 1 x
(a) Lim = 1 = Lim = Lim = Lim
x 0 x x 0 x x 0 x x 0 x
[Where x is measured in radians]
x
1
(b) Lim (1 + x)1/x = e = Lim 1   note however there hLim
0
(1 – h)n = 0
x 0 x   x n

and hLim
0
(1 + h )n 

n

(c) If Lim f(x) = 1 and Lim (x) = , then ;


x a x a
Lim ( x )[ f ( x ) 1]
Lim f ( x )( x )  e x a
x a
(d) If Lim f(x) = A > 0 & Lim (x) = B (a finite quantity) then ;
x a x a
Lim [f(x)](x) = ez {where z = Lim (x). ln[f(x)]} = eBlnA = AB
x a x a
x x
(e) Lim a  1 = ln a (a > 0). In particular Lim e 1 = 1
x 0 x x 0 x
x ann
(f) Lim  n a n 1
x a x  a

4. SQUEEZE PLAY THEOREM :


If f(x)  g(x)  h(x)  x & Limit Limit Limit
x  a f(x) = l = x  a h(x) then x  a g(x) = l.

5. INDETERMINANT FORMS :
0 
, , 0  , 0º, º,    and 1
0 

1
Note :
(i) We cannot plot  on the paper. Infinity () is a symbol & not a number. It does not obey the
laws of elementry algebra.
(ii) += (iii)  ×  =  (iv) (a/) = 0 if a is finite
a
(v) is not defined.
0
(vi) a b = 0 , if & only if a = 0 or b = 0 and a & b are finite.
6. The following strategies should be born in mind for evaluating the limits:
(a) Factorisation
(b) Rationalisation or double rationalisation
(c) Use of trigonometric transformation ;
appropriate substitution and using standard limits
(d) Expansion of function like Binomial expansion, exponential & logarithmic expansion, expansion of sinx ,
cosx , tanx should be remembered by heart & are given below :
x ln a x 2 ln 2 a x 3ln 3a
(i) a x  1     .........a  0
1! 2! 3!
x x2 x3
(ii) e x  1     ............ x  R
1! 2! 3!
x 2 x3 x 4
(iii) ln(1+ x) = x     .........for  1  x  1
2 3 4
x3 x5 x7   
(iv) sin x  x     ... x    , 
3! 5! 7!  2 2

x2 x4 x6   
(v) cos x  1     ...... x    , 
2! 4! 6!  2 2

x 3 2x 5   
(vi) tan x = x    ........ x    , 
3 15  2 2

x3 x5 x7
(vii) tan–1x = x     .......
3 5 7

2
Limits

INTRODUCTION TO LIMITS

DEFINITION OF LIMITS :
Definition
We can also say that if f(x) becomes arbitrarily
Limit of a function at any point
close to a unique number L as x approaches
can be said to be the value of
‘c’ from either side, then the limit of f(x) as x
the function at the immediate
approaches ‘c’ is L. This is written as:
neighbourhood of that point.
lim f ( x ) = L
x →c

Let us take an example:


x2 − 4
y= = x + 2; x  ≠  2
x −2
4
0
Here, f(2) is not defined as it is of the form
0

which is an indeterminate form.


But as x → 2, f → 4
which means if x is sufficiently close to 2, 2
then f is arbitrarily close to 4.
Here, solving limit leads to dividing by (x – 2)
(Really!)
In above example, x→2 means x is very close
to 2 like 2.000000 ... 00001 or
1.9999 ... 999999.
Similarly, x → 2 ⇒ f → 4 which means when
x becomes very close to 2, then f becomes
very close to 4 like 4.00000000 ... 0001 or
3.99999 … 99.
In such case it is said that limit of function at
x = 2 is 4. It is represented by lim f ( x ) = 4
x →2

(approx. value).
x2 − 4
Now SOLVING TRICK for lim  :
x →2 x−2
0
y Since it is of the form at x=2, cancel out
0
the common factor x-2 So, that it is no longer
0
of the form .
Limits

0
3.
( x − 2)( x + 2) = lim
lim
x →2 x−2 x →2
( x + 2) = 4
Now, there is no problem in putting the value
of x = 2 in the limit.

Deleted neighbourhood of a point ‘a’


(a − δ, a + δ) − {a} is called deleted
neighbourhood of ‘a’ where δ is very small
positive.
Deleted neighbourhood
(a − δ, a) is left neighbourhood. of ‘a’
(a, a + δ) is right neighbourhood.
a-δ a a+δ
Meaning of small positive is something like
0.00000000000000 … ∞
To find limit, function must be defined in
neighbourhood of the point.
Only comment on Limit when function is
continuous in neighbourhood.

Left and Right Hand limits :


Y

x →a
( )
Lim− f ( x ) =  = f a  means as x tends to a

y=x+2
from left hand side, f tends to  , x ≠ a.
2 X
from Right
( )
lim f ( x ) = M = f a + means as x tends to a
x →a +
from left
from right hand side, f tends to M, x ≠ a.

Left Hand limit (L.H.L) here means value of


function at left neighbourhood of point.
L.H.L = lim−  ( x + 2 ) = 4
x →2

x → 2 means x = 2 - 0.000000 … 001


Similarly, Right Hand limit means value of


function at right neighbourhood of point.
Limits

4
R.H.L = lim+  ( x + 2 ) = 4
x →2

x → 2+ means x=2 + 0.000000 … 0001.

Existence of limit :
If L.H.L = R.H.L = finite value M, then limit
exists and is equal to ‘M’.
( ) ( )
Or f a −1 = f a + = finite = Limit

In previous example, L.H.L = 4 = R.H.L at x = 2.


So, limit exists at x = 2 and lim ( x + 2 ) = 4
x →2

7 INDETERMINATE FORMS
0 ∞
,   ,  0 × ∞,  ∞ − ∞,  1∞ ,  00 ,  ∞0  
0 ∞
f
If lim  f ( x ) = lim g ( x ) = 0 , then lim    take
x →a x →a x →a g

0
the form of .
0
Sometimes this limit exists and has finite
value. The determination of limits in such case
is referred to as evaluation of indeterminate
form.
Some types of Determinate form ⇒ ∞ + ∞ =
∞; 0∞ = 0
Some examples of Indeterminate form:

x2 − 4  0  x2 4

x − 2 0
 form  ,   
 x−2 x−2
− ∞− ( )
∞  form ,

1
 1  ∞ 
(x −4 
2
)
 x − 2 
( )
0 × ∞  form ,    x −
1
2
 ∞  form
x2 − 4

All the above examples are for x = 2.

NOTE :
(i)
x →a
()
Limit f x ⇒ x ≠ a

(ii) Limit  f ( x ) is said to exits at x = a if


Limits

x →a

5.
Limit  f ( a + h) = Limit f ( a − h)
h→ 0 h→ 0

= (a finite quantity, disregards whether f is


defined at x = a)
( )
     f a + ( )
                       f a −  
(R.H.L ) (L.H.L )

TYPES OF INDETERMINATE FORMS

0 ∞
1. 2. 3. 0 × ∞
0 ∞

4. ∞-∞ 5. 00 6. 1∞

7. ∞0 Indeterminate form→ that value


of the limit is not obvious

x2 + x − 6 0  9 − 3 − 6 0 
Ex. Lim =    = 
x →−3 x+3 0  −3 + 3 0

→ Cannot be determined.
The above result has no meaning as a real
number. It is called an indeterminate form
because you cannot, from the form alone,
determine the limit.
Make a table to estimate the above limit.
What is your estimate?
The limit is –5.
The solution is as follows:
( x + 3)( x − 2) = lim
lim
x →−3 x+3 x →−3
( x − 2) = −3 − 2 = −5
Limits

6.
Checking if limit Exists
To check if limit exists for f(x) at x = a
We check if :

Left hand limit = Right hand Limit = L


lim f ( x ) = lim+ f ( x ) =L
x →a − x →a

Recall Notation
lim f ( x ) = L
x →a

This means:
As x gets closer to a, f(x) gets closer to L

Determinate forms :
Let a ∈ R
(1) a + ∞ = + ∞
(2) a - ∞= -∞
(3) +∞ + ∞ = + ∞
(4) -∞ - ∞= -∞
 + ∞    if     a > 0
(5) a×(+∞) =  where a ≠ 0
−  ∞    if     a < 0  

 +∞      if     a < 0
(6) a × (-∞) =  where a ≠ 0
−∞       if     a > 0  
a a
(7) = 0 and =0
+∞ −∞
a  + ∞     if     a  >  0
(8)  = where a  ≠ 0
0 −  ∞     if     a  <  0  

Example :
2
lim =  ?
x →0 x

2  2 
lim+ = +∞             
x →0 x  0.00000 ...  0001 
2  2 
lim = −∞              
x →0− x  −0.0000 ...  001 
∴ L.H.L ≠ R.H.L. So limit does not exit.
Limits

7.
Q.1 f(x) = [x], find lim x  .
x →2

Sol. (1) LHL = lim− x  = 1


x →2

(2)  RHL = lim+ x  = 2


x →2

⇒ LHL ≠ RHL

∴ Limit does not exist.

Q.2 lim {x} =  ?


x →3

Sol. LHL = lim− {x} = 1 Y


x →3
y={x}
RHL = lim+ {x} = 0
x →3

∴ LHL ≠ RHL
X
So, limit does not exist at x = 3. 1 2 3

Note :

(1) lim x  does not exist.


x →I

( RHL = I, LHL = I - 1)
(2) lim {x} does not exist.
x →I

( RHL = 0, LHL = 1)

x
Q.3 f (x) = . Find lim f ( x ) .
x x →0

x −x
Sol. LHL = lim−
x →0 x
= lim−
x →0 x
= −1

x x
RHL = lim+ = lim− =1
x →0 x x →0 x

⇒ LHL ≠ RHL

So, limit does not exist.


Limits

8.
1
Q.4 Limit  tan−1
x →0 x
 

1 π
Sol. lim tan−1   = tan−1 ( ∞ ) = = RHL
x →0+ x 2

1 −π
lim tan−1   = tan−1 ( −∞ ) = = LHL
x →0− x 2

∴ LHL ≠ RHL. Limit does not exist.



Q.5 Limit  sin−1 ( sec x )
x →0

Sol. LHL=  lim− sin−1 ( secx ) Y y = sec x


x →0

( )
But sec 0− > 1 and domain of sin−1x is [-1, 1].

∴ sin ( sec0 ) will not exist.


−1 −

Similarly, for RHL, sin ( sec0 ) will not exist.


−1 + ·
∴ Both LHL and RHL does not exist. X

So, limit does not exist.

as x→0, sec→1+ and sin-1(1+) does not exist.

Ex: Same as above question, lim sin−1 x2 + 1


x →0
( )
does not exist. As x → 0, x2 + 1 > 1
( )
So, sin−1 x2 + 1 will not exist.

Q.6 Limit  cot −1x2


x →0

π
Sol. LHL = lim− cot −1 ( x2 ) =
x →0 2
π
RHL = lim+ cot −1 ( x2 ) =
x →0 2
π
∴ LHL = RHL = = limit
2
Limits

9
Q.7 Limit f ( x ) = x  + {x}  
x→1

Sol. R.H.L. = 1 + 0 = 1

L.H.L.=0 + 1 = 1

LHL = RHL = 1

So lim  f ( x ) = 1
x→1

Q.8 Limit x sgn ( x − 1)  
x→1

Sol. As x→1+→ x – 1 > 0 ⇒  sgn(x – 1) = 1

∴ RHL = 1 × 1 = 1

As x → 1− → x – 1 < 0 ⇒   sgn(x – 1) = –1

∴ LHL = 1 × (-1) = – 1

⇒ LHL ≠ RHL, so limit does not exist.

Limit [sin x] 
Q.9 x→
π
2

   π−
Sol. x→
2
⇒ sin x = 1− ⇒ 1−  = 0

⇒ LHL = 0
π
+

x→
2
⇒ sin x = 1− ⇒ 1−  = 0
Y
• 1 •
⇒ RHL = 0 y=[sinx]

∴ LHL = RHL = 0 = Limit -2π
• 3π •

• π •π

X


lim [sin x ] = 0 − 2
x→
π 2 2
2

Note :  -1  • 
Limit is ABSOLUTE in nature, it gives the
EXACT value.
Limits

10.
lim [cos x]
Q.10 x→
π
2

   π−
Sol. x→
2
⇒ cos x = 0+   Y
1
⇒ [cos x] = 0 = LHL y=cosx
π+
x→
2
⇒ cos x = 0− •
π X

⇒ cosx  = −1 = RHL 2

⇒ LHL ≠ RHL

So, limit does not exist.

One sided limit :

If one side of neighbourhood is not in


domain, then the limit of the function will
be the limit on other side.

Ex: lim x  : → Domain is [0, ∞).


x →0

So, left side of function at x=0 does not exist.


So, limit = RHL = lim+ x = 0 
x →0

2x − 3, x ≥ 2
Q.1 Consider f ( x ) =   . Find lim  f ( x ) .
 4 − x , x < 2
2 x →2

Sol. x → 2+ ⇒ RHL = 2 × 2 − 3 = 1 

x → 2− ⇒ LHL = 4 − 22 = 0 
⇒ LHL ≠ RHL
⇒ lim f ( x ) does not exist.
x →2

1
Q.2 Find lim  
x→1 x 
 

1
Sol. Domain of  is ( −∞, 0 ) ∪  1, ∞ )
x 
Limits

11.
So, limit will be equal to RHL as 1– is not domain.

x → 1+

⇒ [x] = 1
1
⇒ RHL =
1
=1

∴ Limit = 1

Methods to Evaluate Limit
Various Strategies (To evaluate limit):
Algebraic Methods
Factorisation / Rationalisation / Double
rationalisation / Use of binomial theorem /
algebraic identities/involving Law of Love if
necessary
(We love ∞ in Denominator and 0 in Numerator)

2n + 3  ∞ 
Q.1 lim
n→∞
 form 
7n − 6  ∞ 

Sol. Applying law of love, divide by ‘n’ in both numerator and denominator.
3
2+
n = 2 + 0 = 2    3 6 
We get, lim
n→∞ 6 7−0 7   and → 0 
 n n 
7−
n

x3 – 1
Q.2 lim
x→1 x−1
 

(A) 0 (B) 3 (C) 2 (D) None of these

Sol. (B)
x3 – 1 0
lim 
x→1 x − 1
  (It is of the
0
form)

( x − 1) ( x 2
) = 1 + 1 + 1 = 3 
+x+1
lim  
x→1 x−1
Limits

12.
x5 − 1
Q.3 lim 
x→1 x − 1

(A) 0 (B) 3 (C) 2 (D) None of these

Sol. (D)
0
Again, it is of the form   .
0

So, using factorisation, remove the factor ‘x – 1’ causing ‘0’.


( x − 1) ( 1 + x + x 2
+ x3 + x4 ) = 1+ 1+ 1+ 1+ 1 = 5
⇒ lim  
x→1 x−1

Standard limits :
x n − an
1. lim = na n− 1 ;n ∈ N
x →a x − a

Proof:
n n
n x x
a   −1   −1

xn − an
= a = an − 1 a  
x−a x  x
a  − 1  −1
a  a
x
let = t . As x→a ⇒ t→1
a
 tn − 1 
⇒ lim   an− 1   = na (Same as Previous Question)
n− 1

t →1
 t − 1 

Note :
xn − an
= xn− 1 + xn−2a + xn−3a2 +  ... + xan−2 + an− 1  
x−a

xn − an
2. lim ;  n ∈ Q , n ∉ N
x →a x−a
Proof :
Let x = a + h, h → 0
n
 h
1+  − 1
(a + h) − a = an lim   a  , h → 0 

n n

⇒ lim 
h→ 0 a +h−a h→ 0 h a
Limits

13.
 h n (n − 1)  h  
2

1 + n +   + … − 1
 a 2 a 
= an lim   
h→ 0 h
n n (n − 1) h n (n − 1)(n − 2 )  h 
2

= a lim +
n
+   +  ... 
h→ 0 a 2! a 3! a
= nan− 1

Alternative Method :
0 ∞
Using L' Hospital rule. (used in case of ,     ) 
0 ∞
f f′ f′′
Lim = lim = lim   (will be studied later on)
x →a g x → a g ′ x → a g ′′

x 100 − 1
Q.1 lim  20
x→1 x −1
(A) 5 (B) 3 (C) 2 (D) None of these

Sol. (A)
0
Limit is of   
0
form

Using L' Hospital method,


100x99 100
⇒ lim
x→1 20x 19
=
20
= 5 

Alternate method :
y5 − 1
 x20 = y ⇒ lim
y →1 y−1
=5

x3 − n
Q.2 lim 
x→1 x − 1
. Limit exists. Find n and limit.

(A) 1, 3 (B) 3, 1 (C) 1, 2 (D) None of these

Sol. (A)

Numerator → 1 – n ; Denominator → 0

If 1 – n ≠ 0, limit does not exist.

If 1 – n = 0, limit may exist if the factors of x – 1 get cancel out.


x3 − 1
∴ n = 1 ⇒ l im
x→1 x−1
= 3 
Limits

14.
1 1

(cosx ) 3 − ( cosx ) 2
Q.3 lim
x →0 sin2 x
 equals

1 1 1
(A) (B) − (C) (D) None of these
6 6 12

Sol. (C)

Let cosx = y6 so as x → 0 ⇒ y → 1

y2 − y3 y2 ( 1 − y )
lim
y →1 1 − y 12
= lim
y →1
(
1 − y6 1 + y6
 
)( )
 y2   ( 1 − y )  = 1 × 1 = 1
 lim   lim
 y → 1 1 + y   y → 1 1 − y  2 6 12
6 6


−1 + cot 3 x
Q.4 lim
x→
π −2 + cotx + cot 3 x
 
4

3 4 3
(A) (B) (C) − (D) Does not exist
4 3 4

Sol. (A)
π
cot x = t so as x → ⇒t→1
4
t3 − 1  t 3 − 1
⇒ lim = lim
t →1
(
t 3 + t − 2 t → 1 ( t − 1) t 2 + t + 2 )
 t3 − 1   1  1 3
lim     = 3× =
 t − 1  t + t + 2 4 4
2


t →1

2x + 23− x − 6
Q.5 Find the value of Limit
x →2
2− x − 21− x
 

Sol. 2x = t for x → 2 ⇒ t → 4
8
t+ −6
t t 2 − 6t + 8
⇒ lim = lim  
x →4
1 2 t →4
t −2

t t

( t − 2)( t-4 ) ( t − 2) ( t +2 )( t −2 )
= lim = lim =2×4=8
x →4
t −2 t →4
t −2
Limits

15.
x
Q.6 limit
x →0
5−x − 5+x
(A) 5 (B) − 5 (C) 2 5 (D) None of these

Sol. (B)

Rationalising, we get
x ( 5−x + 5+x ) = lim x ( 5−x + 5+x ) 
lim
x →0
(5 − x ) − (5 + x ) x →0 −2x

2 5
= =− 5
−2

3− x
Q.7 lim   
x →9
4 − 2x − 2
(A) 1 (B) 2 (C) 3 (D) None of these

Sol. (D)

3− x 3 + x   4 + 2x − 2 
lim
x →9
× × 
4 − 2x − 2  3 + x   4 + 2x − 2 
(Double Rationalization)

(9 − x) (4 + 2x − 2 )
= lim
(3 + x ) (18 − 2x)
x →9

1 (4 + 4) 2
= × =  
2 ( 3 + 3) 3

1
Q8 lim xsin
x →0 x
(A) 1 (B) 2 (C) 3 (D) None of these

Sol. (D)
1
as x → 0,
x
→ ±∞

1
In this range, sin will always lie in [-1, 1] which is finite quantity.
x

Since x→ 0, when multiplied by a finite quantity will always give the limit as 0.
1
So, lim  xsin = 0 
Limits

x →0 x
16.
 π   π 
Q.9 Lim cos 
x →1

 x + 1
. cos 
 x − 1 

(A) 0 (B) 1 (C) –1 (D) None of these

Sol. (A)
π π  π 
As x→1,
x+1
→ ⇒ cos 
2  x + 1
→0

 π 
Again cos   ∈ [ −1, 1]
 x − 1
 π   π   π 
So, 0 × cos   → 0 ⇒ lim cos   cos   = 0 × finite = 0
 x − 1 x → 1
 x + 1  x − 1

ax + b − 2
Q.10 If lim 
x →0 x
= 1 , then (a, b) is equal to

(A) (2, 4) (B) (4, 4) (C) (8, 4) (D) None of these

Sol. (B)

Limit exists and denominator → 0

∴ Numerator must also tend to 0

⇒ b −2 = 0 ⇒b = 4

Using rationalization,
ax + 4 − 2 ax + 4 − 4
lim = lim = 1 
x →0 x x → 0
(
x ax + 4 + 2 )
a
⇒ lim =1
x →0
ax + 4 + 2
a
⇒ = 1 ⇒ a = 4 
2+2

2x2 − 3x + 7
Q.11 lim
x →∞ x 2 − 4x + 2
  is equal to

1 7
(A) 2 (B) (C) (D) Does not exist
2 2

Sol. (A)

Divide by x2 (highest degree)


Limits

17.
3 7
+ 2−
x x2 = 2 − 0 + 0 = 2 
⇒ lim
x →∞ 4 2 1−0+0
1− + 2
x x

2x2 − 7x – 7
Q.12 lim
x →∞ x−4
; x ∈ N is equal to

7
(A) 2 (B) 0 (C) (D) Does not exist
4

Sol. (D)

Divide by x in numerator and denominator


7
2x − 7 −
x → ∞ − 7 → ∞.
lim
x →∞ 4 1−0
1−
x

∞ is not finite ∴ limit does not exist.

3x2 − 7x + 4
Q.13 lim
x →∞ 4x 3 + 3x 2 − 7x + 5
;  x ∈ N  is equal to

3 4
(A) (B) (C) 0 (D) Does not exist
4 5

Sol. (C)

Divide by x2 in numerator and denominator


7 4
3− + 2
x x 3
lim
x →∞ 7 5
=

= 0 
4x + 3 − + 2
x x

12 + 22 + 32 + ... + n2
Q.14 Limit 
n→∞ n3
is equal to

1 1
(A) (B) (C) 0 (D) Does not exist
6 3

Sol. (B)
n 1 1
n (n + 1)( 2n + 1) 1 +  2 + 
n n n  ( 1)( 1 + 0 )( 2 + 0 ) 1
lim = lim  = =  
n→∞ 6n3 n→∞ 6 6 3
Limits

18.
x2 + 1
Q.15 Limit
x →−∞ 3x − 6
is equal to

1 1 1
(A) (B) − (C) (D) Does not exist
3 3 6

Sol. (B)
1 1
x 1+ −x 1 + 2
x2 x −1 1 + 0 1
lim = lim =  =-
x →−∞  6
x 3 − 
 x →−∞  6
x 3 − 
 3−0 3
 x  x

3x2 − 1 − 2x2 − 1
Q.16 Find lim
x →−∞ 4x + 3
 .

1 1
x 3−
x 2
− x 2− 2
x = − 3−0 − 2−0 =
− ( 3+ 2 ) 
Sol. lim
x →−∞  3 4+0 4
x 4 + 
 x 

 1 12 
Q.17 Find the value of limit  x + 2 − x x →−2 3 .
+ 8
 
1 12
Sol. lim 
x →−2  x + 2

( x + 2) x2 − 2x + 4 ( )


 

= lim
x2 − 2x − 8
= lim
( x + 2)( x − 4 )  

x →−2
( x + 2) ( x 2
− 2x + 4 ) x →−2
( x + 2) ( x − 2x + 4 )
2

x−4 −6 1
= lim = =−  
x →−2 x − 2x + 4
2
4+4+4 2

Q.18 Find the value of limit ( x →±∞


x2 − 2x − 1 − x2 − 7x − 3 . )
Sol. Using rationalization, we get :
 2
x 5 + 
( ) (
x2 − 2x − 1 − x2 − 7x − 3 )=  x
limit
x →±∞
x2 − 2x − 1 + x2 − 7x − 3  2 1 7 3 
 
x  1− − 2 + 1− − 2 
 x x x x 


(5 + 0) =±
5
 
1−0−0 + 1−0−0 2
Limits

19.
 4x3 
Q.19 Lim  4x2 + x −
x →∞  x+2
 is equal to
 
4 9 9
(A) (B) (C) (D) None of these
9 4 2

Sol. (B)
 
 1 4
Lim x  4 + −
x →∞
 x

2
 1+ 
 x
Now rationalization, we get:
 ( 4x + 1)( x + 2) − 4x2 
 1 4x  x  
x 4 + −
x x + 2
  x ( x + 2 ) 
Lim   = Lim   
x →∞
1 4 x →∞
1 4
4+ + 4+ +
x 2 x 2
1+ 1+
x x
9x + 2 2
9+
= lim x + 2 = lim x  
x →∞
1 4 x →∞  
4+ +  2 1 4 
x 2
1+ 1 +  4 + + 
x  x  x 2
 1+ 
 x
9 9
= =  
2+2 4
Note :

Polynomial
In case of f ( x ) = ,
Polynomial
where x→±∞; just use the highest degree
term in both numerator and denominator
to find limit and ignore other terms.

Q.20 Limit  tan x ( )
2sin2 x + 3sinx + 4 − sin2 x + 6sinx + 2 is equal to
2
π
x→
2

1 1 1
(A) (B) − (C) (D) None of these
6 6 12
Limits

20
Sol. (A)

Rationalizing, we get:

(
tan2 x sin2 x − 3sinx + 2 )
lim
x→
π
2sin x + 3sinx + 4  +   sin x + 6sinx + 2
2 2
 
2

( )(
sin2 x sin2 x − 1 sin2 x − 2 )
lim  
x→
π
2 cos2 x ( 2sin2 x + 3sinx + 4  +   sin2 x + 6sinx + 2 )
−1 × ( −1) 1 1
= = =  
2 + 3 + 4  +   1 + 6 + 2 3+3 6

ax3 + bx2 + cx + d
Q.21 lim
x →0 x2
 . Limit exists. Find a, b, c, d.

 0 + 0 + 0 + d
Sol. It is 
 0
 form.

If d ≠ 0, then limit does not exist.


ax3 + bx2 + cx ax2 + bx + c
So, d = 0 ⇒ lim
x →0 x2
= lim
x →0 x
 

Again, c must be 0 for limit to exist.


ax2 + bx
∴ lim
x →0 x
= lim  ax + b = b 
x →0

∴ a, b ∈ R and c, d = 0

f (x)
Q.22 If lim
x →0 x4
= 3  where f is a four degree polynomial, then the value of f(1) is

1
(A) 3 (B) (C) 9 (D) None of these
3

Sol. (A)

Let f ( x ) = ax4 + bx3 + cx2 + dx + e

f (x)  b c d e 
∴ lim
x →0 x 4
= lim  a + + 2 + 3 + 4 
x →0
 x x x x 

So, for limit to exist, b = c = d = e = 0


lim a = 3 ⇒ a = 3 
x →0
Limits

⇒ f(1) = 3 × 1 + 0 + 0 + 0 + 0 = 3
21.
etanx − 1
Q.23 limx→
π etanx + 1
 
2

(A) 1 (B) 0 (C) –1 (D) None of these

Sol. (D)
π
→ tan x → ±∞   →  discontinuity at x = π 
As x →
2  2
So, check for LHL and RHL separately.
 1 
etan x  1 − tan x 
π− etan x − 1  e  1−0
⇒ x→ ⇒ lim− tan x = lim− = = 1 = LHL 
2 π e + 1 x→ π tan x  1  1+ 0
2 e
 1 + etan x 
x→
2

π+
x→ ⇒ tan x → −∞ ⇒ etan x → 0 
2
etan x − 1 0 − 1
∴ lim+ = = −1 = RHL 
x→
π etan x + 1 0 + 1
2

∴ LHL ≠ RHL
∴ Limit does not exist.

n
r3 − 8
Q.24 Evaluate lim ∏
n→∞
r =3 r3 + 8
 , where ∏ represents product of function.

2 2 1 2
(A) (B) (C) (D)  
3 5 3 7

Sol. (D)
n
r3 − 8
P = lim ∏
n→∞
r =3 r3 + 8
n
 r − 2   r 2 + 2r + 4 
= lim ∏   2 

n→∞
r = 3  r + 2   r − 2r + 4 

n
 r − 2  n  r 2 +  2r +  4 
= lim ∏  ∏ 2  
r = 3  r + 2  r = 3  r − 2r + 4 
n→∞

 1 2 3 4 5 (n − 5 ) (n − 4 ) (n − 3 ) (n − 2 ) 
= lim  ... ×
n→∞ 5 6 7 8 9
 (n − 1) (n) (n + 1) (n + 2) 

 19 28 39 (n2 − 2n + 4 ) (n2 + 3) × (n2 + 2n + 4) 
 × × ... 2 × 2 
7 12 19 (n − 6n + 12) (n − 4n + 7 ) (n2 − 2n + 4) 
 
Limits

22

= lim 
1⋅2⋅3⋅4
×
( n2 + 3 ) (n2 + 2n + 4 ) 
n→∞ (n − 1 ) n (n + 1 ) (n + 2 )

7 × 12
 

=
2
lim 
 ( )(
 n2 + 3 n2 + 2n + 4 
 )

7  (n − 1) n (n + 1)(n − 2 ) 
n →∞


  3  2 4 
 1 + 2 1 + + 2  
2  n  n n   2 ( 1 + 0 )( 1 + 0 + 0 ) 2
= lim    = =
7 n →∞
 1 − 1  1 1 + 1   1 + 2   7 ( 1 − 0 )( 1 + 0 )( 1 + 0 ) 7
   
n 

n

n  
2
Hence P =
7

1 + x  −  1
Q.25 Evaluate lim x →0 x
.

1 + x  −  1 0
Sol. When x = 0, the expression
x
takes the form ,
0

Rationalizing the numerator, we have

1+ x − 1 ( 1 + x  − 1 )( 1 + x  + 1 )
lim = lim
x →0 x x →0
x ( 1+ x + 1 )
1+ x − 1 1 1
= lim = lim =
x →0
( 1+ x + 1 x ) x →0
1+ x + 1 2

3− x
Q.26 Evaluate lim  4 − x →9
2x − 2
.

3− x( 3+ x ) (
4 + 2x − 2  0  )
Sol. lim   × ×  
x →9
4 − 2x − 2 3+ x (
4 + 2x − 2  0  )
(9 − x ) × ( 4 + 2x − 2 )  
= lim
x →9 16 − ( 2x − 2 ) (3 + x )
= lim
(9 − x ) × ( 4 + 2x − 2 )

x →9 2 (9 − x ) (3 + x )
= lim
1
×
(
4 + 2x − 2 1 4 + 16 2
= × =
)
2 2 3+ 9 3
Limits

x →9
3+ x

23.
Q.27 Evaluate lim
3
x3 + 3x2 − x2 − 2x .
x →∞

Sol. lim 3 x3 + 3x2 − x2 − 2x


x →∞
1 1
  3  3   2  2
= lim x3  1 +   − x2  1 −    
x →∞
  x    x 

 1 1

  3 3  2 2 
= lim x  1 +  −  1 − 
x →∞  x x 
  
1 1
 3 3  2 2
1 +  − 1 − 1 −  + 1
x x
= lim    
x →∞  1
 
x
 1
1  1

  3 3 3  1 − 2  2 − 1 2
3 1 +  − 1
 x   x
 
   
= lim +
x →∞ 3  2
x − 
 x
 1
1  1
1
  3 3 3  1 − 2  2 − 12  2
3 1 +  − 1
 x   
x 
   
= lim +  
x →∞  3   2 
 1 +  − 1  1 −  − 1
 x   x 

 1 1
  21 1

3  t 3 − 13   p − 12 
     3  2
= lim   + lim   ×2 (Let  1 +  = t &  1 −  = p )
t →1
( )
t − 1 p→ 1 p − 1  x  x

 1 −2   1 −1 
= 3 ×  ( 1) 3  + 2  ( 1) 2  = 1 + 1 = 2
3  2 

Q.28 x →∞
(
Evaluate lim ( x + 1)( x + 2 )( x + 3) ) 3
−x.
1

Sol. x →∞
(
lim ( x + 1)( x + 2 )( x + 3) ) 3
− x 
1
( 1 + y )( 1 + 2y )( 1 + 3y )  3
= lim   − 1    Put x = 1 

y →0 y y

 y

Limits

22.
1

= lim
( 6y 3 + 11y 2 + 6y + 1 ) 3
−1
 

y →0 y
1
1+ (6y ) + Higher degree terms − 1
= lim 3 =2
y
y →0

( )
sin2 x

Q.29
2 2 2 2
lim 1cosec x + 2cosec x + 3cosec x + ... + 100cosec x
x →0

( )
sin2 x

Sol.
2 2 2 2
lim 1cos ec x + 2cos ec x + 3cos ec x + ... + 100cos ec x  
x →0
sin2 x
 cosec2 x cosec2 x cosec2 x 
 1   2   99  
= 100lim  + + ... + + 1
x →0  100




 100



 100

 

= 100 (0 + 0 + … + 0 + 1) = 1000

SANDWICH THEOREM OR SQUEEZE PLAY


THEOREM FOR EVALUATING LIMITS :
General :
The squeeze principle is used on limit
problems where the usual algebraic methods
(factorisation or algebraic manipulation
etc.) are not effective. However, it requires
to ‘’squeeze’’ our problem in between two
other simpler function whose limits can be
easily computed and equal. Use of squeeze
principle requires accurate analysis, in-depth
algebra skills careful use of inequalities.
Statement :
If f, g and h are 3 functions such that f(x) ≤
g(x) ≤ h(x) for all x in some interval containing
the point x = c, and if
Lim  f ( x ) = Lim  h ( x ) = L ⇒ Lim  g ( x ) = L
x →c x →c x →c

From the figure note that Lim  g x =1


x →0
()
Limits

23.
Note :

The quantity may be a finite number, +∞ or


–∞.
Similarly, L may be finite number, +∞ or –∞.

x + 7sinx
Q.1 Evaluate Lim 
x →∞ -2x + 13
using Sandwich theorem.

Sol. We know that, -1 ≤ sinx ≤ 1 for all x.

⇒ –7 ≤ 7 sinx ≤ 7

⇒ x – 7 ≤ x + 7 sinx ≤ x + 7

Dividing throughout by -2x + 13, we get


x - 7 x+7sinx x + 7
-2x+13

-2x+13

-2x+13
  for all x that are large.

7
1 - 
x - 7 x =  1 − 0 = - 1  
Now, lim
x →∞ -2x+13
=lim
x →∞ 13 −2 + 0 2
-2 + 
x
7
1 + 
x + 7 x =  1 + 0 = - 1
and lim =lim
x →∞ -2x+13 x →∞ 13 −2 + 0 2
-2 + 
x
x + 7sinx −1
∴ lim =  
x →∞ −2x + 13 2

Q.2 If [·] denotes the greatest integer function, then find the value of
x  + 2x  + ... + nx 
lim     2 .
x →∞ n

Sol. nx − 1 < nx  ≤ nx

Putting n=1, 2, 3, …, n and adding them,

x ∑ n − n < ∑ nx  ≤ x ∑ n

∑ n 1 ∑ nx  ∑n
∴ x
n2
− <
n n 2
≤ x. 2 …(1)
n
 ∑n 1 ∑n 1 x
Now, lim x. 2 −  = x.lim 2 − lim =  
n→∞
 n n n → n x →  ∞ n 2
Limits

24.
 ∑ n ∑n x
lim x. 2  = x lim 2 =
n→∞  
 n  n →∞ n 2

As the two limits are equal, by equation (1),


∑ nx  x
lim
n→∞ n 2
=
2

1 2 n
Q.3 Evaluate lim
n→∞ 1+n 2
+
2+n2
+…+
n + n2
.

1 2 n
Sol. Pn =
1+n 2
+
2+n2
+…+
n + n2
1 2 n 1+ 2 +…+n n (n + 1)
Now, Pn < + + … + = =
1 + n2 1 + n2 1 + n2 1 + n2 2 1 + n2 ( )
Also, Pn >
1
+
2
+
3
+…+
n
=
1+ 2 +…+n
=
n n+ 1 ( )

n+n2
n+n2
n+n2
n+n2
n+n 2
(
2 n + n2 ) ( )
n (n + 1) n (n + 1)
Thus, < Pn <
(
2 n + n2 ) (
2 1 + n2 )
n (n + 1) n (n + 1)
⇒ lim
n→∞
(
2 n+n 2
)
< limPn < lim
n→∞ n→∞
(
2 1 + n2 )
 1  1
1 1 +  1  1 +   
n n
⇒ lim  < limPn < lim 
1   1 

n→∞ n →∞ n →∞
2  + 1   2  2 + 1  
n  n 
1 1 1
⇒ < limPn < ⇒ limPn =  
2 n→∞ 2 n→∞ 2

EVALUATION OF TRIGONOMETRIC LIMITS :
If θ is small and is measured in radians then
sinθ
(i) Lim =1
θ→0 θ

Proof :
θ
Consider a circle of radius r. Let O be the
centre of the circle such that ∠AOB = θ
where θ is measured in radians and it is very
small. Suppose the tangent at A meets OB
produced at P. From figure, we have
Limits

25.
Area of ∆OAB < Area of sector OAB < Area of ∆OAP
1 1 1
( )
2
⇒ OA×OB sinθ < OA θ < OA×AP 
2 2 2
1 2 1 1
⇒ r sinθ < r 2θ < r 2 tanθ [In ∆OAP, AP=OA tanθ]
2 2 2
sinθ
⇒ sinθ < θ < tanθ ⇒ 1 > θ > cosθ
sinθ sinθ
⇒ 1 > lim > limcosθ or, limcosθ < lim <1
θ→0 θ θ→0 θ→0 θ→0 θ
sinθ sinθ
⇒ 1 < lim < 1 ⇒ lim = 1 (By Sandwich Theorem)
θ→0 θ θ→0 θ
tan θ
(ii) lim =1
θ→0 θ
tan θ sin θ 1 sin θ 1
We have lim = lim = lim .lim = 1 
θ→0 θ θ→ 0 θ cos θ θ→ 0 θ θ→ 0 cos θ
sin ( θ − a )
(iii) lim =1
θ→a θ−a
sin ( θ − a ) sin ( a + h − a ) sinh
We have lim = lim = lim = 1 
θ→a θ−a h→ 0 (a + h − a ) h→ 0 h

tan ( θ − a )
(iv) lim =1
θ→a θ−a
sin−1 θ
(v) lim =1
θ→0 θ
tan−1 θ
(vi) lim = 1 
θ→0 θ

Note :

Let [·] denotes greatest integer function


 sinx   sinx 
(i)  Lim   = 0 (ii)  Lim =1
x →0
 x   x →0 x 
 tanx   sin−1x 
(iii)  Lim   = 1 (iv)  Lim  =1
x →0
 x  x →0
 x 
 sin−1 x 
(v)  Lim =1
 x →0 x 
Limits

26.
sin2x
Q.1 Evaluate lim
x →0 5x
 .

sin2x  2   sin2x  2 sin2x 2 2


Sol. We have lim = lim     = lim = ( 1) =
x →0 5x x →0 5
   2x  5 x →0 2x 5 5
 sin2x 
 lim
 x →0 2x
= 1

1 − cosx
Q.2 Evaluate lim
x →0 x2
.

x
2sin2  
1 − cosx 2 = 1 × 1 = 1
Sol. lim
x →0 x 2
= lim
x →0
x
2
2 2
4 
2

Note :

1 − cosx 1
lim = is a standard limit and
x →0 x2 2
should be remembered.

1 − cos5x
Q.3 Evaluate lim
x →0 3x2
.

1 − cos5x  25  1 − cos5x 25 1 − cos5x


Sol. lim
x →0 3x 2
= lim  
x → 0
 3  (5x)
2
  =
3
lim
x → 0 (5x)2
25 1 25
= × =

3 2 6

x3 + x2 − 2
Q.4 Evaluate lim
x→1 sin ( x − 1)
.

x3 + x2 − 2 (
x3 − 1 + x2 − 1 ) ( )
Sol. lim = lim
( ) x→1 sin ( x − 1)
x → 1 sin x − 1

( x − 1) ( x )
+ 1 + x + ( x + 1) 
2
 = lim ( x − 1) × x2 + 2x + 2 = 1 × 1 + 2 + 2 = 5
lim
sin ( x − 1) ( )
( ) ( )

x→1 x → 1 sin x − 1
Limits

27.
1 + tanx − 1 + sinx
Q.5 Evaluate lim
x →0 x3
.

1 + tanx − 1 + sinx ( 1 + tanx + 1 + sinx ) 


Sol. lim ×
1 + sinx )
x →0 x3 ( 1 + tanx +

tanx − sinx
lim


x →0
x 3
( 1 + tanx + 1 + sinx )
sinx ( 1 − cosx ) 1
lim ×


x →0 x cosx
3
( 1 + tanx + 1 + sinx )
lim 
 sinx 
× lim
( 1 − cosx ) × lim 1


x →0
 x  x →0 x2 x →0
cosx ( 1 + tanx + 1 + sinx )
x
2sin2  
= lim
sinx
× lim  2  × lim 1
 


x →0 x x → 0  x2  x →0 cosx
4×  ( 1 + tanx + 1 + sinx )
4
1 1 1
= 1× × 1× =  
2 1 ( 1 + 1) 4

πx
Q.6 Evaluate lim ( 1 − x ) tan
x →1 2
.

 πx   πx 
sin   sin  
 2  = lim 1 − x
Sol. lim ( 1 − x ) ( ) π 2 
x →1  πx  x →1 sin ( 1 − x )
cos  
 2  2
π
(1 − x) 1  πx 
= lim 2 × × sin  
x →1  π   
sin  ( 1 − x )   
π  2 

2  2
2 2
= 1× ×1 =  
π π

Limits

28.
1 − cos ( 1 − cosx )
Q.7 Evaluate lim
x →0 sin4 x
 .

 1 − cos x   x
2sin2   2sin2  sin2 
 2  = lim  2  
Sol. lim
x →0 sin4 x x →0 sin4 x
2 4
  x    x   x 
2sin  sin2    × sin2    ×   2

  2    2   2 
= lim 2 4
 
x →0
 2  x  x
sin    ×    ×  sin x
4

  2    2 
1 1
= 2 × 1 × 1 × =
16 8

1 − cosx cos2x
Q.8 Evaluate lim
x →0 tan2 x
.

1 − cosx cos2x 1 + cosx cos2x ( )


Sol. lim
x →0 tan2 x
×
1 + cosx cos2x ( )
= lim
( )(
1 − cos2 x 2cos2 x − 1 )× 1
x →0  tan x2

 x2 × x 
2 (1 + cosx cos2x )
= lim
(
sin2 x  1 +  2cos x
2

×
1 )
x →0 x2 1 + cosx cos2x  tan2 x 
 2 
 x 
1 +  2 1 3
= 1× × =  
1+ 1 1 2

1  2x 
Q.9 Evaluate lim sin−1 
x →0 x 2 
1+ x 
.

 2x 
Sol. We know that, sin−1 
 1 + x 2 

= 2tan−1x,  for − 1 ≤ x ≤ 1

1  2x  2tan−1x
⇒ lim sin−1  2 
= lim =2
x →0 x
 1 + x  x →0 x
Limits

29.
 x + 1 π
Q.10 Evaluate lim x  tan
−1
− 
x →∞ x + 4 4

 x + 1 π
Sol. We have lim x  tan−1
x →∞
 x
− 
+ 4 4
 x+1 
 −1 x + 1 −1 
 x + 4 − 1
= lim x  tan − tan 1  = lim x tan 
−1

 x+4  x →∞ 1+ x + 1 
x →∞

 x+4
  −3  
 tan−1 
 −3   2x + 5    −3 
= lim xtan−1  = lim   x
x →∞  2x + 5  x→∞  −3   2x + 5 
 2x + 5 
 
 −1  −3    
 tan    −3 
 2x + 5   −3x  −3 3
= lim   lim
  = 1 × lim   = 1× =−
x →∞  −3  x →∞
 2x + 5  x →∞
2 + 5  2 2
   
 2x + 5   x 

1 + cos 2x
Q.11 Evaluate lim
x→
π
( π − 2x )
2
 
2

1 + cos 2x  0 
Sol. We have, lim
x→
π 2 
( π − 2x )  0
form 

2

π 
1 + cos 2  + h 
= lim  2  = lim 1 + cos ( π + 2h) = lim 1 − cos 2h  
h→ 0
 π 
2 h→ 0 4h2 h→ 0 4h2
 π − 2  + h 
 2 
2
2 sin2 h 2  sinh  1
= lim =  lim  =  
h→ 0 4h 2
4 h → 0 h  2

2 cos x − 1
Q.12 lim
x→
π
4
cot x − 1

( 2 cos x − 1 sin x )
Sol. lim
x→
π cos x − sin x
4

lim
( )( 2 cos x + 1) × (cos x + sin x ) . sin x 
2 cos x − 1

x→
π
4 ( 2 cos x + 1) ( cos x − sin x ) ( cos x + sin x )
Limits

30.
= lim 
(2 cos 2
x − 1)
×
(cos x +  sin x ) . sin x
π cos x − sin x
2 2
( 2 cos x + 1 )

x→
4

 1  1
 × 2  . 
(cos x +  sin x ) . ( sin x ) 2  2 = 1
( )×
= lim 1 = 
π
( 2 cos x +  1 )  1  2

x→
4
 2× + 1
 2 

cot2x-cosec2x
Q.13 lim x →0 x
cos2x 1
 −  
sin2x sin2x
Sol. lim
x →0 x

= lim
− ( 1 − cos2x )
= lim
(
− 2sin2 x )  
x →0
( sin2x ) ×  x x →0
( 2sinx cosx ) x
 sinx  1
= lim−  × = ( −1) × 1 = −1
x →0
 x  cosx

tan2x - x
Q.14 lim x →0 3x-sinx
 tan2x − x  tan2x 1
  −
 2x  2x 2
Sol. lim
x →0  3x − sinx 
= lim
x →0 3 1 sinx 

  −  
 2x  2 2 x 
1 1
1−
= 2 = 2 = 1
3 1 1 2
− ( 1)
2 2

EVALUATION OF EXPONENTIAL AND


LOGARITHMIC LIMITS :
In order to evaluate these type of limit, we
use the following standard results.
ax − 1
1. lim = log ea 
x →0 x
Limits

31.
Proof :

1 + (
 x loga ) x2 (loga ) 
2

+ + ...  − 1
 1! 2! 
loga x (loga )
 2

a −1
x
  
lim   = lim = lim + + ...  = log ea
x →0 x x →0 x x →0  1! 2! 

 

ex − 1
2. lim =1 (replace a by e in the above proof)
x →0 x
log ( 1 + x )
3. lim = 1 
x →0 x
Proof :
x2 x3
log ( 1 + x ) x−
2
+
3
− ...  x x2 
lim = lim = lim  1 − + ...  = 1
x →0 x x →0 x x →0
 2 3 

etanx − ex
Q.1 lim
x →0 tanx − x

(etanx
− ex ) = lim e (e x tanx − x
−1 )
Sol. lim
x →0
( tanx − x ) x →0
( tanx − x )
= e° × 1 = 1

2
ex − cosx
Q.2 lim
x →0 x2
x
(e x2
)
− 1 + ( 1 − cosx ) ex − 1
2 2 sin2
2
Sol. lim
x →0 x2
= lim
x →0 x2
+ lim
x →0 x2
  x 
 sin2   


= 1 + lim  2   2
x →0 2
4 ×  x  
  
  2  
2 3
= 1+ ×1=  
4 2
Limits

32.
esin2x − esinx
Q.3 lim
x →0 x
esin2x − esinx esin2x − 1 − esinx + 1
Sol. lim
x →0 x
= lim
x →0 x
esin2x − 1 esinx − 1
lim −
x →0 x x
 e sin 2x − 1 sin2x e sin x − 1 sin x 
= lim  · ·2 − ·
x → 0  sin2x 2x sin x x 

=1×1×2–1×1
=1

ex + e− x − 2
Q.4 lim
x →0 x2
1
( )
2
ex + −2 ex − 1
ex
Sol. lim
x →0 x2
= lim
x →0
ex x2 ( )
2
 ex − 1  1 1
lim   × x = 1× = 1
 x  e 1

x →0

log ( x − a )
Q.5 lim
x →a
(
log ex − ea )
log ( x − a ) log ( x − a )
Sol. lim
x →a
(
log ea ex −a − 1  )
= lim
x →a  ex −a − 1 
 
  loge + log 
a
× ( x − a )
 x−a 
log ( x − a )
= lim
x →a a + log ( x − a )

= lim
1
=
1
=1
x →a a 0+1
+1
log ( x − a )
Limits

33.
1

ex − 1
2

Q.6 lim
x →∞ 2arc tan x2 − π

 x12 
1
 e − 1 
ex − 1
2
 
Sol. lim
x →∞ π 
= lim
x →∞ π − 2 cot −1 x2 − π
2  − cot −1 x2  − π
 2 
1
put x =
y
(e y2
−1−1 )
ey − 1 ( 2

)
lim = lim
y →0 1
−2cot −1 2
2 y →0  tan−1 y 2 
 ×y
2
y  y 2

 ey2 − 1 
 
−1  y2 
= lim   = −1 × 1 = −1  
2 y → 0  tan y 
−1 2
2 1 2
 
 y
2

(
sin ex −2 − 1 ) 
Q.7 lim
x →2 log ( x − 1)

(
sin ex −2 − 1 ) 
Sol. lim
x →2 log ( x − 1)
 e( x −2) − 1 
sin  ( x − 2)
 ( x − 2 ) 
= lim  
x →2
log  1 + ( x − 2 ) 
× ( x − 2)
( x − 2)
= lim
(
sin 1 × ( x − 2) ) = 1 
x →2 1 × ( x − 2)

( )
cos xex − cos xe− x ( )
Q.8 lim
x →0 x 3

x  x  x2 x
2sin  ex + e− x
2
( )  2
(
  sin  e− x )
− ex   ×  
 4
(
e + e− x )(e −x
− ex )
Sol. lim
x   x −x x 
 
( ) ( )
x →0
x3 ×  ex + e− x   ×  2 e − e 
2   
Limits

34.
x  x 
sin  ex + e− x 
 2 
(  2
)
sin  e− x − ex 
 ex + e− x ( ) ( )( 1 − e )
2x

= lim × lim × lim


x →0 x x
e + e− x(
x →0 x −x
)
e − ex
x →0
ex
( ) ( ) (2x )

2 2
( 1 + 1) × −1 = −2
= 1× 1×
1
( )

Q.9 lim
(1 − 3 x
− 4x + 12x )
x →0
( 2cosx + 7 ) − 3
(4 x
)(
− 1 3x − 1 )× 2cosx + 7 + 3
Sol. lim
x →0
2cosx + 7 − 3 2cosx + 7 + 3

lim
(4 x
−1 ) (3 x
−1 )× x ×( 2
2cosx + 7 + 3 )
x →0 x x ( 2cosx + 7 − 9)

lim
(4 x
−1 ) (3 x
−1 )× x ×( 2
2cosx + 7 + 3 )
x x −2 ( 1 − cosx )

x →0

 x2 
(4 x
−1 ) (3 x
−1 )×  
4
lim × ( 2cosx + 7 + 3 )
x x
( − ) sin 2x
x →0
2

= (ln4 )(ln3 ) × ( −1) × ( 9+3 )


= ( − ) 2 (ln2 )(ln3 ) 6

= −12 (ln2 )(ln3 )

π
− tan−1 x 1
Q.10 If lim 4
x →1 esin(ln x ) − xn
exists and has the value equal to , then find n.
8

Sol. Apply L’ Hospital rule


1

1 + x2 1
lim  sin(ln x )
x →1 e cos (ln x )
=
8
− nx n− 1
x
1

2 1 1 1

⇒ = ⇒ =
1−n 8 n−1 4 ⇒ n = 5
Limits

35.
( f ( x ))
g (x )
LIMITS OF THE FORM Lim
x →a

Form : 0°, ∞°  
g(x)
Let L = lim f ( x )
x →a
( )
g(x) 


⇒ log eL = log e lim f ( x )
 x → a  ( )
= lim g ( x ) log e f ( x ) 
x →a

Form : 1∞  

x
1.
1
 1
lim ( 1 + x ) x = e  or lim  1 +  = e
x →0 x →∞
 x


1
Proof: Let y = lim ( 1 + x ) x
x →0

take ln on both side


1
ln y = lim ln ( 1 + x ) x
x →0

ln ( 1 + x )
ln y = lim
x →0 x
ln y = 1
y=e
g(x)
2. L = lim f ( x ) if lim f ( x ) = 1 and lim g ( x ) = ∞
x →a x →a x →a
g(x)
Then L = lim f ( x )
x →a

1
( )
( ( ))
f ( x ) − 1 ×g ( x )
= lim 1 + f ( x ) − 1 f(x)− 1
 
x →a
lim (f ( x ) − 1 ) × g ( x )
  1
  x →a
( )
lim (f ( x ) − 1 ) × g ( x )
= lim  1 + ( f ( x ) − 1 ) f (x )− 1
  = e x →a
x →a 
 
Limits

36.
x+4
x + 6
Q.1 Evaluate lim 
x →∞

 x+ 1
.

 x + 6 − ( x + 1) 
lim ( x + 4 )  
 ( x + 1) 
Sol.
x →∞
e  
 
 4
1+
5x  x 
lim
( x +  4 ) × 5 lim 
x →∞ x  1

x →∞ ( x + 1)  1+ x 
=e =e = e5×1 = e5

x+3
 x + 2
Q.2 Evaluate lim 
x →∞
 x+ 1
 .

 x+2   x+3 
lim ( x + 3 )  − 1 lim   

Sol.
 x+1   x+1 
e =e
x →∞ x →∞

 3
x  1+ 
x
lim   
x →∞  1
x  1+ 
 x
=e  
= e1 = e

Evaluate lim ( 1+x )


cosecx

Q.3 x →0
.

x x
lim
 
1 sinx
 
1 x → 0 sinx
lim ( 1+x ) = lim ( 1 + x ) x  = lim ( 1 + x ) x 
cosecx

Sol. x →0 x →0
  
x →0

= e1 = e 

( )
cotx
Q.4 Evaluate lim cosx
x →0
.

( )
cotx
Sol. lim cosx
x →0

cosx − 1
 1
 tanx

x →0

(
= lim  1 + ( cosx − 1) ) cosx − 1


cosx − 1
1 lim cosx − 1
  x →0 tanx
= lim (1 + ( cosx − 1 )) cosx − 1 
lim cosx

= ex →0 sinx
 
 x →0 
cosx − 1  cosx − 1 
lim cosxsinx lim  .cosx.sinx 
= ex → 0 =e  
sin2 x x → 0  1− cos2 x 

sinxcosx
− lim
=e x →0 1+ cosx
= e0 = 1 
Limits

37.
 sinx 
 
 sinx  x −sinx 
Q.5 Evaluate lim 
x →0
 x 
  

sinx sinx 1 1
Sol. Since lim = 1 and lim
x →0 x − sinx
= lim =
1− 1
= ∞ 
x →0 x x →0  x 
 sinx − 1
 sinx 
   sinx  sinx  sinx
 sinx  x −sinx  lim 
x
− 1  
 x − sinx 
− lim 1
⇒ lim  =e  =e = e−1 =  
x →0 x →0 x

x →0
 x  e
2
 a x + bx + cx  x
Q.6 Evaluate lim   ; ( a,b, c > 0 ) .
x →0
 3 
2
 a x + bx + cx  x
Sol. lim
We have x →0 
 3 
 a x +bx + cx  2 2  a x +bx + cx − 3 
lim   − 1 lim  
3 x 3 x → 0 x
=e =e
x → 0 

2  a x -1 bx -1 cx -1  2  a x -1 bx -1 cx -1 
lim   +   +    lim    + lim    + lim   
3 x → 0  x x x  3 x → 0 x x →0 x x →0 x 
=e =e 
 
2 2
 ln( abc ) 2
2
  lna +lnb +lnc 
= ( abc ) 3
3 3 ln( abc ) 3
=e =e =e

nx
 x1 1 1 1

Q.7 Evaluate lim 
x →∞
 1 + 2 x + 3 x + ... + nx
n

 ,  n ∈ N .
 
 
1
Sol. Put x =
y
nx
 x1 1 1 1

 1 + 2 x
+ 3 x
+ ... + n x

lim  
x →∞


n 

n
 1y + 2y + ... + ny  y
= lim    
y →0
 n 
n  1y + 2y + ... +ny −n 
lim    
y →0 y  n 
=e  


lim  
( ) (
 1y − 1
+
2y − 1 )
+ ... +
(
ny − 1 ) 

y →0
 y y y 
=e  
 
ln1 +  ln2 +  ... +  ln n
=e
Limits

log n!
= e e = n!
38.
c + dx
 1 
Q.8 Evaluate lim  1 +
x →∞


a + bx 
where a, b, c and d are positive.

c + dx
 1 
Sol. lim  1 +
x →∞


a + bx 
 1 
(
 lim c + dx  1+ ) − 1
 a +bx 
=e x→∞

 c
x d+ 
x
 lim 
 1  x →∞  a
(
 lim c + dx  )  x b + 
 a +bx   x
= e x →∞ =e
d
= eb
π
sec2
 π  2 −bx

Q.9 Evaluate lim  sin2


x →0  2 − ax 
 .

π
sec2
 π  2 −bx

Sol. lim  sin2


x →0  2 − ax 
 2 π  2 π  
 lim sec  sin  − 1
 2 −bx    2 − ax  
= e  x→0
 
2
  π 
 cos 
 2 − ax  

lim − 
x→0
()
 π 
 cos 
 2 −bx  
=e 
 
2
  π  
 − sin
 2 − ax 
( )(
−2
×π − 2 − ax × − a ) ( )
()
− lim 
x→0  π 


 − sin
 2 −bx 
( )(
−2
π − 2 −bx × −b ) ( ) 
=e  
 
2
a

=e b2

L’ HOSPITAL’S RULE FOR EVALUATING LIMITS :

Rule :

f (x ) 0 ∞
If  lim takes or   form, then,
x →a g (x ) 0 ∞

f (x) f′ ( x )
 lim = lim  
x →a g (x) x →a g′ ( x )
Limits

39.
Q.1 Evaluate lim  log tan2 x ( tan2 2x ) .
x →0

(
log tan2 2x  ∞  )
Sol. L = lim
x →0
(
log tan x 
2  form 
∞  )
Using L′ hospital Rule


1 
2tan2x sec2 2x  × 2
tan 2x
2

We have L = lim    
x →0 1
2tanxsec x2

tan2 x
 1   1 
2   
sin2xcos2x  sin2xcos2x  1
= lim  = lim  = 1lim = 1 
x →0 1 x →0  1  x → 0 cos2x
sinxcosx  
 sin2x 

Evaluate lim+ xm (logx ) ,m,n ∈ N .


n

Q.2 x →0

(logx)
n
∞ 
(log x)
n

Sol. lim+ x = lim  ∞ form


m

x →0 x → 0+ x −m

(n− 1) 1
n (logx )
= lim+ x   (using L′ hospital Rule)
x →0 −mx −m− 1
(n− 1)
n (logx ) ∞ 
= lim+   form 
x →0 −mx −m
∞ 
(n − 2 ) 1
n (n − 1)(logx )
= lim+ x   (using L′hospital Rule)
x →0
( −m ) x 2 −m − 1

(n − 2 )
n (n − 1)(logx ) ∞ 
= lim+  form 
x →0 mx 2 −m
∞ 
n!
= lim+ = 0  (differentiating Nr and Dr n times)
( −m)
n
x →0
x −m

sin−1x − tan−1x
Q.3 Evaluate lim
x →0 x3
 .

sin−1x − tan−1x
Sol. lim
x →0 x3
 
Limits

40.
= lim
(1 + x ) − 2
1 − x2
  (Using L′ hospital’s Rule)
x →0
( )
3x2 1 − x2 1 + x2

(1 + x ) − (1 − x ) ×
2
2 2
1
= lim   (Rationalizing)
x →0
3x 1 − x ( 1 + x ) ( 1 + x ) +
2 2 2 2
1 − x2

x4 + 3x2 1
= lim ×  
x →0
3x2 1 − x2 1 + x2 ( ) (1 + x ) + 2
1 − x2

x2 + 3 1 1
= lim × =  
x →0
3 1 − x2 1 + x2 ( ) (1 + x ) + 2
1 − x2 2

LIMITS OF FUNCTIONS HAVING BUILT IN LIMIT WITH THEM :


Examples :

0, 0<a<1  ∞,           0<a<1


 n 
Lim  a =  1,
n
a=1 ,  Lim  a =  1,          a=1
n→∞ n→−∞
 ∞, a>1 0,                  a>1
 

n
tan πx2 + ( x + 1) sin x
Q.1 f ( x ) = Lim n
, find Lim f x . ()
n→∞
x2 + ( x + 1 ) x →0

 sin x, x >0

Sol. f ( x ) =  tan πx 2
, − 1 <x < 0
 

 x2
Lim+ f ( x ) = Lim+ sin x = 0
x →0 x →0

tan πx2
Lim− f ( x ) = Lim− =π
x →0 x →0 x2
⇒ LHL ≠ RHL ⇒ Lim f ( x ) = DNE .
x →0

cos πx  - x 2n sin ( x  - 1 )


Q.2 f ( x ) = lim , find lim f ( x ) .
n→∞ 1 + x 2n+ 1  - x 2n x→1

cos πx − x2n sin x − 1 ( )


Sol. ( )f x = lim
n→∞ 1+ x⋅x 2n
−x 2n

when 0 < x2 < 1, lim x 2n


→ 0 
Limits

n→∞

41.
∴ f(x) = cosπx

when x2 = 1,  Lim  x2 → 1 
n→∞

cos πx − sin ( x − 1) cos πx − sin ( x − 1)


∴ f (x) = =
1+x−1 x
when x2 > 1

Lim  x2n → ∞
n→∞

1
∴ Lim →0
n→∞ x2n
 cos πx 
− sin ( x − 1 )
 x 2n  0 − sin ( x − 1 ) − sin ( x − 1 )
f ( x ) = Lim = =
n→∞ 1 0+x −1 x −1
+ x − 1
x 2n

 cos πx,                     0 < x2 < 1

 cos πx − sin ( x − 1)
f (x) =  ,             x2 = 1 
 x
 − sin ( x − 1)
 ,                        x2 > 1 
 x−1
− sin ( x − 1)
Lim− f ( x ) = Limcos πx = −1, Lim f ( x ) = Lim = −1
x →1 −
x →1 +
x →I +
x →I ( x − 1)
∴ Lim f ( x ) = 1 .
x →1

(
tan f ( x ) ) (
sin f ( x ) )
e −e
Q.3 Let f ( x ) = min (1, x , x
2n 2n + 1
) , n ∈ N . The value of Lim , is equal
x →0
( )
tan f ( x ) − sin f ( x ) ( )
to
(A) 0 (B) 1 (C) 2 (D) does not exist.

Sol. (B)
f ( x ) = min. ( 1, x 2n
)
, x2n+ 1 ,n ∈ N

 x 2n+ 1 , x ≤ 1
=
 1, x >1
Limits

42.
(
tan f ( x ) ) (
sin f ( x ) )
e −e
2n+1
y=x
y
Lim  
2n
y=x2n y=x

x →0
( )
tan f ( x ) − sin f ( x ) ( )
(e )
y=1
(
sin f ( x ) ) ( )
tan f ( x ) − sin f ( x )( )
e −1
= Lim

x →0
(
tan f ( x ) − sin f ( x ) ) ( ) -1 O 1

 tan( x ) −sin( x ) 2n + 1 2n + 1

e − 1
 = Lim e − 1 = 1
y -1
= Lim 


x →0
( ) ( )
tan x2n+ 1 − sin x2n+ 1 y →0 y

where y = tan ( x ) − sin ( x ) 2n+ 1 2n+ 1


tan πx
sin ( πx4 ) + ( x + 2) .
n

x + 1 , then Lim f x is equal to


Q.4 Let f ( x ) = Lim n→∞
1 + ( x + 2) − x
n 4
( )
x →−1

22
(A) π (B) (C) 1 (D) Non existent
7

Sol. (A)
tan πx
sin ( πx4 ) + ( x + 2)
n

( x + 1)
Let f ( x ) = Lim f ( x )
n→∞
1 + ( x + 2) − x4
n

 sin ( πx4 )

 ,      x < −1
f (x) =  1 − x
4

 tan πx ,          x > −1


 x + 1

Lim f ( x ) = Lim+
tan πx
= Lim+
( tan π ( 1 + x ) ) = π
x →−1 x →−1 ( x + 1) x→−1 (1 + x)
Lim− f ( x ) = Lim−
sin ( πx4 )
= Lim−
(
sin π ( 1 − x4 ) )=π

x →−1 x →−1
(1 − x ) 4 x →−1
(1 − x ) 4

Lim− f ( x ) = π

x →−1
Limits

43.
EXPANSION OF FUNCTION :
Expansion of function like Binomial expansion,
exponential and logarithmic expansion,
expansion of sinx, cosx, tanx should be
remembered & are given below:
x lna x2ln2a x3ln3a
(i) ax = 1 + + + + ...  (a > 0)
1! 2! 3!
x x2 x3
(ii) ex = 1 + + + + ...∀x ∈ R 
1! 2 ! 3 !
x2 x3 x4
(iii) ln ( 1 + x ) = x − + − + ...  for − 1 < x ≤ 1
2 3 4
x 3 x5 x 7  π π
(iv) sinx = x − + − + ...   − < x < 
3! 5! 7 !  2 2
x 2 x 4 x6  π π
(v) cosx = 1 − + − + ...  x ∈  − , 
2! 4! 6!  2 2
x3 2x5  π π
(vi) tanx = x + + + ...  x ∈  − , 
3 15  2 2
x 3 x5 x 7
(vii) tan−1x = x − + − + ... 
3 5 7
12 3 12.32 5 12.32.52 7
(viii) sin−1x = x + x + x + x + ...
3! 5! 7!
x2 5x4 61x6
(ix) sec−1x = 1 + + + + ... 
2! 4! 6!

Q.1 Evaluate the following limit:


ex − 1 − x
(i) Lim
x →0 x2
 x2 
 1 + x + + ...  − 1 − x
e − 1− x
x
2! 1 1
Sol. Lim = Lim   = =
x →0 x 2 x →0 x 2
2! 2

x − sinx
Q. (ii) Lim
x →0 x3
 x 3 x5 
x − x − + − ... 
x − sinx  3! 5!  = 1 = 1
Sol. Lim
x →0 x 3
= Lim
x → 0 x 3
3! 6
Limits

44.
ex − e− x − 2x
Q. (iii) Lim
x →0 x3
 x2   x2   2x3 
 1 + x + + ...  −  1 − x + …  − 2x  2x + + ...  − 2x
e − e − 2x
x −x
2! 2!  3! 2 1
Sol. Lim = Lim    = Lim   = =  
x →0 x 3 x → 0 x 3 x → 0 x 3
3! 3
 x2   x2   2x3 
1 + x + + ...  −  1 − x + … − 2x  2x + + ...  − 2x
2! 2!  3! 2 1
m   = Lim   =  
→0 x 3 x → 0 x 3
=
3! 3

x − tanx
Q. (iv) Lim
x →0 x3
 x3 2x5 
x − x + + + ... 
x − tanx  3 15  = −1  
Sol. Lim
x →0 x 3
= Lim
x → 0 x 3
3

tan2 x − x2
Q.2 Evaluate Lim
x →0 x2 tan2 x
 .

tan2 x − x2 ( tanx − x )( tanx + x )  


Sol. Lim
x →0 x2 tan2 x
= Lim
x →0 x2 tan2 x
 1 3   tanx 
 x + x + ... − x  ( tanx + x )  + 1
3 1 x  = 1  1 + 1 = 2
= Lim   =   

x →0
4  tanx 
x 
2
3  tanx 
2
3 1  3
  
 x   x 
3
ex − 1 − x3
Q.3 Evaluate Lim
x →0 sin6 2x
 .

 x6 
 1 + x 3
+ + ...  − 1− x
3

2 !
3
e − 1− x
x 3

Sol. Lim = Lim    


x →0 sin 2x
6 x 0 sin 2x
6
. ( 2x )
→ 6

( )
6
2x

1 x3 1
+ + ...
2! 3! 2 1
Lim = 6 =  
x →0 2 6
2 128


Limits

45.
1 1
Q.4 Evaluate Lim − .
(sin x ) x2
x →0 2
−1

1 1 sin2 θ − θ2
Sol. Put x = sin θ ⇒ Lim
θ→0 θ2
− = Lim
sin2 θ θ→0 θ2 sin2 θ
 

( sin θ − θ ) ( sin θ + θ ) sin θ − θ 1


Lim = 2.Lim =−  
θ→0 θ 4 θ→ 0 θ 3
3

Don’t do it :
1 1 1 x2 1 1 1
Lim − = Lim . − = Lim 2 − 2 = 0 , is wrong.
(sin x ) x2 x →0 x 2
( ) x x →0 x x
x →0 2 2 2
−1
sin−1x

 1
Q.5 Evaluate Lim  x − x2 n  1 +  .
x →∞
 x 
 1 1
Sol. Lim  x − x2 n  1 + 
x →∞
 x
put x =
y
 y2 y3 
y − y − + − …
= Lim −
1 ln ( ) = Lim
1 + y y − ln ( ) = Lim  2 3 !  = 1  
1 + y

y →0 y y y2 y2 2
2 y → 0 y →0

Don’t do it :
 1 ln ( 1 + y ) 1  1 1 ln ( 1 + y )
Lim  − .  = − = 0 as Lim = 1 , is not correct.
y →0  y y y y y y →0 y
 

Q.6 Evaluate the following limits :


2
ex − cosx
(i) Lim  
x →0 x2

(x ) + (X )
2 3
2 2
 x2 
1+ x 2
+ − 1 − + ... 
2! 3! 2!
2
ex − cosx   = 3 
Sol. Lim
x →0 x2
= Lim
x →0 x 2
2

esin2x − esinx
Q. (ii) Lim
x →0 x
 


( sin2x ) 
2

 1 + sin2x + + ...  − ( 1 + sinx + ... )


 2! 
esin2x − esinx  
Sol. Lim
x →0 x
= Lim
x →0 x
 

sin2x − sinx
Limits

= Lim = 2 - 1 = 1 
x →0 x
46.
1

(1 + x) x −e
Q.7 Evaluate Lim
x →0 x
 

 ln( 1+ x ) − x 
1 1 e e x − 1
(1 + x) x −e e x
ln( 1+ x )
−e
= Lim 
   ln ( 1 + x ) − x 
 .
Sol. Lim
x →0 x
= Lim
x →0 x x →0 
ln ( 1 + x ) − x   x2


  
 x 
 
 ln(1+ x) − 1 
e x − 1
   ln 1 + x − x  ( )
 −1 −e
= eLim Lim   = e 1 .  = ()
x →0 
ln 1 + x (  x →0
 ) x 2
  2 2

 x
− 1

Don’t do it :

(1 + x)
1/x
−e
Lim =l
x →0 x
y
Let x =
2
2 1

 y y  y2 y 1  y2 
 1 +  − e  1 + + y  −e  1+ + y − 1 
y  4
2 4 e 
−e
l = lim  = 2 ⋅ lim   = 2 ⋅ lim *
y →0 y y →0 y y → 0 y
2
 y2  1
 + y . y y
 4  y +1
e  
−e e4 −e e 4 − 1 1 2e e
= 2Lim = 2.Lim = 2.eLim . = =   
y →0 y y →0 y y →0 y 4 4 2
4
Note that mistake occurred at *.

Acosx + Bxsinx − 5
Q.8 If Lim
x →0 x4
 exists and finite. Find A & B and also the limit.

A cosx +  Bxsinx −  5
Sol. Let L = Lim
x →0 x4
 

 x2 x4   x3 
A 1 − + ...  + Bx  x − + ...  − 5
2! 4!  3!
= Lim     
x →0 x 4

( A − 5) + B − A2  x2 +  24
A B 4
− x
6
L = Lim    = finite value
x →0 x 4

A 5 A B −5
⇒ A = 5,B = = ,L = − =  .
Limits

2 2 24 6 24
47.
4 + sin2x + Asinx + Bcosx
Q.9 Let f ( x ) = . If Lim f ( x ) exists and finite find
x2 x →0

A, B and the limit.

4 + sin2x + Asinx + Bcosx


Sol. Let L = Lim
x →0 x2
 


( ) 
3
2x  x3   x2 x4 
4 +  2x − + ... + A  x − + ... + B  1 − + + ...
 3!   3!   2! 4! 
L = Lim  
x →0 x2

( 4 + B) + ( A + 2) x − B2 x 2
+ ...
L = Lim  
x →0 x2
⇒ B = – 4, A = –2, L= 2.

Q.10 An arc PQ of a circle subtends a central angle θ as shown. Let A(θ) be the area
between the chord PQ and the arc PQ. Let B(θ) be the area between the tan-
A ( θ)
gent lines PR and QR and the arc PQ. Find Lim  .
θ→0 B ( θ )

Sol.
A(θ) = Area of sector PCQP - Area of ∆PCQ
1 1 1
= r2 θ − r2 sin θ = r2 ( θ − sin θ )
2 2 2
B(θ) = Area of quadrilateral PCQR – Area of sector PCQP
= 2(Area of ∆CPR) – Area of sector PCQP
1 θ 1 r2  θ 
= 2  .r 2 . tan  − r 2θ =  2 tan 2 − θ
2 2 2 2
A ( θ) θ − sin θ
Lim = − Lim  
θ→0 B ( θ) θ→0 θ
2 tan   − θ
2
 θ3 θ4  1
θ −  θ  −    +    ...
 3! 4!  3 ! = 3.8 = 2 
= Lim =
θ→0   θ
3
 θ
5
 1 2.6
2 2.
      3.2 3
 θ  2  2
2  +    +    ... − θ
2 3 15 
 
 
Limits

48.
Q.11 Suppose that circle of equal diameter are packed tightly
in a rows inside an equilateral triangle. (The figure illus-
trates the case n = 4.) If A is the area of the triangle and
An  is the total area occupied by the circles in n rows
An
then Lim equals
n→∞ A
π π 3 π π
(A) (B)   (C) (D)
3 6 2 3 6

Sol. Let radius of each circle = r and side of triangle = a

( )
a = n − 2 2r + 2 r + r cot 30o( )

(
= r 2n − 4 + 2 + 2 3 )
= r ( 2n − 2 + 2 3 )

3 2 3 2
( )
2
A= a = r · 4 n − 1 + 3  
4 4

( )
2
A = 3r 2 n + 3 − 1  

n (n + 1 )
An = πr2 ( 1 + 2 + ... + n) = πr2
2
n (n + 1 ) 2 r r
πr r
An 2 r
Lim = Lim  
n→∞ A
( )
n→∞ 2
3r2 n + 3 − 1 30o 30o

 1
1 +  π ( 1 + 0)
π  n  π
= Lim 2
= 2
=
n→∞ 2 3
 3 − 1 2 3 ( 1 + 0) 2 3
1 + 
 n 
Limits

49.
Maths IIT-JEE ‘Best Approach’ (MC SIR) Limit

SOLVED EXAMPLE
 x4
 , x4
1. Evaluate the left and right hand limits of the function f(x) =  x4 at x  4.
0, x4

Sol. LHL of f(x) at x = 4 is
lim f (x)  lim f (4  h)
x  4 h 0

4 h  4 h
 lim  lim
h 0 4 h  4 h 0 h
h
 lim  lim  1  1
h 0  h h  0
RHL of f(x) at x = 4 is
lim f (x)  lim f (4  h)
x  4 h0

4h4
 lim
h0 4h4
h h
 lim  lim  lim 1  1
h0 h h 0 h h  0

cos[x], x  0
2. Let f(x) =  x  a, x  0 . Then find the value of a, so that lim f (x) exists, where [x] denotes the greatest
 x 0

integer function less than or equal to x.


Sol. Since lim
x 0
f(x) exists, we have
lim f (x)  lim f (x)
x 0  x 0

or lim f (0  h)  lim f (0  h)
h  0 h 0

or lim | 0  h |  a  lim cos[0  h]


h  0 h 0

or a = cos 0 = 1
 a=1

2
3. Evaluate lim x  x loge 2x  loge x  1
x 1 (x  1)

x 2  x log e x  log e x  1 0 
Sol. lim  0 form 
x 1 (x 2  1)  
log e (x)  x  1
 lim
x 1 x 1
log e 1  1  1 0  2
  1
1 1 2

4. Suppose ABC is a triangle such that BAC is a right angle. A point H is located at BC such that AH is
AC  AH
perpendicular to BC. If ABC = , AB = a, and the value of lim = 10, then the value of a
 0 
is equal to :
(A) 1 (B) 2 (C) 5 (D) 10

Get 10% Instant Discount On Unacademy Plus [Use Referral Code: MCSIR] 50
Maths IIT-JEE ‘Best Approach’ (MC SIR) Limit
A
Sol. AC = a tan  ; AH = a sin 
AC  AH  a tan   a sin  
Now, lim  lim  
 0   0
   a

= a(1 + 1) = 2a = 10  a = 5 Ans.
 C
B
H

x x x  x 4
5. Evaluate lim
x 1 x 1

x x x  x 4
Sol. lim
x 1 x 1
x1/2  x1/4  x1/8  x1/16  4
lim
x 1 x 1
 x1/2  1 x1/4  1 x1/8  1 x1/16  1 
lim     
x 1
 x 1 x 1 x 1 x 1 

1 1 1 1  xn  an 
     lim  na n 1 
2 4 8 16  x a x  a 

8  4  2 1

16
15

16

6. Evaluate lim x ( x  c  x )
x 

f (x)
Sol. The given expression is in the form   . So, we first write in the rational form . We have
g(x)

x ( x  c  x )( x  c  x )
lim x ( x  c  x )  lim
x  x  ( x  c  x)

x (x  c  c) c x   c
 lim ;  lim  Form  ;  lim
x  xc  x x  xc  x   x  c
1 1
x
[Dividing N' and D' by x]
c c
 
1 0 1 2

Get 10% Instant Discount On Unacademy Plus [Use Referral Code: MCSIR] 51
Maths IIT-JEE ‘Best Approach’ (MC SIR) Limit

7. Evaluate lim(4n  5n )1/n .


n

n
Sol. L = lim(4  5n )1/n
n

1/n
  4  n    4 n 
 lim 5 1     = 5     0 as n   
n 
  5     5  

2  3 cos x  sin x
8. Evaluate lim
x (6x  ) 2
6

2  3 cos x  sin x
Sol. L  lim
x (6x  )2
6


Let  x  h
6

   
2  3 cos   h   sin   h 
 L  lim 6  6 
h 0 2
   
6  6  h    
   

       
2  3  cos cosh  sin sinh    sin cosh  cos sinh 
 6 6   6 6 
 lim 2
h 0 36h

3 3 1 3
2  cosh  sinh  cosh  sinh
 lim 2 2 2 2
h 0 36h 2
2(1  cosh)
 lim
h 0 36h 2

h
2sin 2  
1 2  1
 lim 2
18 h 0 h 36

 sin x 
 
 sin x  x sin x 
9. Evaluate lim  
x 0  x 

sin x sin x 1 1
Sol. Since lim  1 and lim  lim  
x 0 x x 0 x  sin x x 0  x  11
  1
 sin x 
 sin x 
   sin x  sin x 
 sin x  xsin x  lim 
x
1 
 x sin x 
We have lim    e x  0
x 0  x 

Get 10% Instant Discount On Unacademy Plus [Use Referral Code: MCSIR] 52
Maths IIT-JEE ‘Best Approach’ (MC SIR) Limit
sin x
lim  1
e x 0 x
 e 1 
e

10. Evaluate lim log tan 2 x (tan 2 2x)


x 0

log(tan 2 2x)  
Sol. L = lim  form 
x 0 log(tan 2 x)  
Using L'Hopital's rule, we have

 1 
 2
2 tan 2x sec 2 2x   2
tan 2x
L  lim  
x 0 1
2 tan x.sec 2 x
tan 2 x

 1   1 
2  
 sin 2x cos 2x   sin 2x cos 2x 
 lim  lim
x 0  1  x 0  1 
 sin x cos x   sin 2x 

1
 lim 1
x 0 cos 2x

cos 4x  a cos 2x  b
11. If lim is finite, find a and b using expansion formula.
x 0 x4
cos 4x  a cos 2x  b
Sol. lim  finite
x 0 x4
Using expansion formula for cos 4x and cos 2x, we get
 (4x) 2 (4x) 4   (2x) 2 (2x) 4 
1     a 1   b
 2! 4!   2! 4! 
lim
x 0 x4

 32 2 
(1  a  b)  (8  2a)x 2    a  x 4  ....
or lim  3 3 
4
x 0 x
or 1+a+b=0
–8 – 2a = 0
Solving (i) and (ii) for a and b, we get
a = –4 and b = 3
32 2 32  8
Also, L =  a 8
3 3 3

Get 10% Instant Discount On Unacademy Plus [Use Referral Code: MCSIR] 53
Maths IIT-JEE ‘Best Approach’ (MC SIR) Limit
2
n  an 
12. Let an, bn  Q, such that 7  4 3    a n  bn 3 , for n  N. Then the value of  lim
 n  b
 is
n 

n
Sol.  74 3   a n  bn 3
n

 74 3   a n  bn 3

1 n n 1  n n
 an 
2 

74 3    7  4 3   and b n 
2 3 

74 3    7  4 3  
2
a  a 
 lim n  3   lim n   3 Ans.
n  b
n  n  b n 

  [12 (sin x) x ]  [22 (sin x) x ]  ......[n 2 (sin x) x ]  


13. lim  lim    where [.] denotes G.I.F. is :
x 0

n 
 n3  
2 4 1
(A) 0 (B) (C) (D)
3 3 3
Sol. x – 1 < [x]  x
 12(xin x)x – 1 < [12(sinx)x]  12 (sin x)x
22(xin x)x – 1 < [22(sinx)x]  22 (sin x)x
.
.
.
n2 (sin x)x – 1 < [n2 (sin x)x]  n2 (sin x)x
Add all
n
 x (n)(n  1)(2n  1) 
 (sin x)  n   [k 2 (sin x) x ]
6 n(n  1)(2n  1)
 3
  k 1
3
 3
(sin x) x
n n 6n
(sin x) x n(n  1)(2n  1)  n 1
As lim  (sin x) x
n  6n 3 3
n(n  1)(2n  1) 1
and lim 3
(sin x) x  (sin x) x
n  6n 3
1
 Required limit  xlim (sin x) x
 0 3

A  lim (sin x) x
x0

n sin x cot x
nA  lim  lim 2
x 0 1/ x x  0 1/ x

x2
 lim 0
x  0 tan x

 A = e0 = 1

Get 10% Instant Discount On Unacademy Plus [Use Referral Code: MCSIR] 54
Maths IIT-JEE ‘Best Approach’ (MC SIR) Limit
n
[k 2 (sin x) x ] 1
 xlim lim   Ans.
 0 n 
k 1 n3 3

1 1 n
14. Evaluate nlim 2
 2
 ...... 
 1 n 2 n n  n2
1 2 n
Sol. Pn  2
 2
 .... 
1 n 2 n n  n2
1 2 n
Now, Pn  2
 2
 ..... 
1 n 1 n 1 n2
1
 (1  2  3  ......  n)
1 n2
n(n  1)

2(1  n 2 )

1 2 n n(n  1)
Also, Pn    .....  
nn 2
nn 2
nn 2 2(n  n 2 )

n(n  1) n(n  1)
Thus, 2(n  n 2 )  Pn  2(1  n 2 )

 1  1
11   1 1  
 n  n
or lim  lim Pn  lim
n   1  n  n   1 
2   1 2  2  1
n  n 

1 1
or  lim Pn 
2 n  2
1
or lim p n 
n  2

 x1/3 
15. Let f(x) = 8x3 + 3x then the value of lim  1  is
x  f (8x)  f 1 (x)
 
1 1
(A) 1 (B) (C) (D) 2
2 4
1/3
f 1 (x) y  f (y)  1 1 1
Sol. Let   lim  lim  lim  3   . So required limit   2
x  x 1/3 y  (f (y))1/3 y 
 y  2 2   

2
ex x 2

16. Let f : R  (0, ) be such that f (x)   e x  e x  x  0 , then lim f (x) is


f (x) x 1

1
(A) 1 (B) (C) e (D) 2e
e
2 2
Sol. (f (x))2  (e x  e x )f (x)  e x .e x  (0)
2
(f (x)  e x )(f (x)  e x )  0
2
e x  f (x)  e x ; x  (0,1)
Get 10% Instant Discount On Unacademy Plus [Use Referral Code: MCSIR] 55
Maths IIT-JEE ‘Best Approach’ (MC SIR) Limit
2
e x  f (x)  e x ; x  (1, )
2
 lim(e x )  lim(e x )  e
x 1 x 1

Hence lim f (x)  e , by sandwich theorem.


x 1

tan(x)
sin(x 4 )  (x  2)n .
17. Let f (x)  lim x  1 , then lim f (x) is equal to
x 1
n  1  (x  2)n  x 4
2 2
(A)  (B) (C) 1 (D)  
7 7

 tan(x) 
 x  1 ; x  1
Sol. f (x)   4
 sin(x ) ; x  1
 1  x 4

tan x tan(   x)


 lim f (x)  lim  lim (As tan  = tan( + ))
x 1 x 1 x  1 x 1 x 1
tan (x  1)
 lim 
x 1 x 1
sin x 4 sin(  x 4 )
And lim f (x)  lim  lim (As sin  = sin( – ))
x 1 x 1 1 x4 x 1 1  x4
sin (1  x 4 )
 lim 
x 1 (1  x 4 )

Hence xlim f (x)   Ans.


1

18. Let f, g and h be monic polynomials of degree m,n and p respectively where m,n and p are prime
h(x) h(6x)
numbers (m < n < p). If lim 13
 1 and L = lim
x  f (2x).g(3x)
is non-zero finite then L equals
x  x
(A) 1 (B) 9 × 213 (C) 9 × 111 (D) 4 × 311
h(x)
Sol. Given, lim 1
x x13
 degree of h(x) = p = 13
h(6x)
Also, lim is non-zero finite quantity is L
f (2x)g(3x)
x 

Degree of Dr must be 13 ; hence degree of f(x) = 2 and degree of g(x) must be 11


613.x13
L  lim
x  2 2.x 2 .311.x 11

= 32 × 211 = 9 × 211 Ans.

Get 10% Instant Discount On Unacademy Plus [Use Referral Code: MCSIR] 56
Maths IIT-JEE ‘Best Approach’ (MC SIR) Limit
1
 f ( x)  x 1
19. Let f be a biquadratic function of x such that lim  3 
 3 , then find the value of |f(1)|.
x 0
 2x  e
Sol. Let f(x) = ax4 + bx3 + cx2 + dx + e
1  f (  x) 
We have, lim  3
 1  3
x 0 x
 2x 

 ax 4  (b  2)x 3  cx 2  dx  e 
 lim    6
x 0
 x4 
 a = –6, b = –2, c = d = e = 0
So, |f(1)| = |a + b| = 8 Ans.

20. In the figure lengths of AB and AC are both equal to 1, the length of BD is denoted as x and the length
x
of CE is denoted as y. BE is perpendicular to AC and ED is perpendicular to AB. The value of lim ,
 0 y
is
B
1 Dx
 y C
A
E

1
(A) 0 (B) 1 (C) 2 (D)
2
Sol. From given figure,
In AEB, AE = AB cos 
 1 – y = cos  y = 1 – cos 
Also, in ADE, AD = AE cos 
AD = cos2 [As, AE = cos ]
So, DB = x = AB – AD = (1 – cos2 )
x  1  cos 2  
Now, lim  lim    lim(1  cos )  1  1  2 Ans.
 0 y 0 1  cos 
  0

1
21. Let L  lim
n 
1 1 1 1 1 1 1 1 1
 .   .......n terms
2 2 2 2 2 2 2 2 2


Find the value of 2 3 cot L 
1  
Sol. If  cos  cos , where  
2 4 4

r   
 D  cos  cos cos 2 .....cos n 1
2 2 2

Get 10% Instant Discount On Unacademy Plus [Use Referral Code: MCSIR] 57
Maths IIT-JEE ‘Best Approach’ (MC SIR) Limit

  
 lim cos  cos cos 2 .....cos n 1
n  2 2 2
(sin 2). 1 sin 2
 lim . 
n  n   2
2 sin n 1
2
2 
 L 
sin 2 2
 
 
Hence 2 3 cot L  2 3  cot   0 Ans.
 2


 4  
(1  n 1 ) 2
22. If L    1  2  and N   1
. Then :
n 3  n  n 1 1  2n

15 1
(A) N = 2 (B) L–1 + N–1 = 8 (C) L  N  (D) LN 
2 3
 4  4  4
Sol. L   1  2  1  2  1  2  .......
 3  4   5 
 2   2  2  2   2  2  
 L   1   1  1   1   1   1   .....
 3   3  4  4   5  5  

 2  2  2  2    2  2  2  
 L   1   1   1   1   ......  1   1   1   .......
 3  4  5  6    3  4  5  
1 2 3 4  5 6 7 
 L       ......       ...... 
3 4 5 6  3 4 5 
1 2 1
L  
3 4 6
2
 1 (r  1)2
 
1  
n  2 
(r  1) 2 
r 1 r 1
N   lim  r  lim   lim  .
n 1  2  n  r 1 (r  2) n r 1 r(r  2) n  r 1 r r  2
1   r
 n

2 3 4  2 3 4 
 lim     ......       ...... 
n  1 2 3
  3 4 5 
N =2 Ans.

Get 10% Instant Discount On Unacademy Plus [Use Referral Code: MCSIR] 58
Maths IIT-JEE ‘Best Approach’ (MC SIR) Limit

GN BERMAN
Finding Limits. Comparison of infinitesimals
In Problems 245 to 267 find the limits.
2

245. lim
n 1
246. lim
 n  1
n  n n  2n 2
3 3

lim
 n  1   n  1 n 3  100n 2  1
247. 2 2 248. lim
n 
 n  1   n  1 n  100n 2  15n

4 4
1000n 3  3n 2 lim
 n  1   n  1
249. lim 250. 4 4
n  0.001n 4  100n 3  1 n 
 n  1   n  1
4 4

lim
 2n  1   n  1 3
n 3  2n  1
251. 4 4 252. lim
n 
 2n  1   n  1 n  n2
2

253. lim
3 2
n n
254. lim
 n2 1  n 
n  n 1 n  3
n6 1

n 3  2n 2  1  3 n 4  1 4
n5  2  3 n 2  1
255. lim 256. lim
n  4
n 6  6n 5  2  5 n 7  3n 3  1 n  5
n 4  2  n3  1

lim
n!
lim
 n  2 !  n  1!
257. n   n  1 ! n! 258. n   n  3!
1 1 1
1  ...  n
lim
 n  2 !  n  1! lim 2 4 2
259. n   n  2  !  n  1 ! 260. n  1 1 1
1    ...  n
3 9 3
1  1  2  3  ...  n n 
261. lim
n  n 2
1  2  3  ...  n  262. lim 
n  n2
 
2

 1  2  3  4  ...  2n   1 1 1 
263. lim   264. lim    ...  
n 
 n2 1  n  1  2
 2 3  n  1 n 
 1 1 1  2n  1
265. lim    ...   266. lim
n  1  3
 35  2n  1 2n  1  n  2 n  1

21/ n  1
267. lim
n  21/ n  1

Functions of the continuous argument


In Problems 268 to 304 find the limits

x2  5  x3  3x 1 
268. lim 2 269. lim 1
x 2 x  3 x0
 x 4 

Get 10% Instant Discount On Unacademy Plus [Use Referral Code: MCSIR] 59
Maths IIT-JEE ‘Best Approach’ (MC SIR) Limit

x x2  3
270. lim 271. lim
x 1 1  x x 3 x4  x2 1

x 2  2x  1 x 3  3x 2  2x
272. lim 273. lim
x 1 x3  x x 2 x2  x  6

 x  1 2 x 8x 3  1
274. lim 275. lim 2
x  6x  5x  1
2 1
x 1 x 1 2

x3  x  2  1
lim  
3 
276. lim 3 277. 
x 1 x  x 2  x  1 x 1 1  x
 1 x3 

 1 1   x 2 x4 
278. lim    279. lim  2  
3  x 2  3x  2  
2 2
x 2
 x  x  2  x  3x  2  x 1 x  5x  4


xm 1 x3  x
280. lim (m and n integers) 281. lim
x 1 x n  1 x  x 4  3x 2  1

x 4  5x x 2 1
282. lim 2 283. lim 2
x  x  3x  1 x  2x  1

1  x  3x 3  x3 
284. lim 285. lim  2 x
x  1  x 2  3x 3 x  x  1
 

 x3 x2   3x 2  2x  1  3x 2  x  2 
286. lim  2   287. lim   
x  2x  1
 2x  1  x  2x  1
 4x 2 

10 10 10
 x  1   x  2   ...   x  100  x2 1  x
288. lim 289. lim
4
x  x10  1010 x  
x3  x  x

x2 1  3 x2 1 6
x 7  3  4 2x 3  3
290. lim 291. lim
x  4
x4 1  5 x4 1 x  6
x8  x 7  1  x
3
x 4  3  5 x3  4 1 x2 1
292. lim 293. lim
3
x 
x7  1 x 0 x

1  x 1 x2  1 1
294. lim 295. lim
x 0 x2 x 0
x 2  16  4

x 1  2 x2  x
296. lim 297. lim
x 5 x 5 x 1 x 1
3
xh  x 1 x2 1
298. lim 299. lim
h 0 h x 0 x2
3
1 x  3 1 x xb  a b
300. lim 301. lim  a b
x 0 x xa x2 a2

Get 10% Instant Discount On Unacademy Plus [Use Referral Code: MCSIR] 60
Maths IIT-JEE ‘Best Approach’ (MC SIR) Limit

n 3
lim m
x 1 1  x 2  4 1  2x
302. (m and n integers) 303. lim
x 1 x 1 x 0 x  x2
3
7  x3  3  x2
304. lim
x 1 x 1
305. How do the roots of the equation ax2 + bx + c = 0 change when b and c retain constant values
 b  0 and a tends to zero?
In Problems 306 to 378 find the limits.

306. lim
x 
 xa  x  307. lim
x 
 x2 1  x2 1 
1
308. lim
x  
 x2 1  x  309. lim
x  
x2 1  x 
310. lim
x  
  x  a  x  b   x  311. lim 
x  
x 2  2x  1  x 2  7x  3 
312. lim
x   3
 x  1
2
 3  x  1
2
 313. lim x 3/ 2
x 
 x3  1  x3 1 
sin 3x tan kx
314. lim 315. lim
x 0 x x 0 x
k sin x
316. lim
x 0 sin  x

In problems where it is given x    , consider separately the cases x    and x    .

tan 2x sin   n 
317. lim 318. lim m (m and n positive integers)
x  sin 5x  0
 sin  
2 arcsin x 2x  arcsin x
319. lim 320. lim
x 0 3x x  0 2x  arctan x

1  cos x 1  cos3 x
321. lim 322. lim
x 0 x2 x  0 x sin 2x

tan 
lim 1  sin x  cos x
323.  0 3 2 324. lim
1  cos   x  0 1  sin x  cos x

lim
tan   sin 
lim
1  cos  
325. 326.
 0 3  0 tan 3   sin 3 
1  sin x
 1 1  lim 2

327. lim    328. x  
x  sin x
 tan x 
2
 2  x
 
cos x
lim sin 3x
329.  2 330. lim
x 3
2 1  sin x  x  sin 2x

Get 10% Instant Discount On Unacademy Plus [Use Referral Code: MCSIR] 61
Maths IIT-JEE ‘Best Approach’ (MC SIR) Limit

sin 
  lim
331. lim   x  tan x 332.  a2
x  2  1 2
2 
z  ya y 
333. lim 1  z  tan 334. lim  sin  tan 
z 1 2 y a
 2 2a 
 
sin  x  
cos x  sin x  6
lim lim
335. x
 cos 2x 336. x 3
4 6  cos x
2
x
1  sin
lim 2   
337. x  x x x 338. lim  2x tan x  
cos  cos  sin  x  cos x 
2
2 4 4

cos  a  x   cos  a  x  cos  x  cos  x


339. lim 340. lim
x 0 x x 0 x2
sin  a  x   sin  a  x  sin 2   sin 2 
341. lim 342. lim
x 0 tan  a  x   tan  a  x    2  2

sin  a  2h   2sin  a  h   sin a tan  a  2h   2 tan  a  h   tan a


343. lim 344. lim
h 0 h2 h 0 h2
2  1  cos x 1  sin x  1  sin x
345. lim 346. lim
x 0 sin 2 x x 0 tan x

1  x sin x  cos 2x
lim 1  cos x cos 2x
347. x 0 x 348. lim
tan 2 x 0 x2
2
3
1  arctan 3x  3 1  arcsin 3x   arccos x
349. lim 350. lim
x 0 1  arcsin 2x  1  arctan 2x x  1 x 1
x t
 x   1
351. lim   352. lim  1  
x  1  x
  t 
 t
x 1 mx
 1 x  k
353. lim  1   354. lim  1  
x 
 x x 
 x

2x 1 x 1
 x 1   3x  4  3
355. lim   356. lim  
x  x  2 x  3x  2
   
x2 x
 x2 1   x 1 
357. lim  2  358. lim  
x  x  1 x    2x  1
   

Get 10% Instant Discount On Unacademy Plus [Use Referral Code: MCSIR] 62
Maths IIT-JEE ‘Best Approach’ (MC SIR) Limit
x x
 2x  1   1 
359. lim   360. lim  1  2 
x  
 x 1  x 
 x 
x2
 1  x 2  2x  1 
361. lim 1   362. lim  2 
x  x  4x  2
x  
 x  
cos ec x 1/ 2x
363. lim 1  sin x 
x 0
364.
x 0

lim 1  tan 2 x 
n 1  kx  n  a  x   n a
365. lim 366. lim
x 0 x x 0 x
n x  1
367. x 

lim x  n  x  a   n x   368. lim
x e x  e
a h 1 e 2x  1
369. lim 370. lim
h 0 h x 0 3x
2
ex  e e x  cos x
371. lim 372. lim
x 1 x  1 x 0 x2
e x  e x esin 2x  esin x
373. lim 374. lim
x  0 sin x x 0 x
eax  e bx
375. lim 376. lim x  e1/ x  1
x 
x 0 x

377. lim  cosh x  sinh x  378. lim tanh x


x   x  

Various limits
In Problems 379 to 401 find the limits.

379. lim
 ax  1 . Consider separately the cases when n is (1) positive integer, (2) negative integer, (3)
x  xn  A
zero.
ax
380. lim x  x 2  x 4  1  x 2  381. lim  a  0
x     x   a x  1

a x  a x sin x
382. lim 383 lim
x   a x  a x x  x
arctan x x  sin x
384. lim 385. lim
x  x x  x  cos x

arcsin x
lim
386. x 1 x
tan
2
sin  a  3h   3sin  a  2h   3sin  a  h   sin a
387. lim
h 0 h3

Get 10% Instant Discount On Unacademy Plus [Use Referral Code: MCSIR] 63
Maths IIT-JEE ‘Best Approach’ (MC SIR) Limit

388.
lim tan 2 x
x
  2sin 2 x  3sin x  4  sin 2 x  6sin x  2 
2

1  cos 1  cos x   x x x 
389. lim 390. lim  cos  cos ...cos n 
x 0 x4 n 
 2 4 2 

 1
391. lim x 2 1  cos 
x 
 x
392. x 

lim cos x  1  cos x 
 x 1    x 1 x 
393. lim x  arctan   394. lim x  arctan  arctan 
x 
 x2 4 x 
 x2 x2
n
arcsin x  arctan x  1 
385. lim 396. lim  1  n   n  0 
x 0 x3 x  
 x 
1/sin x n cos x
397. lim  cos.x  398. lim
x 0 x 0 x2
sin x
 sin x  x sin x lim  cos x  sin x 
1/ x
399. lim   400. x 0
x 0
 x 
1/ x
401. lim  cos x  a sin bx 
x 0

Get 10% Instant Discount On Unacademy Plus [Use Referral Code: MCSIR] 64
Maths IIT-JEE ‘Best Approach’ (MC SIR) Limit
CONCEPT BUILDING - 1
1. Examine the graph of y = f(x) as shown and evaluate the following limits :
y
3
2

x
1 2 3 4

(i) lim
x 1
f (x) (ii) lim
x 2
f (x) (iii) lim
x 3
f (x) (iv) xlim
1.99
f (x)

(v) xlim
3
f (x) 

2. Evaluate the following limits,


Where [.] represents greatest integer function and {.} represents fractional part function
  x
(i) lim[cos x] (ii) lim   (iii) lim sgn[tan x] (iv) lim cos 1 (ln x)
x 0 x 3 3 x 0 x 1
 

3. lim cos 1[sec x] is equal to (where [.] denotes greatest integer function)
x 0


(A) (B) 1 (C) 0 (D) Does not exist
2

 x 2  2, x  2 2x, x  6
4. If f (x)  8  x, x  2 and g(x)   , evaluate lim g(f (x)) .
3  x, x  6 x 2

5. Which of the following are indeterminate forms. Also state the type.
[x]
(i) xlim 
, where [.] denotes the greatest integer function
0 x

(ii) xlim x2 1  x


lim(tan x) tan 2x
(iii) x

2

1
(iv) lim({x}) nx , where {.} denotes the fractional part function
x 1

6. Consider the following statements :


[x]
S1 : xlim 
is an indeterminate form (where [.] denotes greatest integer function).
0 x
sin(3x )
S2 : xlim 1
 3x

Get 10% Instant Discount On Unacademy Plus [Use Referral Code: MCSIR] 65
Maths IIT-JEE ‘Best Approach’ (MC SIR) Limit

x  sin x
S3 = lim does not exist.
x  x  cos2 x
(n  2)! (n  1)!
S4 = lim (n  N)  0
x  (n  3)!
State, in order, whether S1, S2, S3, S4 are true or false.
(A) FTFT (B) FTTT (C) FTFF (D)TTFT

7. lim(1  x  [x  2]  [2  x]) is equal to (where [.] denotes greatest integer function)


x 2

(A) 0 (B) 2 (C) –2 (D) does not exist

sin 1 (sin x)
8. lim is equal to
x  0 cos 1 (cos x)

(A) 0 (B) 1 (C) –1 (D) does not exist

9. Evaluate the following limits, if exists


sin 3x sin 2 5x
(i) lim (ii) lim
x 0 tan 1 2x x 0 x2
ln(1  2x) e bx  e ax
(iii) lim (iv) lim , where 0 < a < b
x 0 2x  1 x 0 x
1  cos 5x x(e 2  x  e 2 )
(v) lim (vi) lim
x 0 1  cos 4x x 0 1  cos x

3 sin x  cos x
lim ln(2  x)  ln 0.5
(vii) x
  (viii) lim
6 x x 0 x
6

1  cos 2x
(a  x)2 sin(a  x)  a 2 sin a
(ix) x 0
lim (x) lim 2
x x 0 x

 tan 2x  3x
 x n  3n
(xi) lim 
x  0 3x  sin 2 x
 (xii) Find n  N, if lim  405
  x 3 x 3

cos1 (1  x)
10. lim is equal to
x 0 x
1
(A) (B) 2 (C) 1 (D) 0
2

Get 10% Instant Discount On Unacademy Plus [Use Referral Code: MCSIR] 66
Maths IIT-JEE ‘Best Approach’ (MC SIR) Limit
CONCEPT BUILDING - 2
 100 k 
  (x  1)   100
1. lim  k 1  is equal to
x 2 (x  2)
(A) 0 (B) 5050 (C) 4550 (D) –5050

(4 x  1)3
2. lim
x 0  x 2  is equal to
sin x ln  1  
 3 
(A) 9(ln 4) (B) 3(ln 4)3 (C) 12(ln 4)3 (D) 27(ln 4)2

 
x  2 
lim
   cos x  is equal to (where [.] represents greatest integer function)
3. x
2  
 
(A) 1 (B) 0 (C) – 2 (D) does not exist

4. Evaluate the following limits


3x 2  2x  9 x 3  x  cos 2 x
(i) lim (ii) lim
x  x2 1 x  2x 3  sin x

 1 2 x  n 3  2n 2  1  3 n 4  1
(iii) lim   2  .....  2 
x  x 2
(iv) lim ,nN
 x x  n  4
n 6  6n 5  2  5 n 7  3n 3  1
1
(3x 4  2x 2 ) sin  | x |3 5
(v) lim x
x  | x |3  | x |2  | x | 1

7n  5n  4n 1
5. lim n 1 n n  2 , n  N is equal to
n  7 2 3
1 1
(A) (B) (C) 1 (D) zero
9 7

     
6. lim n cos   sin   , n  N is equal to
n 
 7n   3n 
  
(A) (B) (C) (D) does not exist
3 4 6

Get 10% Instant Discount On Unacademy Plus [Use Referral Code: MCSIR] 67
Maths IIT-JEE ‘Best Approach’ (MC SIR) Limit

 2 1 
7. lim  2
  is equal to
x 1 1  x x 1

1 1
(A) (B)  (C) –1 (D) does not exist
2 2

  1 
8. lim  x  x ln 1    is equal to :
x 
  x 

1 3 1
(A) (B) (C) (D) 1
2 2 3

9. Evaluate the following limits using expansions :


(x  2)3/ 2  (a  2)3/ 2 (x  2)1/2  (15x  2)1/5
(i) lim (ii) lim
x 0 x a x 2 (7x  2)1/4  x

x tan 2 x
e  1  sin x  (ln(1  x)  ln 2)(3.4x 1  3x)
(iii) lim 2 (iv) lim
x 1 [(7  x)1/3  (1  3x)1/2 ]sin(x  1)
x 0 x3
2
 x 
 e 2  cos x  x 2  2 
lim   is equal to
10. x 0 x 2 sin 2 x
 
 
1 1 1 1
(A) (B) (C) (D)
4 6 12 8

CONCEPT BUILDING - 3
1. Find the values of a and b so that :
1  ax sin x  b cos x
(i) lim may have a finite limit.
x 0 x4

(ii) lim
x 
 
x 4  ax 3  3x 2  bx  2  x 4  2x 3  cx 2  3x  d  4

axe x  b ln(1  x)  cxe  x


(iii) lim 2
x 0 x 2 sin x

tan(a  2h)  2 tan(a  h)  tan a


2. Find the value of lim
h 0 h2

sin(a  3h)  3sin(a  2h)  3sin(a  h)  sin a


3. lim is equal to
h 0 h3
(A) cos a (B) – cos a (C) sin a (D) sin a cos a

Get 10% Instant Discount On Unacademy Plus [Use Referral Code: MCSIR] 68
Maths IIT-JEE ‘Best Approach’ (MC SIR) Limit

4. Evaluate the following limits :


x x
tan
 x  2  1  3x 
(i) lim  tan  (ii) lim 
x  1  4x

x 1
 4   
1
x
sec   x
(iii) lim(1   ln x) 2 (iv) lim  tan   x 
x 1 x 0
 4 

lim([x]) tan x
5. The value of x
 is equal to (where [.] denotes the greatest integer function)
2

(A) 0 (B) 1 (C) e (D) e–1

n 1/ n
 x x 
6. lim  sin  sin  , (x  0) is
n   n n 
 
x2
(A) e x 2
(B) e 2 (C) 1 (D) e–1
7. Evaluate the following limits :
2 lim (tan x)cos x
(i) lim(| x |) x (ii) x

x 0 2

1 x
(iii) xlim([x])
1  , where [.] denotes greatest integer funciton
lim e tan x
(iv) x

2

[1.3x]  [2.4x]  ....  [n.(n  2)x]


8. Evaluate lim , where [.] denotes greatest integer function.
n  n3

9. If [x] denotes greatest integer less than or equal to x, then


1 2
lim 3
([1 x]  [2 2 x]  .......  [n 2 x]) is equal to
n  n
x x x x
(A) (B) (C) (D)
2 3 6 4

x 2n  1
10. If f (x)  lim , n  N find range of f(x).
n  x 2n  1

Get 10% Instant Discount On Unacademy Plus [Use Referral Code: MCSIR] 69
Maths IIT-JEE ‘Best Approach’ (MC SIR) Limit
EXERCISE–I
13
x2  x x 7 x 2
Lim x  x.1nx  1nx  1
Q.1 Lim
x 1
Q.2 Lim Q.3 x 1
x 1 x 1 5 x 3 x x 1

 100 k 
  x  100 2 x  3x1/ 3  5x1/ 5 1  3 tan x
Q.4 Lim K1  Q.5 Lim
x  1/ 3 Q.6 Lim
x 1 3x  2  (2x  3)  1  2 cos2 x
x 1 x
4

Lim sec 4 x  sec 2x  p q 


Q.7 Q.8 Lim    p, q N
x 0
sec 3x  sec x x 1  1  x 1  x q 
p

tan x  sin x
Q.9 Find the sum of an infinite geometric series whose first term is the limit of the function f(x) =
sin 3 x
1 x
as x  0 and whose common ratio is the limit of the function g(x) = as x  1.
(cos 1 x) 2
t t
Q.10 Lim (x  l n cosh x) where cosh t = e  e .
x  2

cos 1 2 x 1  x 2  [x ]2  15[ x ]  56


  1  sin 2x
Q.11 (a) Lim ; (b) Lim
x  4 ; (c) Lim
1 1   4x x 7 sin( x  7) sin( x  8)
x
2
x
2 where [ ] denotes the greatest
integer function

Lim 1  tan x 8  x2 x2 x2 x2 
Q.12 Q.13 Lim 1  cos  cos  cos cos 
x  4 x 0 x8 2 4 2 4
1  2 sin x 
 cos x
Lim 2  cos   sin  Lim 2 1
Q.14  4 Q.15 
x 2 
( 4   ) 2 x(x  2 )

Q.16 If Lim a sin x 3 sin 2x is finite then find the value of 'a' & the limit.
x 0 tan x
a  2x x
Q.17 (a) Lim tan
1
Lim 
2 , where a R ; (b) Plot the graph of the function f(x) = t 0
tan 1 2 
x 0 x   t 
(ln (1  x )  ln 2)(3.4 x 1  3x )
Q.18 Lim [ln (1 + sin²x). cot(ln2 (1 + x))] Q.19 Lim 1 1
x 1
x 0 [(7  x ) 3  (1  3x ) 2 ]. sin( x  1)
n
  
Q.20 If l = Lim
n
  (r  1) sin r  1  r sin r  then find { l }. (where { } denotes the fractional part function)
r 2

(3x 4  2 x 2 ) sin 1x  | x |3 5 (x 3  27 ) 1n (x  2)
Q.21 Lim Q.22 Lim
x  3 2 x 3 2
| x |  | x |  | x | 1 x 9

x x x
Q.23 Lim 27  9  3  1
x 0 2  1  cos x

Get 10% Instant Discount On Unacademy Plus [Use Referral Code: MCSIR] 70
Maths IIT-JEE ‘Best Approach’ (MC SIR) Limit

x
Q.24 Let f ( x )  , x 0 and g( x )  x  3, x 1
sin x
 2  x, x  0  x 2  2 x  2, 1  x  2
 x  5, x2
find LHL and RHL of g f (x )  at x = 0 and hence find Lim g f ( x )  .
x0
Pn 1 Pn
Q.25 Let Pn  a  1 ,  n = 2, 3,.......and Let P1 = ax – 1 where a  R+ then evaluate Lim .
x 0 x
1  1 1  ax  1 2 3
Q.26 If the Lim 3
   exists and has the value equal to l, then find the value of   .
x 0 x  1  x 1  bx  a l b
Q.27 Let {an}, {bn}, {cn} be sequences such that
(i) an + bn + cn = 2n + 1 ; (ii) anbn +bncn + cnan = 2n – 1 ; (iii) anbncn = – 1 ; (iv) an < bn < cn
Then find the value of Lim na n .
n
Q.28 If n  N and an = 22 + 42 + 62 + ....... + (2n)2 and bn = 12 + 32 + 52 + ..... + (2n – 1)2. Find the value

an  bn
Lim .
n  n
Q.29 At the end points A, B of the fixed segment of length L, lines are drawn meeting in C and making angles
 and 2 respectively with the given segment. Let D be the foot of the altitude CD and let x represents
the length of AD. Find the value of x as  tends to zero i.e. Lim x .
0

Q.30 At the end-points and the midpoint of a circular arc AB tangent lines are drawn, and the points A and B
are joined with a chord. Prove that the ratio of the areas of the two triangles thus formed tends to 4 as the
arc AB decreases indefinitely.

a tan x  a sin x
Q.31 Evaluate Lim , a > 0. [REE 2001, 3 out of 100]
x 0 tan x  sin x

2 1  1  
Q.32 Find the value of Lim
n
  (n  1) cos  n   n  . [ JEE ' 2004, 2 out of 60]
   

EXERCISE–II
1/ x
8 x 2 3 x  1  x 1/ x 
2 xc
Q.1 Lim  2 x  3  Q.2 Lim
x     4 then find c Q.3 Lim 
x 0  e


x  2  xc  
 2x  5 

2 n 2  n 1
 n 2  n 1  π
Q.4 Lim   Q.5 Lim x sin n cos
2
n  n  x  x
 
x2
  a  tan 2x
Q.6 Lim  cos  2   x    aR Q.7 Lim  tan x 
x     1  x    x 1
 4 
  

Get 10% Instant Discount On Unacademy Plus [Use Referral Code: MCSIR] 71
Maths IIT-JEE ‘Best Approach’ (MC SIR) Limit
1 nx
 x1 1 1 1
x 
Lim  x  1  cos x 
x
Lim  a 1  a 2  a 3 ..... a n 
x x

Q.8 x 0 Q.9 x   n  where a1,a2,a3,......an > 0


 x   

sin 1 (1 {x}).cos 1 (1 {x})


Q.10 Let f(x) = then find xLim
0 
f(x) and xLim
0 
f(x), where {x} denotes the fractional
2{x} . (1  {x})
part function.
a e x  b cos x  ce  x
Q.11 Find the values of a, b & c so that Lim
x 0 2
x. sin x
1  a2  x2  a   x  

Q.12 Lim
x a (a  x 2 ) 2
2  ax  2 sin  2  sin  2   where a is an odd integer
 
2 2
Lim tan x  x
Q.13 x 0
x 2 tan 2 x

(1  x )(1  x 2 )(1  x 3 )......(1  x 2n )


Q.14 If L = Lim then show that L can be equal to
x 1 [(1  x )(1  x 2 )(1  x 3 ).........(1  x n )]2

n
nr 1 n
(a)  r (b)  ( 4r  2)
n! r 1
r 1

(c) The sum of the coefficients of two middle terms in the expansion of (1 + x)2n – 1.
(d) The coefficient of xn in the expansion of (1 + x)2n.

Q.15 Lim [1.x ]  [ 2 .x ]  [ 3.x ]  .....  [ n .x ] , Where [.] denotes the greatest integer function.
n 
n2
1  x  ln x
Q.16 Evaluate, Lim
x 1 1  cos x
  ay   by  
 exp x ln(1  )  exp x1n(1  ) 
 x   x 
Q.17 Lim  Limit
y0  x y 
 
 

Q.18 Let x0 = 2 cos and xn = 2  x n 1 , n = 1, 2, 3, .........., find Lim 2 (n 1) · 2  x n .
6 n

Lim  n (1  x)  1 
1 x
 
Q.19 x 0 2
 x x

 
4 
  n3 1  
(1  n 1 ) 2
Q.20 Let L =  1  2  ; M =   n 3  1  and N =  1  2n 1
, then find the value of
n 3  n  n 2   n 1
L–1 + M–1 + N–1.

Q.21 A circular arc of radius 1 subtends an angle of x radians, 0 < x < as shown in
2
the figure. The point C is the intersection of the two tangent lines at A & B. Let
T(x) be the area of triangle ABC & let S(x) be the area of the shaded region.
Compute:
(a) T(x) (b) S(x) & (c) the limit of T (x ) as x  0.
S(x )

Get 10% Instant Discount On Unacademy Plus [Use Referral Code: MCSIR] 72
Maths IIT-JEE ‘Best Approach’ (MC SIR) Limit
n
x
Q.22 Let f (x) = Lim  3n 1 sin 3 n and g (x) = x – 4 f (x). Evaluate Lim 1  g( x ) cot x .
n 3 x 0
n 1
n
 2  
Q.23 If f (n,)=  1  tan r  , then compute Lim f (n , )
r 1  2  n 

cos 2x  (1  3x )1 3 3 4 cos3 x  ln (1  x ) 4

Q.24 L = Lim 2 4
x 0 x
If L = a b where 'a' and 'b' are relatively primes find (a + b).
x2
 cosh ( x )  et  et
Q.25 Lim   where cosh t =
x    cos (  x ) 

2

f (x) x (1  a cos x )  b sin x


Q.26 f (x) is the function such that Lim  1 . If Lim  1 , then find the value of
x 0 x x 0 f ( x) 3
a and b.
Q.27 Through a point A on a circle, a chord AP is drawn & on the tangent at A a point T is taken such that
AT = AP. If TP produced meet the diameter through A at Q, prove that the limiting value of AQ when P
moves upto A is double the diameter of the circle.
Q.28 Using Sandwich theorem, evaluate

 1 1 1 1 
(a) Lim     ...........  
n  2 
 n n2 1 n2  2 n 2  2n 

1 2 n
(b) Lim 2 + 2 + ......... +
n  1 n 2n n  n2

 x2  1   x 2  x  1  ax  b = 0
Q.29 Lim
Find a & b if : (i) x   x  1  ax  b  = 0 (ii) xLim
 
 
 

 
Q.30 If L = Lim 
1

1  then find the value of L  153 .
x 0  ln (1  x )
 ln ( x  1  x 2 )  L

EXERCISE–III
Q.1 Lim x tan 2x  2x tan x is : [ JEE '99, 2 (out of 200) ]
x 0 2
(1  cos 2x)
1 1
(A) 2 (B)  2 (C) (D) 
2 2
x
 x  3
Q.2 For x  R , Lim
x    = [ JEE 2000, Screening]
 x  2
(A) e (B) e 1 (C) e 5 (D) e5

Get 10% Instant Discount On Unacademy Plus [Use Referral Code: MCSIR] 73
Maths IIT-JEE ‘Best Approach’ (MC SIR) Limit

sin(  cos 2 x )
Q.3 Lim equals [ JEE 2001, Screening]
x 0 x2

(A) – (B)  (C) (D) 1
2
(cos x  1)(cos x  e x )
Q.4 The integer n for which Lim is a finite non-zero number is
x 0 xn
(A) 1 (B) 2 (C) 3 (D) 4
[JEE 2002 (screening), 3]
sin( n x )[(a  n )n x  tan x ]
Q.5 If Lim  0 (n > 0) then the value of 'a' is equal to
x 0 x2
1 n2 1
(A) (B) n2 + 1 (C) (D) None
n n
[JEE 2003 (screening)]
Q.6 If lim (x–3 sin 3x + ax–2 + b) exists and is equal to zero then :
x 0
(A) a = –3 & b = 9/2 (B) a = 3 & b = 9/2 (C) a = –3 & b = –9/2 (D) a = 3 & b = –9/2
1 1
 xn  ex  xn  ex
 2   3 
Q.7 The value of lim     (where n  N) is
x  xn
2 2
(A) ln   (B) 0 (C) n ln   (D) not defined
3 3

x
4  x 2 cos
lim 4
Q.8 is
x 2 sin(x  2)

1
(A) equal to  (B) equal to  (C) equal to   (D) non existent

1  cos(ax 2  bx  c)
Q.9 Let  are the roots of the quadratic equation ax2 + bx + c = 0 then lim equals
x  (x  ) 2

1 a2 a2
(A) 0 (B) ( – )2 (C) ( – )2 (D)  ( – )2
2 2 2

Q.10 ABC is an isosceles triangle inscribed in a circle of radius r. If AB = AC & h is the altitude fromA to BC

and P be the perimeter of ABC then lim equals (where  is the area of the triangle)
h 0 P3

1 1 1
(A) (B) (C) (D) none
32r 64r 128r

Get 10% Instant Discount On Unacademy Plus [Use Referral Code: MCSIR] 74
Maths IIT-JEE ‘Best Approach’ (MC SIR) Limit

 
 (1  cos x )  (1  cos x )  (1  cos x )  .........    1
Q.11 Lim  2

equals
x 0 x
1
(A) 0 (B) (C) 1 (D) 2
2

ln(3  x)  ln(3  x)
Q.12 If lim = k, the value of k is
x 0 x
2 1 2
(A) (B)  (C)  (D) 0
3 3 3
x 2n  1
Q.13 The function f (x) = Lim is identical with the function
n  x 2 n  1

(A) g (x) = sgn(x – 1) (B) h (x) = sgn (tan–1x)


(C) u (x) = sgn( | x | – 1) (D) v (x) = sgn (cot–1x)

Q.14 Which one of the following statement is true?


(A) If Lim f ( x ) ·g( x ) and Lim f ( x ) exist, then Lim g ( x ) exists.
xc xc xc

(B) If Lim f ( x ) ·g( x ) exists, then Lim f ( x ) and Lim g ( x ) exist.


xc xc xc

(C) If Lim f (x )  g ( x )  and Lim f ( x ) exist, then Lim g ( x ) exist.


x c xc xc

(D) If Lim f (x )  g ( x )  exists, then Lim f ( x ) and Lim g ( x ) exist.


x c xc xc

xf (3)  3f (x)
Q.15 If f(3) = 6 & f '(3) = 2, then xlim is given by
3 x 3
(A) 6 (B) 4 (C) 0 (D) none of these

Q.16 If Lim [ f ( x )  g ( x )]  2 and Lim [ f ( x )  g ( x )]  1 , then Lim f (x ) g ( x )


x a x a x a

3 3 4
(A) need not exist (B) exist and is (C) exists and is – (D) exists and is
4 4 3

Q.17 Given 0 < a < b, the value of lim (an + bn)1/n is


n 

(A) a (B) b (C) 1/a (D) 1/b

12 n 2 2 (n1)32 (n2).....n 2.1


Q.18 Lim is equal to :
n 13 23 33 ......n 3
1 2 1 1
(A) (B) (C) (D)
3 3 2 6

Get 10% Instant Discount On Unacademy Plus [Use Referral Code: MCSIR] 75
Maths IIT-JEE ‘Best Approach’ (MC SIR) Limit

Limit

cot  1 x  a log a x 
Q.19 The value of (a > 1) is equal to
x 
sec 1 a x
log x a
(A) 1 (B) 0 (C) /2 (D) does not exist

Q.20 Let f : (1, 2)  R satisfie the inequality


cos(2x  4)  33 x 2 | 4x  8 |
< f(x) <  x  (1, 2). Then lim f(x) is equal to
2 x 2 x  2

(A) 16 (B) –16


(C) cannot be determined from the given information
(D) does not exists

sin x cos x n
Q.21 Let a = min [x2 + 2x + 3, x  R] and b = Lim
x 0 e x  e x
. Then the value of  a r b n r is
r 0

2 n 1  1 2n 1  1 2n  1 4n 1  1
(A) (B) (C) (D)
3 ·2 n 3 ·2 n 3 ·2 n 3 ·2 n

    1    
Q.22 Given l1 = Lim cos 1 sec x   ; l2 = Lim sin cosec x    ;
x  4   4  x  4   4 

1     1    
l3 = Lim tan cot  x   ; l4 = Lim cot  tan  x  
x  4   4  x  4   4 
where [x] denotes greatest integer function then which of the following limits exist?
(A) l1 and l2 only (B) l1 and l3 only (C) l1 and l4 only (D) All of them

Q.23 Suppose that a and b (b  a) are real positive numbers then the value of
1/ t
 b t 1  a t 1 
Lim  
 has the value equals to
t 0
 b  a 
1
a ln b  b ln a b l n b  a ln a  b b  ba
(A) (B) (C) b ln b – a ln a (D)  
ba ba  aa 
 

/x 1 
Q.24 If xlim x.n 0 1/ x  = –5, where  are finite real numbers then

1 0 1/ x

(A)  = 2,  = 1,  R (B)  = 2,  = 2, = 5


(C)  R,  = 1,  R (D)   R,  = 1, = 5

Get 10% Instant Discount On Unacademy Plus [Use Referral Code: MCSIR] 76
Maths IIT-JEE ‘Best Approach’ (MC SIR) Limit

 1  1
Q.25 Range of the function f(x) =  2   is, where [*] denotes the greatest integer function
 n(x  e)  1  x2
and e = lim (1 + )1/
0

 e 1 
(A)  0,   {2} (B) (0, 1) (C) (0, 1]  {2} (D) (0, 1)  {2}
 e 

Q.26 lim sin–1[tan x] = l then {l} is equla to


x 0

  
(A) 0 (B) 1 – (C) –1 (D) 2 –
2 2 2
where [] and {} denotes greatest integer and fractional part function.
Q.27 Which one of the following limit does not tends to unity ?
sin(tan t) sin(cos x)
(A) lim (B) lim
t 0 sin t x 0 cos x

1  x  1 x x2
(C) lim (D) lim
x 0 x x 0 x

cos ecx
 3 
Q.28 Limit   has the value equal to :
x0 
1  4  x 
(A) e 1/12 (B) e 1/6 (C) e 1/4 (D) e 1/3

Limit

cot 1 x  1  x 
x   x
Q.29 1  2 x  1   is equal to
sec   
 x  1  

(A) 1 (B) 0 (C) /2 (D) non existent


1 x 1 x 1 1 x 
Q.30 lim  tan  2 tan 2  3 tan  .....  n tan n  is equal to
n   2 2 2 2 2 2 2 
1 tan x tan x  x
(A) – tan x (B) (C) (D) x – cot x
x x x tan x

x 2  1 , x  0, 2
sin x x  n, n  0, 1, 2, 3.... 
Q.31 f(x) =  and g(x) =  4 , x  0 then lim g(f(x))
 2 otherwise  5 , x2
x 0

(A) 1 (B) 4 (C) 5 (D) non existent

Get 10% Instant Discount On Unacademy Plus [Use Referral Code: MCSIR] 77
Maths IIT-JEE ‘Best Approach’ (MC SIR) Limit

n(x 2  e x )
Q.32 Let f(x) = . If lim f(x) =  and lim f(x) = m then
n(x 4  e2x ) x  x 

(A)  = m (B)  = 2m (C) 2 = m (D)  + m = 0

Q.33 Lim cos   n 2  n  when n is an integer :


n  
(A) is equal to 1 (B) is equal to  1 (C) is equal to zero (D) does not exist

[x 2 ] [x 2 ]
Q.34 Let lim lim
= l & x 0 2 = m, where [] denotes greatest integer, then :
x 0 x x
(A) l exists but m does not (B) m exists but l does not
(C) l & m both exist (D) neither l nor m exists

2
 2

Q.35 lim  1  log x cos x 
x 0  cos 
 2 
(A) is equal to 4 (B) is equal to 9 (C) is equal to 289 (D) is non existent

Q.36 The value of Limit


 tan  {x }  1   sin {x }
where { x } denotes the fractional part function:
x0 {x }  {x }  1 
(A) is 1 (B) is tan 1 (C) is sin 1 (D) is non existent
2  2 x  sin 2x
Q.37 Lim is :
x  (2 x  sin 2x )esin x
(A) equal to zero (B) equal to 1 (C) equal to  1 (D) non existent

| ax 2  bx  c |
Q.38 Let  &  be the roots of the equation, ax2 + bx + c = 0 where 1 <  < , then lim =1
x m ax 2  bx  c

when :
(A) a > 0 & m > 1 (B) a < 0 & m < 1
|a|
(C) a < 0 &  < m <  (D) =1&m>
a

2
lim
Q.39 The value of x 0  cos ax cos ec bx is
 8b2   8a 2   a2   b2 
       
 2 
 2   2  
 a   b   2b   2a 2 
(A) e (B) e (C) e (D) e

Q.40 If x is a real number in [0, 1] then the value of Limit Limit


m   n   [1 + cos
2m (n !  x)] is given by

(A) 1 or 2 according as x is rational or irrational


(B) 2 or 1 according as x is rational or irrational
(C) 1 for all x
(D) 2 for all x

Get 10% Instant Discount On Unacademy Plus [Use Referral Code: MCSIR] 78
Maths IIT-JEE ‘Best Approach’ (MC SIR) Limit
n
r
Q.41 Lim
n 
 n2  n  r equals
r 1
(A) 0 (B) 1/3 (C) 1/2 (D) 1
(sin x  x) 2  1  cos x 3
Q.42 The value of the limit, lim is equal to
x 0 x 5 sin x  (1  cos x) 2 (2x 2  sin x 2 )

19 4 19
(A) (B) (C) (D) 0
45 9 25

Q.43 Lim  n 2  n  1   n 2  n  1   where [ ] denotes the greatest integer function is


n     
(A) 0 (B) 1/2 (C) 2/3 (D) 1/4
n
n p n q 
Q.44 Lim   , p, q > 0 equals
n  2 
 
pq
(A) 1 (B) pq (C) pq (D)
2

f (x  c) f (x  2c) f (x  3c)
g(x)
Q.45 Let g(x) = f (c) f (2c) f (3c) , where c is constant then lim is equal to
x 0 x
f '(c) f '(2c) f '(3c)
(A) 0 (B) 1 (C) –1 (D) f(c)

  x  n2  2x  e 2g(x)  ef (x)
Q.46 Let f(x) = lim tan 1  4n 2  1  cos   and g(x) = lim n cos   then lim
n    n  n  2  n  x 0 x6
equals
8 7 5 2
(A) (B) (C) (D)
3 3 3 3
3  4sin A sin C
Q.47 In ABC, angles A, B, C are in A.P. then lim is
AC |A C|
(A) 1 (B) 2 (C) 3 (D) not exist

n(x  1)  x
Q.48 The value of lim equal to
x 0 x((1  x)1/x  e)
(A) ee (B) e (C) 1/e (D) 1

 1
Q.49 lim  x   , where {.} denotes the fractional part function.
x 1  x
(A) is equal to 1 (B) is equal to 0 (C) does not exist (D) none of these

Get 10% Instant Discount On Unacademy Plus [Use Referral Code: MCSIR] 79
Maths IIT-JEE ‘Best Approach’ (MC SIR) Limit

k k
1 k r (1  x 2 )1/3  (1  2x)1/4
Q.50 If  cos r  for all k  1 and A =   r  , then lim =
r 1 2 r 1 x A x  x2
1 A 
(A) (B) 0 (C) (D)
2 2 2

 x 
a tan  
 a  2a 
Q.51 If lim  2    e , then a is equal to
x a  x
  2
(A) – (B)  (C) (D) 
2 2 

1 1 f (x)
Q.52 If f(x) = lim lim (sin (x + h + t) – sin(x + h) – sin(x + t) + sin x), then lim is
h 0 h t 0 t x 0 x
(A) 0 (B) 1 (C) –1 (D) does not exist

n
Q.53 Let Pn =  cos(x . 2–k) and g(x) = nlim P then
 n
lim g(x) is
k 1 x 0
(A) 0 (B) 1 (C) –1 (D) does not exist

[COMPREHENSION TYPE]
Paragraph for Question Nos. 54 to 56

 n
  1
  f  x  n  

Let f(x) = (x + 1) (x + 2) (x + 2 ) (x + 2 ) ..... (x + 2 ) and g(x) =  lim 
2 3 n–1  if f (x)  0
n   f (x) 
 0 if f (x)  0
Q.54 The value of g(0) is equal to
f '(0) f '(0) f (0)
f 2 (0) f (0) f '(0)
(A) e (B) ef '(0) (C) e (D) e

Q.55 If x1, x2, x3,....., xn denote the values of x where f(x) vanishes such that x1 > x2 > x3 > ..... > xn, then
n
r
lim  is equal to
r 1 x r
n 

(A) –8 (B) –4 (C) –2 (D) –1

Q.56 Let f(x) = x sin x and g(x) = f(x) f '(x), then number of distinct real roots of equation g(x) = 0 where
x  (–2, 2) is
(A) 5 (B) 6 (C) 7 (D) 8

Get 10% Instant Discount On Unacademy Plus [Use Referral Code: MCSIR] 80
Maths IIT-JEE ‘Best Approach’ (MC SIR) Limit
Paragraph for Question Nos. 57 to 59
Let a sequence be defined by a1 = 0 and an+1 = an + 4n + 3 for all n  1 (n  N)
Q.57 The value ak in terms of k is (k  N)
(A) (k – 1)(2k + 3) (B) (k – 1)(3k + 1) (C) (k – 1)(k + 3) (D) (k – 1)(2k + 5)

a n  a 4n  a  .....  a
42 n 410 n
Q.58 The value of lim
n  a n  a 2n  a  .....  a
22 n 210 n

(A) 685 (B) 683 (C) 687 (D) 690

a n  a 9n  a  .....  a  4L 
92 n 910 n
Q.59 If the value of lim = L, then  11  is
n  a n  a 3n  a  .....  a 3 
32 n 310 n

[.] denotes greatest integer function


(A) 4 (B) 3 (C) 2 (D) 1

Paragraph for Question Nos. 60 to 61

sin x  ae x  be  x  cn(1  x)
Let lim 1
x 0 x2
Q.60 The ordered triplet (a, b, c) is
 1 1   1 1   1 1  1 1 
(A)   ,  , 1 (B)   , ,1  (C)   , , 2  (D)  ,  , 2 
 2 2   2 2   2 2  2 2 
Q.61 The roots of the quadratic equation ax2 + bx + c = 0
(A) both positive (B) both negative
(C) one positive and one negative (D) complex conjugate

Paragraph for Question Nos. 62 to 64

 
 1   n 
Let f(x) = lim   where x  R –   , n  I
y   sin(x  (1/ y))   2
 y n  sin x

  

n
a = lim lim ,nI
n  x 0 cot 2 x cot 2 x 2 x tan 2 x
(1 2  .....  (2n)cot )
1
4  2
b = lim  tan 1 x  x 1 (where N represent set of natural numbers, I represent set of integers and R
x 1   
represent set of real numbers)

Get 10% Instant Discount On Unacademy Plus [Use Referral Code: MCSIR] 81
Maths IIT-JEE ‘Best Approach’ (MC SIR) Limit
Q.62 f(x) = ax has 5 roots in (0, m), m  N, then m =
(A) 5 (B) 6 (C) 8 (D) 10

Q.63 If f(x) = bx has 6 roots in (0, m), m  N then m =


(A) 5 (B) 6 (C) 8 (D) 10

Q.64 The number of points of intersection of two curves y = |f(x)| and y = b in interval (0, 4) is
(A) 5 (B) 6 (C) 8 (D) 10

Q.65 Statement-1 : If  and  are positive reals and [*] denotes greatest integers function then
x   
lim  
x 0    x 
because
{y}
Statement-2 : lim= 0, {.} denotes fractional part function.
y y

(A) Statement-1 is true, statement-2 is true; statement-2 is correct explanation for statement-1.
(B) Statement-1 is true, statement-2 is true; statement-2 is NOT the correct explanation for statement-1.
(C) Statement-1 is true, statement-2 is false.
(D) Statement-1 is false, statement-2 is true.

Q.66 Lim f(x) does not exist when


x c
(A) f(x) = [[x]]  [2x  1], c = 3 (B) f(x) = [x]  x, c = 1
tan (sgn x)
(C) f(x) = {x}2  {x}2, c = 0 (D) f(x) = ,c =0.
sgn x
where [x] denotes step up function & {x} fractional part function.

x.2 x  x  n2 
Q.67 Let f(x) = & g(x) = 2x sin  x  then :
1  cos x  2 
(A) lim f(x) = ln 2 (B) lim g(x) = ln 4 (C) lim f(x) = ln 4 (D) lim g(x) = ln 2
x 0 x  x 0 x 

x 1
Q.68 Let f(x) = 2 . Then :
2x  7x  5
1 1
(A) lim f(x) =  (B) lim f(x) =  (C) lim f(x) = 0 (D) lim does not exist
x 1 3 x 0 5 x  x 5/2

Q.69 Which of the following limits vanish?


1 1
(A) Limit
x   x sin
4 (B) Limit
x   /2 (1  sin x) . tan x
x
2
2x  3 [x]2  9
(C) Limit . sgn (x) (D) Limit
x  3
x x2  x  5 2
x 9
where [ ] denotes greatest integer function

Get 10% Instant Discount On Unacademy Plus [Use Referral Code: MCSIR] 82
Maths IIT-JEE ‘Best Approach’ (MC SIR) Limit
Q.70 In which of the following cases limit exists at the indicated points.
[x  | x |]
(A) f(x) = at x = 0 where [] denotes the greatest integer function
x
(B) f(x) = (x – 3)1/5 Sgn(x – 3) at x = 3, where Sgn stands for Signum function
1
xe x tan 1 | x |
(C) f(x) = 1 at x = 0 (D) f(x) = at x = 0
x
1 ex
x
 ax  1  2 2
Q.71 Consider the function f(x) =   where a + b  0 then xlim f(x)
 bx  2  

(A) exists for all values of a and b (B) is zero for 0 < a < b
1 1
   
(C) is non existent for a > b > 0 (D) is e a or e b if a = b

2    2 f () 2  2  1
Q.72 Assume that Lim f () exists and   holds for certain interval
  1 3 2 3
containing the point  = – 1 then Lim f ()
  1
(A) is equal to f (–1) (B) is equal to 1 (C) is non existent (D) is equal to – 1

a(2x 3  x 2 )  b(x 3  5x 2  1)  c(3x 3  x 2 )


Q.73 If lim = 1 then which of the following relations between
x  a(5x 4  x)  bx 4  c(4x 4  1)  2x 2  5x

a, b and c must hold good ?


(A) a + 2b + c = 1 (B) 5a – b + 4c = 0
(C) 2a + b – 3c = 0 (D) a – 5b + c + 2 = 0

h2 x3 1
Q.74 If lim  x then f(x) can not be equal to
h 0 f (x  2h)  2f (x  h)  f (x) 2 2x
(A) tan–1(x2) + ax + b (B) tan–1(x) + ax2 + b
(C) tan–1(x2) + ax2 + b (D) tan–1(x3) + ax + b

(sin(sin x)  sin x) 1
Q.75 If lim 3 5
 , then
x 0 ax  bx  c 12
(A) a = 2 (B) a = –2 (C) c = 0 (D) b  R
1 
Q.76 If lim n = 1, where x  (2n + 1) ,  n  I, then possible value(s) of  will be :
n 0
 tan 2 x    2
 2  1
 tan x  1 
1 3 1 5
(A)  (B) (C) (D)
2 4 4 4

Get 10% Instant Discount On Unacademy Plus [Use Referral Code: MCSIR] 83
Maths IIT-JEE ‘Best Approach’ (MC SIR) Limit

Q.77 Column–I Column-II


nx
(A) lim (P) 2
x 1 x 4  1

3e x  x 3  3x  3 2
(B) lim (Q)
x 0 tan 2 x 3

  2 tan 1 x 3
(C) lim (R)
x   1 2
n  1  
 x
2sin x  sin 2x 1
(D) lim (S)
x 0 x(cos x  cos 2x) 4

e x  e x  2x 11
(E) lim (T)
x 0 x  sin x 2

1  1  x 2 cos x.cos 3x
(F) lim
x 0 x2

  cos x  cos a 
xlim   , a  n
a  cot x  cot a 

Q.78 Let f(a) = 

2 , a  (2n  1) , n  I and b = lim
n 
 n 2  4n  1  n 2  2 
2 , a  2n



Column–I Column-II
(A) If f(x) is periodic with fundamental period k where k  R, then k  (P) 0
(B) The integers in the range of |f(x)| is/are (Q) 1
(C) Let k where k  R be the root of the equation |f(x)| = b in (R) 2
interval [–4, 4], then possible values of |k| is/are

 b|x| | x | 2
(D) Let g(x) =  then possible values g(x) (S) 4
sgn | 2b  x | 2  | x |
can attain is/are

EXERCISE–IV
1  cos 2x
1. lim is [2002]
x 0 2x
(A) 1 (B) –1 (C) zero (D) does note exist

Get 10% Instant Discount On Unacademy Plus [Use Referral Code: MCSIR] 84
Maths IIT-JEE ‘Best Approach’ (MC SIR) Limit
x
 x 2  5x  3 
2. lim  2  [2002]
x   x  x  3 

(A) e4 (B) e2 (C) e3 (D) 1

xf (2)  2f (x)
3. Let f(x) = 4 and f (x)  4 . Then lim is given by [2002]
x 2 x2
(A) 2 (B) –2 (C) –4 (D) 3

log x n  [x]
4. lim , n  N , ([x] denotes greatest integer less than or equal to x) [2002]
x 0 [x]
(A) has value –1 (B) has value 0 (C) has value 1 (D) does not exist

f (x)  1
5. If f(1) = 1, f1(1) = 2, then lim is [2002]
x 1 x 1
1
(A) 2 (B) 4 (C) 1 (D)
2

1  2 4  34  ......n 4 1  23  33  ......n 3
6. lim  lim [2003]
n  n5 n  n5
1 1 1
(A) (B) (C) Zero (D)
5 30 4

log(3  x)  log(3  x)
7. If lim  k , the value of k is [2003]
x 0 x
2 1 2
(A)  (B) 0 (C)  (D)
3 3 3

  x 
1  tan  2   [1  sin x]
8. lim is [2003]
x   x  3
2 1  tan   [  2x]
  2  

1 1
(A)  (B) (C) 0 (D)
8 32
2x
 a b 
9. If lim  1   2   e 2 , then the values of a and b, are [2004]
x   x x 
(A) a = 1 and b = 2 (B) a = 1, b  R (C) a  R, b = 2 (D) a  R, b  R

Get 10% Instant Discount On Unacademy Plus [Use Referral Code: MCSIR] 85
Maths IIT-JEE ‘Best Approach’ (MC SIR) Limit

1  cos(ax 2  bx  c)
10. Let  and  be the distinct roots of ax2 + bx + c = 0, then lim is equal to
x  (x  )2

a2 a 2 1
(A) (  )2 (B) 0 (C) (  )2 (D) (  ) 2 [2005]
2 2 2

f (3x) f (2x)
11. Let f : R  R be a positive increasing function with xlim  1. Then lim 
 f (x) x  f (x)

2 3
(A) (B) (C) 3 (D) 1 [2010]
3 2

 1  cos{2(x  2)} 
12. lim   [2011]
x 2  x2 
1
(A) equals 2 (B) equals – 2 (C) equals (D) does not exist
2

13. Let f : R  [0, ) be such that lim f (x) exists and lim
 f (x)  9
 0 . Then lim f (x) equals
x 5 x 5 | x 5| x 5

[JEE Mains 2011]


(A) 0 (B) 1 (C) 2 (D) 3

(1  cos 2x)(3  cos x)


14. lim is equal to [IIT Mains 2013]
x 0 x tan 4x
1 1
(A) 2 (B) – (C) (D) 1
4 2
2
sin( cos x)
15. lim is equal to [IIT Mains 2014]
x 0 x2

(A)  (B) (C) 1 (D) –
2
 f (x) 
16. Let f(x) be a polynomial of degree four having extreme values at x = 1 and x = 2. If lim 1  2   3 ,
x 0  x 
then f(2) is equal to [IIT Mains 2015]
(A) 4 (B) –8 (C) –4 (D) 0

(1  cos 2x)(3  cos x)


17. lim is equal to [IIT Mains 2015]
x 0 x tan 4x
1
(A) (B) 4 (C) 3 (D) 2
2
1
18.
x 0

Let p = lim 1  tan 2 x  2x then log p is equal to [IIT Mains 2016]

1 1
(A) 2 (B) 1 (C) (D)
2 4

Get 10% Instant Discount On Unacademy Plus [Use Referral Code: MCSIR] 86
Maths IIT-JEE ‘Best Approach’ (MC SIR) Limit

19.. For x  R, f(x) = |log2 – sin x| and g(x) = f(f(x)), then : [JEE(Main)-2016]
(A) g is differentiable at x = 0 and g'(0) = –sin(log2)
(B) g is not differentiable at x = 0
(C) g'(0) = cos(log2)
(D) g'(0) = –cos(log2)

cot x  cos x
20. lim equals [JEE Mains 2017]
x
 (  2x)3
2
1 1 1 1
(A) (B) (C) (D)
24 16 8 4

21. For each t  R, let [t] be the greatest integer less than or equal to t. Then, [JEE Mains 2018]
1 2 15  
lim x        ....     .
x 0
x x  x 
(A) does not exist (in R) (B) is equal to 0
(C) is equal to 15 (D) is equal to 120

1  1  y4  2
22. lim [JEE Main 2019 (09-01-2019-Shift-1)]
y 0 y4
1
(A) exists and equals (B) does not exist
4 2
1 1
(C) exsits and equals (D) exists and equals
2 2 2 2  2 1
x [x] | x | sin[x]
23. For each x R, let [x] be the greatest integer less than or equal to x. Then lim is
|x|
x 0

equal to: [JEE Main 2019 (09-01-2019-Shift-2)]


(A) 1 (B) sin 1 (C) – sin 1 (D) 0

24. For each t  R, let [t] be the greatest integer less than or equal to t. Then

1  x  sin 1  x  sin  2 1  x 
lim   [JEE Main 2019 (10-01-2019-Shift-1)]
x 1 1  x 1  x 
(A) does not exist (B) equals 1 (C) equals – 1 (D) equals 0

25. Let [x] denote the greatest integer less than or equal to x. then :
2

lim

tan   sin 2 x   x  sin  x  x    [JEE Main 2019 (11-01-2019-Shift-1)]
x 0 x2
(A) does not exist (B) equals 0 (C) equals  (D) equals 

x cot(4x)
26. lim
x 0 sin 2 x cot 2  2x  is equal to: [JEE Main 2019 (11-01-2019-Shift-2)]

Get 10% Instant Discount On Unacademy Plus [Use Referral Code: MCSIR] 87
Maths IIT-JEE ‘Best Approach’ (MC SIR) Limit
(A) 4 (B) 1 (C) 0 (D) 2

cot 3 x  tan x
lim
27. x  / 4  
cos  x   is : [JEE Main 2019 (12-01-2019-Shift-1)]
 4
(A) 8 2 (B) 8 (C) 4 2 (D) 4

  2 sin 1 x
28. l im is equal to: [JEE Main 2019 (12-01-2019-Shift-2)]
x 1 1 x
2 1 
(A) (B) (C) (D) 
 2 2

sin 2 x
29. lim equals: [JEE Main 2019 (08-04-2019-Shift-1)]
x 0
2  1  cos x
(A) 4 (B) 2 2 (C) 4 2 (D) 2

x4 1 x3  k3
30. If lim  lim 2 , then k is : [JEE Main 2019 (10-04-2019-Shift-1)]
x  1 x k x  k 2
x 1

4 8 3 3
(A) (B) (C) (D)
3 3 2 8

x 2  ax  b
31. If lim  5 , then a + b is equal to : [JEE Main 2019 (10-04-2019-Shift-2)]
x 1 x 1
(A) -7 (B) 4 (C) 1 (D) 5

32. Let f(x) = 5 – |x – 2| and g(x) = |x + 1|, x  R. If f(x) attains maximum value at  and g(x) attains
 x  1  x 2  5x  6 
minimum value at ,then lim is equal to
x  x 2  6x  8
[JEE Main 2019 (12-04-2019-Shift-2)]
(A) 3/2 (B) –3/2 (C) –1/2 (D) 1/2

x  2sinx
33. lim is : [JEE Main 2019 (12-04-2019-Shift-2)]
x 0
x  2sin x  1  sin 2 x  x  1
2

(A) 1 (B) 3 (C) 6 (D) 2

3x  33 x  12
34. lim  x/ 2  x is equal to _______. [JEE Main 2020 (07-01-2020-Shift-1)]
x 2 3 3

1/ x 2
 3x 2  2 
35. lim  2  is equal to : [JEE Main 2020 (08-01-2020-Shift-1)]
x  0 7x  2
 

Get 10% Instant Discount On Unacademy Plus [Use Referral Code: MCSIR] 88
Maths IIT-JEE ‘Best Approach’ (MC SIR) Limit

1 1
(A) (B) e2 (C) e (D)
e2 e

x  x 2  x 3  ...  x n  n
36. If lim  820,  n  N  then the value of n is equal to :
x 1 x 1
[JEE Main 2020 (02-09-2020-Shift-1)]
1/ x
  
37. lim  tan   x   is equal to: [JEE Main 2020 (02-09-2020-Shift-2)]
x 0
 4 
(A) e (B) e2 (C) 2 (D) 1

38. Let [t] denote the greatest integer  t. if for some   R  0,1

lim 1 x | x |
x 0  L , then L is equal to: [JEE Main 2020 (03-09-2020-Shift-1)]
  x  x
1
(A) 0 (B) 2 (C) (D) 1
2

lim
 1  x2 x2 x2 x 2   k
39. If x 0  8 1  cos  cos  cos cos    2 , then the vlaue of k is ______
 x  2 4 2 4  
[JEE Main 2020 (03-09-2020-Shift-1)]
1 1
(a  2x) 3  (3x) 3
40. lim 1 1
(a  0) is equal to : [JEE Main 2020 (03-09-2020-Shift-2)]
x a
3 3
(3a  x)  (4x)
4 4 1 1
 2 3  2 3  2  2  3  2  2  3
(A)   (B)   (C)    (D)   
9  3  3  9   9  3 
1  cos  p  x  
41. If  is positive root of the equation, p(x) = x2  x  2 = 0, then lim is equal to :
x 4
x 

[JEE Main 2020 (05-09-2020-Shift-1)]


1 3 3 1
(A) (B) (C) (D)
2 2 2 2

  1 x 2  x 4 1/ x 
xe 1 
42. lim   [JEE Main 2020 (05-09-2020-Shift-2)]
x 0 2 4
1 x  x 1
(A) is equal to e (B) is equal to 1 (C) is equal to 0 (D) does not exist
n
 1 1 
 1  2  .......  n 
43. lim  1   is equal to : [JEE Main 2021 (25-02-2021-Shift-1)]
n 
 n2 
 

Get 10% Instant Discount On Unacademy Plus [Use Referral Code: MCSIR] 89
Maths IIT-JEE ‘Best Approach’ (MC SIR) Limit

1 1
(A) (B) 0 (C) (D) 1
2 e

ax   e 4x  1
44. If lim exists and is equal to b, then the value of a – 2b is ______.
x 0 ax  e 4x  1
[JEE Main 2021 (25-02-2021-Shift-2)]

    
 3 sin  6  h   cos  6  h  
   
45. The value of lim 2  is [JEE Main 2021 (26-02-2021-Shift-1)]
h 0
 3h 3 cosh  sinh   
 
4 2 3 2
(A) (B) (C) (D)
3 3 4 3

ae x  b cos x  ce  x
46. If lim  2 , then a + b + c is equal to _______.
x 0 x sin x
[JEE Main 2021 (16-03-2021-Shift-1)]

2 2
cos 1 (x   x  )  sin 1 (x   x  )
47. The value of lim , where [x] denotes the greatest integer  x is :
x 0 x  x3
[JEE Main 2021 (17-03-2021-Shift-1)]
 
(A)  (B) 0 (C) (D)
4 2

[r]  [2r]  .....  [nr]


48. The value of lim , where r is non-zero real number and [r] denotes the greatest
n  n2
integer less than or equal to r, is equal to : [JEE Main 2021 (17-03-2021-Shift-2)]
r
(A) (B) r (C) 2r (D) 0
2

tan   cos 2  
49. The value of the limit lim is equal to : [JEE Main 2021 (17-03-2021-Shift-2)]
 0 sin  2 sin 2  
1 1 1
(A)  (B)  (C) 0 (D)
2 4 4
sin 1 x  tan 1 x
50. If lim is equal to L, then the value of (6L + 1) is
x 0 3x 3
[JEE Main 2021 (18-03-2021-Shift-1)]
1 1
(A) (B) (C) 6 (D) 2
6 2
 x 2 
 2 
51. 
If the value of lim 2  cos x cos 2x
x 0
 x 
is equal to ea, then a is equal to
[JEE Main 2021 (20-07-2021-Shift-1)]

Get 10% Instant Discount On Unacademy Plus [Use Referral Code: MCSIR] 90
Maths IIT-JEE ‘Best Approach’ (MC SIR) Limit

xe x –  log (1  x)   x 2 e – x
52. If lim  10,  R, then value of  is _____.
x 0 x sin 2 x
[JEE Main 2021 (20-07-2021-Shift-2)]
x 2f (2)  4f (x)
53. Let f : R  R be a function such that f(2) = 4 and f (2) = 1. Then, the value of lim is
x 2 x2
equal to : [JEE Main 2021 (27-07-2021-Shift-1)]
(A) 12 (B) 8 (C) 4 (D) 16

 x 
54. The value of lim
x 0  8 8  is equal to : [JEE Main 2021 (27-07-2021-Shift-2)]
 1  sin x – 1  sin x 
(A) – 1 (B) 4 (C) 0 (D) – 4

 9 x 
55. lim    is equal to: [JEE Main 2021 (26-08-2021-Shift-2)]
 2
 n 1 n  n  1 x  2  2n  1 x  4 
x 2

5 1 9 7
(A) (B) (C) (D)
24 5 44 36


2 x 2  bx  c 
e  1  2  x 2  bx  c 
56. If  ,  are the distinct roots of x 2  bx  c  0 , then lim 2
is equal to:
x 
 x  
[JEE Main 2021 (27-08-2021-Shift-1)]
(A) b2  4c (B) 2  b 2  4c  (C) 2  b 2  4c  (D) b2  4c

57. If lim
x 
 
x 2  x  1  ax  b , then the ordered pair (a, b) is: [JEE Main 2021 (27-08-2021-Shift-2)]

 1  1  1  1
(A) 1,   (B)  1,  (C)  1,   (D) 1, 
 2  2  2  2

sin 2   cos 4 x 
58. lim is equal to: [JEE Main 2021 (31-08-2021-Shift-1)]
x 0 x4
(A) 4 (B) 22 (C) 42 (D)  2

tan 3 x  tan x cot x


59. Let   lim and   lim  cos x  are the roots of equation ax 2  bx  4  0 , then
x   x  0
4 cos  x  
 4
the ordered pair (a, b) is: [JEE Main 2021 (31-08-2021-Shift-2)]

(A) (1, –3) (B) (–1, 3) (C) (1, 3) (D) (–1, –3)

60. Let f  x   x 6  2x 4  x 3  2x  3, x  R. Then the natural number n for which

x n f 1  f  x 
lim  44 is [JEE Main 2021 (01-09-2021-Shift-2)]
x 1 x 1

Get 10% Instant Discount On Unacademy Plus [Use Referral Code: MCSIR] 91
Maths IIT-JEE ‘Best Approach’ (MC SIR) Limit
EXERCISE–V
2
x
a  a2  x2 
Q.1 Let L = lim 4 , a > 0. If L is finite, then [JEE 2009, 3]
x 0 x4
1 1
(A) a = 2 (B) a = 1 (C) L = (D) L =
64 32
1/x
Q.2 If lim 1  x ln(1  b 2 )  = 2b sin2 , b > 0 and  (,], then the value of  is
x 0

[JEEAdv 2012]
   
(A)  (B)  (C)  (D) 
4 3 6 2

Q.3 Let (a) and (a) be the roots of the equation [JEE 2012]

 3

1  a 1 x 2   1 a 1 x    6

1  a  1  0 where a > –1

Then lim (a) and lim (a) are(


a 0 a 0

5 1 7 9
(A) – and 1 (B) – and –1 (C) – and 2 (D) – and 3
2 2 2 2

 x2  x  1 
Q.4 If lim   ax  b  = 4, then [JEE 2012]
x 
 x 1 
(A) a = 1, b = 4 (B) a = 1, b = –4 (C) a = 2, b = –3 (D) a = 2, b = 3
1 x
 ax  sin(x  1)  a 1 x 1
Q.5 The largest value of the non-negative integer a for which lim    is
x 1
 x  sin(x  1)  1  4
[IIT Adv. 2014]

 ecos( n )  e  e m
Q.6 Let m and n be two positive integers greater than 1. If lim       then the value of
0  m  2 n
  
is [IIT Adv. 2015]
x 2 sin( x)
Q.7 Let  R be such that lim = 1. Then 6() equals [IIT Adv. 2016]
x 0 x  sin x

1  x(1 | 1  x |)  1 
Q.8 Let f(x) = cos   for x  1. Then [JEE Adv 2017]
|1  x | 1 x 
(A) lim f(x) does not exist (B) lim f(x) = 0
x 1 x 1

(C) lim f(x) = 0 (D) lim f(x) does not exist


x 1 x 1

Q.9 Let f :    be a function. We say that f has


f (h)  f (0)
PROPERTY 1 : if hlim
0 h exists and is finite, and

Get 10% Instant Discount On Unacademy Plus [Use Referral Code: MCSIR] 92
Maths IIT-JEE ‘Best Approach’ (MC SIR) Limit

f (h)  f (0)
PROPERTY 2 : if hlim exists and is finite.
0 h2
Then which of the following option(s) is /are correct?
[JEE (advanced) 2019]
(A) f(x) = x |x| has PROPERTY 2 (B) f (x) = x2/3 PROPERTY 1
(C) f (x) = sin x has PROPERTY 2 (D) f (x) = |x| has PROPERTY 1

Q.10 For non - negative inters n, let


n
 k 1  k2 
 sin  n  2   sin  n  2  
k 0
f (n)  n
 k 1 
 sin 2  n  2  
k 0

Assuming cos-1 x takes values in [0,  ], which of the following opiton(s) is / are correct ?
[JEE (advanced) 2019]
3
(A) sin (7 cos-1 f(5)) = 0 (B) f  4  
2
1
(C) lim f (n)  (D) If   tan(cos1 f (6)), then  2  2  1  0
n  2

Q.11 Let e denote the base of the natural logarithm. The value of the real number a for which the right hand
1
1
x
limit lim (1  x ) a  e is equal to a non-zero real number, is____ [JEE Advanced 2020]
x 0  x

4 2(sin 3x  sin x )
Q.12 The value of the limit lim
x  3x 5x   3x  is
2  2sin 2 x sin  cos    2  2 cos 2 x  cos 
 2 2   2 
[JEE Advanced 2020]

Get 10% Instant Discount On Unacademy Plus [Use Referral Code: MCSIR] 93
Maths IIT-JEE ‘Best Approach’ (MC SIR) Limit

ANSWER KEY
GN BERMAN
1
245. 1 246. 247. 3 248.  249. 0 250. 0
2
15
251. 252. 1 253. 0 254. 4 255. 1 256. 0
17
4 1 1
257. 0 258. 0 259. 1 260. 261. 262. 
3 2 2
1 1 1
263.  1 264. 1. Note that  n  1 n  n  1  n

1 3
265. 266. 1 267. 0 268. 9 269. 270. 
2 4
2 1
271. 0 272. 0 273.  274. 275. 6 276. 
5 2
m
277.  1 278.  279. 0 280. 281. 0 282. 
n
1 1 1
283. 284.  1 285. 0 286. 287.  288. 100
2 4 2
289.  1 290. 1 291.  292. 0 293. 0 294. 
1 1
295. 4 296. 297. 3 298. if x  0;  if x  0
4 2 x
1 2 1 m
299. 300. 301. 302.
3 3 4a a  b n

1 1
303. . Add unity to, and subtract from the numerator.. 304. 
2 4
c
305. One root tends to  306. 0 307. 0
b
1
308. 0 if x   ;  if x    309. if x  ;   if x   
2
ab 5
310. if x   ,  if x    311.  312. 0 313. 1
2 2
 2
314. 3 315. k 316.  317.
5
318. 0 if n > m, 1 if n = m,  if n < m 319. 2/3 320. 1/3
321. 1/2 322. 3/4 323.  324.  1 325. 1/2 326. 
327. 0 328. 1/2 329.  330.  3/2 331. 1 332.  / 2

Get 10% Instant Discount On Unacademy Plus [Use Referral Code: MCSIR] 94
Maths IIT-JEE ‘Best Approach’ (MC SIR) Limit

a 2 2
333. 2 /  334.  335. 336. 2 337. 338. -2
 2 2
2   2 sin 2
339..  2 sin a 340. 341. cos 3  342. 2
2
2 sin a 2
343.  sin  344. 345. 346. 1 347. 6
cos3 a 8
3 1 1 1
348. 349. -1 350. Put arccos x = y 351. 352.
2 2 e e
353. 1 354. emk 355. e6 356. e 2/3 357. e2
358. 0 if x  ,  if 359.  if x   , 0 if x   360. 1
361.  if x    , 0 if x    362. e2 363. e 364. e
365. k 366. 1/a 367. a 368. 1/e 369. ln a 370. 2/3
371. e 372. 3/2; add unity to, and subtract from, the numerator.
373. 2 374. 1 375. a  b 376. 1
377. 0 if x   ,  if x    378. 1 if x   ,  1 if x   
1
379. 1 a n (2) 0 if A  0, a n if A = 0 amd a  0 , and  , if A = a = 0; (3)
1 A
380. 0 if x   ;   if x   
381. For a > 1 the limit equals 1 if x   , and 0 if x    . For a < 1 the limit equals 0
if x   , and 1 if x    . For a = 1 the limit is equal to 1/2.
382. For a > 1 the limit equals 1 if x   , and 0 if -1 if x    . For a < 1 vice versa.
383. 0 384. 0 385. 1 386. 0 387.  cos a 388. 1/12
sin x x
389. 1/8 390. Multiply and divide by sin n 391. 1/2 392. 0
x 2
ba 1
393.  1/2 Use the formula arctan b - arctan a = arctan 394.
1  ab 2
1 x
395. Replace arcsin x by arctan and take advantage of the hint to Problem
2 1 x2
393.
396.  if n < 1; e if n = 1; 1 if n > 1
397. 1. Take the expression 1  (1  cos x) instead of cos x. 398.  1/2
399. 1/e 400. e 401. eab

Get 10% Instant Discount On Unacademy Plus [Use Referral Code: MCSIR] 95
Maths IIT-JEE ‘Best Approach’ (MC SIR) Limit

CONCEPT BUILDING - 1
1. (i) 2 (ii) 2 (iii) Limit does not exist (iv) 2 (v) 2

2. (i) 0 (ii) Limit does not exist (iii) Limit does not exist (iv)
2
3. C 4. 12
5. (i) No (ii) No (iii) Yes, 0 form (iv) No
6. A 7. C 8. D
3 2 25 1
9. (i) (ii) 25 (iii) (iv) (b – a) (v) (vi) 2e2 (vii) 2 (viii)
2 ln 2 16 2
1
(ix) 2a sina + a2 cosa (x) limit does not exist (xi)  (xii) 5
3
10. B
CONCEPT BUILDING - 2
1. B 2. B 3. A
1 1
4. (i) 3 (ii) (iii) (iv) 1 (v) –2
2 2
5. B 6. A 7. A 8. A
3 2 1 9 4
9. (i) (a  2)1/ 2 (ii)  (iii) (iv)  ln
2 25 3 4 e
10. B

CONCEPT BUILDING - 3
1
1. (i) a  , b  1 (ii) a = 2, b  R, c = 5, d  R (iii) a = 3, b = 12, c = 9
2
2. 2(sec2a)tana
3. B
4. (i) e–1 (ii) 0 (iii) e–2 (iv) e2
5. B 6. C
7. (i) 1 (ii) 1 (iii) 0 (iv) limit does not exist
x
8. a = b and bc = –3 9. B 10.
3

EXERCISE–I
45 2 1
Q 1. 3 Q 2. Q 3. 2 Q.4 5050 Q 5. Q.6 –
91 3 3

3 pq 1 1 2
Q.7 Q.8 Q.9 a = ;r= ;S= Q 10. l n 2
2 2 2 4 3
1 1
Q.11 (a) does not exist; (b) does not exist; (c) 0 Q 12. 2 Q.13 Q.14
32 16 2

Get 10% Instant Discount On Unacademy Plus [Use Referral Code: MCSIR] 96
Maths IIT-JEE ‘Best Approach’ (MC SIR) Limit

Q.15 21n 2 Q.16 a = 2 ; limit = 1 Q.17 (a) /2 if a > 0 ; 0 if a = 0 and –/2 if a < 0; (b) f(x) = | x |

9 4
Q.18 1 Q 19.  1n Q.20  – 3 Q.21 2 Q.22 9 Q.23 8 2 (1n 3) 2
4 e

3 2L
Q.24 – 3, –3, – 3 Q.25 (ln a)n Q.26 72 Q.27 – 1/2 Q.28 Q.29 Q.30 4
2 3
2
Q.31 lna Q.32 1

EXERCISE–II
1 2 2 a2
Q.1 e-8 Q.2 c = ln2 Q.3 e  2 Q.4 e–1 Q.5  Q.6 e  2 
4
 
Q.7 e-1 Q.8 e–1/2 Q.9 (a1.a2.a3....an) Q.10 ,
2 2 2

2a 2  4 2 x 1
Q.11 a = c = 1, b = 2 Q.12 Q.13 Q.15 Q.16 –
16a 4
3 2 2

Q.17 a - b Q.18 Q.19 1/2 Q.20 8
3
1 x x sin x 1 1 3
Q.21 T(x) = tan2 . sin x or tan  , S(x) = x  sin x, limit =
2 2 2 2 2 2 2
2
Q.22 g (x) = sin x and l = e Q.23  Q.24 19 Q.25 e 
tan 
1
Q.26 a = – 5/2, b = – 3/2 Q.28 (a) 2; (b) 1/2 Q.29 (i) a =1, b = 1 (ii) a = 1 , b = Q.30 307
2

EXERCISE–III
Q.1 C Q.2 C Q.3 B Q.4 C Q.5 C Q.6 A Q.7 B
Q.8 C Q.9 C Q.10 C Q.11 B Q.12 A Q.13 C Q.14 C
Q.15 C Q.16 B Q.17 B Q.18 A Q.19 A Q.20 B Q.21 D
Q.22 A Q.23 D Q.24 D Q.25 D Q.26 D Q.27 B,D Q.28 A
Q.29 A Q.30 C Q.31 A Q.32 A Q.33 C Q.34 C Q.35 C
Q.36 D Q.37 D Q.38 C Q.39 C Q.40 B Q.41 C Q.42 A
Q.43 B Q.44 B Q.45 A Q.46 A Q.47 A Q.48 C Q.49 A
Q.50 A Q.51 B Q.52 B Q.53 B Q.54 C Q.55 B Q.56 C
Q.57 A Q.58 B Q.59 D Q.60 D Q.61 C Q.62 B Q.63 B
Q.64 C Q.65 A Q.66 B,C Q.67 C,D Q.68 A,B,C,D Q.69 A,B,D
Q.70 A,B,C Q.71 B,C,D Q.72 A,D Q.73 B,C,D Q.74 A,B,C,D Q.75 A,C,D
Q.76 A,B,C
Q.77 (A)  S ; (B)  R ; (C)  P ; (D)  Q ; (E)  P ; (F)  T
Q.78 (A)  R,S ; (B)  Q,R ; (C)  P,Q,R,S ; (D)  P,Q,R,S

Get 10% Instant Discount On Unacademy Plus [Use Referral Code: MCSIR] 97
Maths IIT-JEE ‘Best Approach’ (MC SIR) Limit
EXERCISE–IV
1. D 2. A 3. C 4. D 5. A 6. A 7. A
8. D 9. B 10. A 11. D 12. D 13. D 14. A
15. A 16. D 17. D 18. C 19. C 20. B 21. D
22. A 23. C 24. D 25. A 26. B 27. B 28. A
29. C 30. B 31. A 32. D 33. D 34. 36 35. A
36. 40 37. B 38. B 39. 8 40. C 41. B 42. B
43. D 44. 5 45. A 46. 4 47. D 48. A 49. A
50. D 51. 3 52. 3 53. A 54. D 55. C 56. C
57. A 58. C 59. C 60. 7

EXERCISE–V
Q.1 A,C Q.2 D Q.3 B Q.4 B Q.5 0 Q.6 2 Q.7 7
Q.8 B, D Q.9 B, D Q.10 A,B,D Q.11 1.00 Q.12 8

Get 10% Instant Discount On Unacademy Plus [Use Referral Code: MCSIR] 98
Maths IIT-JEE ‘Best Approach’ (MC SIR) Limit
REVISION PLANNER FOR MAINS
CONCEPT BUILDING 01 :
4. 8.
CONCEPT BUILDING 02 :
4. (iv) (v)
CONCEPT BUILDING 03 :
2. 8.
EXERCISE–I
8. 13. 28.
EXERCISE–II
9. 11. 29.
EXERCISE–III
4. 9. 11. 31. 38.
EXERCISE–IV
10. 24. 41.
EXERCISE–V
2.
REVISION PLANNER FOR ADVANCED
CONCEPT BUILDING 02 :
10.
CONCEPT BUILDING 03 :
1. (i) 3.
EXERCISE–I
17. (b) 20. 25. 27. 29.
EXERCISE–II
18. 20. 23. 30.
EXERCISE–III
10. 19. 30. 33. 40.
EXERCISE–IV
13. 18. 21.
EXERCISE–V
3. 5.

Get 10% Instant Discount On Unacademy Plus [Use Referral Code: MCSIR] 99
CONCEPT BUILDING – 1

1. Examine the graph of y = f(x) as shown and evaluate the following limits :
y
3
2

x
1 2 3 4
(i) lim f (x)
x →1

Sol. lim f (x) = 2 { LHL = RHL = 2}


x →1

(ii) lim f (x)


x →2

Sol. lim f (x) = 2 { LHL = RHL = 2}


x →2

(iii) lim f (x)


x →3

Sol. lim f (x) = Does not exist


x →3

{ LHL =1 & RHL = 2}

(iv) lim f (x)


x →1.99

Sol. lim f (x) = 2


x →1.99

(v) lim f (x)


x →3+

Sol. lim f (x) = 2


x →3+

2. Evaluate the following limits,


Where [.] represents greatest integer function and {.} represents fractional
part function
(i) lim[cos x]
x →0

Sol. RHL = lim [cos(0 + h)] = 0


x →0 + h
1

LHL = lim [cos(0 – h)] = 0


x →0 – h
1

 lim [cos x] = 0 Ans.


x →0

100
x 
(ii) lim  
 
x →3 3

x 
Sol. lim  
 
x →3 3

 h h
RHL = lim 1 +  = lim = 0
x →3+ h
 3  h →0 3
 h h
LHL = lim 1 –  =lim 1 − = 1
x →3– h
 3 h → 0 3
x 
 LHL  RHL  lim   does not exist Ans.
 
x →3 3

(iii) lim sgn[tan x]


x →0

Sol.  RHL = lim sgn[tan (0 + h)] = sgn (0) = 0


x →0 + h

LHL = lim sgn[tan (0 – h)] = sgn (–1) = –1


x →0–h

 LHL  RHL
 lim sgn[tan x] does not exist.
x →0

(iv) limcos−1 (ln x)


x →1


Sol. RHL = lim cos −1 (ln(1 + h)) = cos −1 (0) =
x →1+ h 2

LHL = lim cos −1 (ln(1 – h)) = cos −1 (0) =
x →1–h 2

 lim cos −1 (ln x) = Ans.
x →1 2

3. limcos−1[sec x] is equal to (where [.] denotes greatest integer function)


x →0


(A) (B) 1 (C) 0 (D) Does not exist
2
Sol. RHL = lim cos −1[sec (0 + h)] = cos −1 (1) = 0
x →0 + h
1

LHL = lim cos [sec (0 – h)] = cos −1 (1) = 0


−1
x → 0– h
1
−1
 lim cos [sec x] = 0 Ans.
x →0

 x 2 + 2, x  2 2x, x  6
4. If f (x) =  and g(x) =  , evaluate lim g(f (x)) .
8 − x, x  2 3 − x, x  6 x →2

Sol. Let P(x) = g(f(x))


RHL = lim P(x) =lim g(f (2 + h)) =lim g((2 + h) 2 + 2)
x →2+ h h →0 h →0

101
=lim + g(6 + 2h) =lim + 2(6 + 2h) = 12
h →0 h →0
6

LHL = lim P(x) =lim + g(f (2 – h)) =lim + g(8 – (2 – h))


x → 2–h h →0 h →0

=lim + g(6 + h) =lim + 2(6 + 2h) = 12


h →0 h →0
6

 lim g(f (x)) = 12 Ans.


x →2

5. Which of the following are indeterminate forms. Also state the type.
[x]
(i) lim+ , where [.] denotes the greatest integer function
x →0 x
(ii) lim x2 +1 − x
x →−

(iii) lim(tan x) tan 2x



x→
2
1
(iv) lim({x})
+
nx
, where {.} denotes the fractional part function
x →1

[x] Exact 0
Sol. (i) lim+ = lim =0
x →0 x h →0 tending to 0
(ii) lim x 2 + 1 − x =  +  = 
x →−

(iii) lim(tan x) tan 2x = o indeterminate form



x→
2
1
(iv) lim({x})
+
nx
= 0 = 0
x →1

6. Consider the following statements :


[x]
S1 : lim +
is an indeterminate form (where [.] denotes greatest integer
x →0 x
function).
sin(3x )
S2 : lim =1
x →− 3x
x + sin x
S3 = lim does not exist.
x → x − cos 2 x
(n + 2)!+ (n + 1)!
S4 = lim (n  N) = 0
x → (n + 3)!
State, in order, whether S1, S2, S3, S4 are true or false.
(A) FTFT (B) FTTT (C) FTFF (D)TTFT
[x]
Sol. S1 : lim+ = 0 Not an indeterminate form
x →0 x

sin(3x )  0 
S2 : lim   =1
x→–  3x  0 

102
sin x
1+
x + sin x x =1
S3 : lim = lim
x → x − cos 2 x x → cos 2 x
1−
x
(n + 2)!+ (n + 1)!
S4 = lim
n → (n + 3)!
(n + 2) + (n + 1)!+ (n + 1)! (n + 3)
= lim = lim =0
n → (n + 3)(n + 2)(n + 1)! n → (n + 3) (n + 2)

7. lim(1 − x + [x − 2] + [2 − x]) is equal to (where [.] denotes greatest integer


x →2

function)
(A) 0 (B) 2 (C) –2 (D) does not
exist
Sol. lim(1 − x + [x] − 2 + 2 + [− x])
x →2

0 ; x  I
= lim(1 − x + [x] + [− x]) ; = [x] + [–x] = 
x →2
 –1 ; x  I
lim(1 − x –1) = – 2 Ans.
x →2

sin −1 (sin x)
8. lim is equal to
x →0 cos −1 (cos x)

(A) 0 (B) 1 (C) –1 (D) does not exist


x
Sol. LHL = lim− = −1
x →0 −x
x
RHL = lim+ =1
x →0 x

sin −1 (sin x)
 LHL  RHL, lim+ does not exist. Ans.
x →0 cos −1 (cos x)

9. Evaluate the following limits, if exists


sin 3x
(i) lim
x →0 tan −1 2x

sin 3x
sin 3x  0   3x
3x 0 3
Sol. lim −  
x → 0 tan (2x)  0 
= lim −   = Ans.
0 2
1 1
x →0 tan (2x)
 2x
(2x)
sin 2 5x  0 
(ii) lim  
x →0 x2 0
2
 sin 5x 
Sol. lim 
x →0  5x
  25 = 25 Ans.

103
ln(1 + 2x)  0 
(iii) lim  
x →0 2x − 1  0 
ln(1 + 2x)
 2x
2x 2
Sol. lim = Ans.
x →0  2 − 1 
x
ln 2
  x
 x 

ebx − eax
(iv) lim , where 0 < a < b
x →0 x
e bx − eax  0  e(b − a )x − 1
Sol. lim   = lim eax   (b − a)
x →0 x  0  x →0 (b − a)x
= (b – a) Ans.

1 − cos 5x  0 
(v) lim  
x →0 1 − cos 4x  0 

 5x 
sin 2   2
 2    5x 
 
 2 
2
 5x   5x 
2sin 2    
Sol. lim  2  = lim  2
2
=
25
Ans.
x →0 2sin 2 2x x →0 sin 2x 16
 4x 2

(2x) 2
x(e 2+ x − e 2 )  0 
(vi) lim
x → 0 1 − cos x
 
0
 e −1 
x
x  e2   x
Sol. lim  x 
= 2e2 Ans.
x →0 sin 2 ( x / 2 )
1 2  ( x 2 / 4)
( x / 4)
2

3 sin x − cos x  0 
(vii) lim
  
x→

x− 0
6
6
 3 1 
2 sin x − cos x 
Sol. lim  2 2  = lim 2sin (x –  / 6)
x→
 x−/6 x→
 ( –  / 6)
6 6
= 2 Ans.

ln(2 + x) + ln 0.5  0 
(viii) lim  
x →0 x 0
ln(1 + x / 2) 1 1
Sol. lim  = Ans.
x →0 x/2 2 2

104
1 − cos 2x
(ix) lim 2
x →0 x
| sin x |
Sol. lim  RHL = 1 & LHL = – 1
x →0 x
| sin x |
 lim does not exist Ans.
x →0 x
 tan 2x − 3x 
(x) lim 
x → 0 3x − sin 2 x

 
tan 2x
2−3
2x 2 − 3 −1
Sol. lim 2
= = Ans.
x →0 sin x 3 3
3− 2 x
x
x n − 3n
(xi) Find n  N, if lim = 405
x →3 x − 3

x n − 3n
Sol. lim = 405
x →3 x − 3

 n (3)n–1 = 405  n  3n–1 = 5  34  n = 5 Ans.

cos −1 (1 − x)
10. lim is equal to
x → 0+ x
(A) (B) (C) 1 (D) 0
cos−1 (1 − x)  0 
Sol. lim  
x →3 x 0

 −1 
  (−1) 
1 − (1 − x) 2
2 x
= lim−   = lim
x →0  1  x →0 x (− x + 2)

 
 2 x 

2
= = 2 Ans.
2

105
CONCEPT BUILDING - 2
 100
k 
  (x − 1)  − 100
1. lim  k =1  is equal to
x →2 (x − 2)
(A) 0 (B) 5050 (C) 4550 (D) –5050
 100
k 
  (x − 1)  − 100 0
  Put x − 1 = t
Sol. lim  k =1 
  ;
x →2 (x − 2) 0 x → 2  t →1

100

 (t) k
− 100
t + t 2 + t 3 + .... + t100 − (1 + 1 + ....1)
= lim k =1
= lim
t →1 (t − 1) t →1 t −1
(t − 1) + (t − 1) + .... + (t100 − 1)
2
= lim
t →1 (t − 1)
100 101
= 1 + 2 + 3 + .... + 100 = = 5050 Ans.
2

(4x − 1)3
2. lim is equal to
x →0  x2 
sin x ln 1 + 
 3 
(A) 9(ln 4) (B) 3(ln 4)3 (C) 12(ln 4)3 (D) 27(ln 4)2
3
 4x − 1 
  x
3
( n 4)
3

Sol. lim  x  = = 3( n 4)3 Ans.


x →0  x  2
1/ 3
ln 1 +  2
x  2 3 
sin x x
x x /3 3

 
x + 2 
lim 
− cos x 
3. is equal to (where [.] represents greatest integer function)
2  
x→

 
(A) 1 (B) 0 (C) – 2 (D) does not exist
   
 – +h+ 
Sol. RHL = lim  2 2  = lim  h  = 1
−
   h →0  sin h 
x→ +h 
2 cos  – + h    
  2 
   
 – −h+ 
LHL = lim  2 2  = lim  – h  = 1
−
   h →0  – sin h 
x → –h 
2 cos  – − h    
  2  

106
 
x + 2 
 lim = 1 Ans.
−  cos x 
2  
x→

 

4. Evaluate the following limits


3x 2 + 2x + 9
(i) lim
x → x2 +1
2 9
3+ + 2
3x 2 + 2x + 9    x x = 3 Ans.
Sol. lim   = lim
x → x 2 + 1    x → 1 + 1/ x 2

x 3 + x + cos 2 x
(ii) lim
x → 2x 3 + sin x
1 cos 2 x
1+ 2 +
x 3 + x + cos 2 x    x x3
Sol. lim   = lim
x → 2x + sin x    x → 2 + sin x / x 3
3

1
= Ans.
2
 1 2 x 
(iii) lim  2 + 2 + ..... + 2 
x → x
 x x 
 x +1
x 
1 + 2 + 3 + .... + x  2 
Sol. lim = lim
x → x2 x → x2
1 1 1
= lim 1 +  = Ans.
x → 2  x 2

n 3 − 2n 2 + 1 + 3 n 4 + 1
(iv) lim ,nN
n → 4
n 6 + 6n 5 + 2 − 5 n 7 + 3n 3 + 1

n 3 − 2n 2 + 1 + 3 n 4 + 1
Sol. lim
n → 4
n 6 + 6n 5 + 2 − 5 n 7 + 3n 3 + 1

n 3/2 1 − 2 / n + 1/ n 2 + n 4/3 1 + 1/ n 4
= lim
n →
n 3/2 1 + 6 / n + 2 / n 6 − n 7/5 1 + 3 / n 4 + 1/ n 7
1
1 + 1/6
n 3/2 + n 4/3 n = 1 Ans.
= lim 3/2 = lim
n → n − n 7/5 n → 1 − 1
n1/10

107
1
(3x 4 + 2x 2 ) sin + | x |3 +5
(v) lim x
x→– | x |3 + | x |2 + | x | +1

1
(3x 4 + 2x 2 )sin   – x 3 + 5
Sol. lim x
x→– – x + x – x +1
3 2

 (1/ x)  3
(3x 3 + 2x)  sin  – x +5
= lim  1/ x 
x→– – x3 + x 2 − x + 1
 2   sin (1/ x)  
3 + 2   −1+ 3
 x   (1 − x)  x
= lim
x→– 1 1 1
−1 + − 2 + 3
x x x
3 −1
= = – 2 Ans.
−1

7n + 5n − 4n +1
5. lim n +1 n n + 2 , n  N is equal to
n → 7 +2 +3
1 1
(A) (B) (C) 1 (D) zero
9 7

7 n + 5n − 4  4 n
Sol. lim ; Divide by 7n
n → 7  7 n + 2n + 9  3n
n n
5 4
1+   − 4 
= lim 7  7  = 1 Ans.
n n
n →
2 3 7
7 +   + 9 
7 7

     
6. lim n cos   sin   , n  N is equal to
n →
 7n   3n 
  
(A) (B) (C) (D) does not exist
3 4 6

     
Sol. lim n cos    sin  
n →
 7n   3n 

  
sin  
    
= lim cos       = Ans.
3n
n →
 7n     3 3
 
 3n 

108
 2 1 
7. lim  +  is equal to
x →1 1 − x 2 x −1 

1 1
(A) (B) − (C) –1 (D) does not exist
2 2

 2 1   1 2 
Sol. lim 
x →1 1 − x 2
+  = lim  − 2  ( – )
 x −1  x →1
 x −1 x −1 
 x +1− 2   1  1
= lim   = lim   = Ans.
x →1 (x − 1) (x + 1) x →1 x + 1
    2

  1 
8. lim  x − x ln 1 +   is equal to :
x →
  x 

1 3 1
(A) (B) (C) (D) 1
2 2 3

  1 
Sol. lim  x − x ln 1 +   ( – )
x →
  x 

 1 (1/ x )2 (1/ x )3 
= lim x − x  − + ...... 
x →  x 2 3 
 
1 2 1
= lim x − x + − = Ans.
x → 2 3 x 2

9. Evaluate the following limits using expansions :


(x + 2)3/2 − (a + 2)3/2
(i) lim
x →a x −a
3 3

Sol. = lim
( x + 2) 2 − ( a + 2) 2
x →a x −a
3 3

= lim
( a + h + 2) 2 − ( a + 2) 2
h →0 (a + h − a )
3

( a + 2 ) 1 +
3
h 2 3
2
 − ( a + 2 ) 2
 a + 2 
= lim
h →0 h

( a + 2 ) 2 1 + 
3
h 3
 + ... − 1
 a+2 2 
= lim
h →0 h
109
3
= a + 2 Ans.
2

(x + 2)1/2 − (15x + 2)1/5


(ii) lim
x →2 (7x + 2)1/4 − x

(x + 2)1/2 − (15x + 2)1/5  0 


Sol. lim  
x →2 (7x + 2)1/4 − x 0
1 1
( 2 + h + 2 ) 2 − 15 ( 2 + h ) + 2 5
lim 1
h →0
7 ( 2 + h ) + 2  4 − ( 2 + h )
1 1
 h  2  15h  5
1 +  − 1 + 
=lim 
4  32 
h →0 1
 7h  4  h 
1 +  − 1 + 
 16   2 

 h   3h  1 3 
1 +  − 1 +   − h
=lim  8   32  = lim  8 32 
h →0  7h   h  h →0  7 1 
1 +  − 1 +   − h
 64   2   64 2 
2
=− Ans.
25
tan 2 x
e x − 1 − sin x −
2 0
(iii) lim  
x →0 x3 0
2
 x 2 x3   x3  1  x3 
1 + x + +  −1−  x −  −  x + 
Sol. lim  2! 3  
3
3!  2  3 
x →0 x
 1 1 1 3
 + − x
= lim  3! 3! 3 6  = Ans.
1
x →0 x 6

(ln(1 + x) − ln 2)(3.4x −1 − 3x)  0 


(iv) lim  
x →1 [(7 + x)1/3 − (1 + 3x)1/2 ]sin(x − 1)  0 

[ln(2 + h) − ln 2]  3[4h − (1 + h)]


Sol. lim
h →0 sin h
[(8 + h)1/3 − (4 + 3h)1/2 ]  h
h

110
 h   (h n 4) 2  
3ln 1 +    1 + (h n 4) +  − (1 + h) 
 2   2!  
= lim
h →0  h 1/3  3h 1/2 
2  1 +  −  1 +    h
 8   4  
 h  h( n 4) 2 
n 1 +  h  n 4 + − 1
3  2  2 
= lim 
h →0 2 h
2  h 3h 
1 + − 1 − 
2  24 8 
3 ( n 4 –1)  h − 9
= lim = n (4 / e) Ans.
4 h→0  1 3  4
 − h
 24 8 

 −x 
2

 e 2 + cos x + x − 2 
2
10. lim   is equal to
x →0
 x 2 sin 2 x 
 
1 1 1 1
(A) (B) (C) (D)
4 6 12 8
x2

e 2
+ cos x + x 2 − 2 0
Sol. lim  
x →0 sin 2 x 0
x2  2  x2
x

 x 2   (− x 2 / 2) 2   x2 x2  2
1 −  +   + 1 − + +x −2
= lim  2   2!   2! 4! 
x →0 4
x
1 1  4
 + x
= lim  8 24 4
 = 1 Ans.
x →0 x 6

111
CONCEPT BUILDING – 3

1. Find the values of a and b so that :


1 − ax sin x + b cos x
(i) lim may have a finite limit.
x →0 x4
 x3   x 2 x 4 
1 − ax  x −  + b 1 − + 
Sol. lim  3!   2! 4!  = finite
x →0 4
x
 1 + b   +b/2  a b 
a
 lim  4  −   +  +  = finite
x →0  x   x 2   3! 4! 
b
 1+b=0& a+ =0
2
1
 b = – 1 & a = Ans.
2
(ii) lim
x →
( )
x 4 + ax 3 + 3x 2 + bx + 2 − x 4 + 2x 3 − cx 2 + 3x − d = 4
Sol. Rationalise
 ( 0o )
b−3 2+d
(a − 2)x + (3 + c) + + 2
 lim x x =4
x → a 3 b 2 2 c d
1+ + 2 + 3 + 4 + 1+ − 2 − 4
x x x x x x x
b−3 2+d
(a − 2)x + (3 + c) + + 2
 lim = x x =4
x → 2
3+ c
 a − 2 = 0, = 4, b  R, d  R
2
 a = 2, c = 5, b  R, d  R Ans.

axe x − b ln(1 + x) + cxe− x


(iii) lim =2
x →0 x 2 sin x
 x2   x 2 x3   x2 
ax 1 + x +  − b  x − +  + cx 1 − x + 
Sol. lim  2!   2 3  2!  = 2
x →0 sin x
x2 x
x

  b  
 a − b + c   a + 2 – c   a b c  
 lim  +  +  – +  = 2
x →0  x 2
  x   2! 3 2!  

 

a −b+c = 0 …(1)
b
a+ −c = 0 …(2)
2

112
a b c
− + =2 …(3)
2 3 2
Adding (1) and (2),
b
2a − =0
2
b
a=
4
1
Adding  ( 2 ) + ( 3) we get,
2
b
a+ =2
12
b b
+ =2
4 12
b=6
9
and c = Ans.
2

tan(a + 2h) − 2 tan(a + h) + tan a


2. Find the value of lim
h →0 h2
tan(a + 2h) − 2 tan(a + h) + tan a  0 
Sol. lim  
h →0 h2 0
Apply L-H Rule
2sec 2 (a + 2h) − 2sec 2 (a + h)  0 
= lim  
h →0 2h 0
Again apply L-H Rule
4sec2 (a + 2h)  tan (a + 2h) – 2sec 2 (a + h) tan (a + h)
lim
h →0 1
= 4sec a  tan a – 2sec a  tan a = 2sec2 a tan a Ans.
2 2

sin(a + 3h) − 3sin(a + 2h) + 3sin(a + h) − sin a


3. lim is equal to
h →0 h3
(A) cos a (B) – cos a (C) sin a (D) sin a cos a
Sol. Apply L-H Rule
3cos (a + 3h) − 6cos(a + 2h) + 3cos (a + h) 0
lim  
h →0 3h 2 0
– 3sin (a + 3h) + 4sin(a + 2h) – sin (a + h)  0 
= lim  
h →0 2h 0
L-H Rule

113
– 9 cos (a + 3h) + 8cos(a + 2h) – cos (a + h)
lim
h →0 2
− 9 cos a + 8cos a – cos a
= = − cos a Ans.
2

4. Evaluate the following limits :


x
x 
tan
 2
(i) lim  tan 
x →1
 4 
x
x 
tan
 2
Sol. (i) lim  tan  (1 )
x →1
 4 
 x 
tan −1
 4 
lim cot x
0
=e x→1 2
 
0
 x  
sec2   
 4  4
2  x  
–( 2 ) 
2 1
lim – cosec   
=e x →1  2  2
=e 2
= e−1 .

 1 + 3x 
x

(ii) lim  
x → 1 + 4x
 

 

 1 + 3x   3 
x
3
Sol. lim   =   = 0 Ans. 1
x → 1 + 4x
  4 4

x
sec
(iii) lim(1 +  ln x) 2
x →1
x
sec
Sol. lim(1 +  ln x) 2
(1 )
x →1
 x 
lim (  n x) sec  
=e x→1  2 

 nx
lim
x
0
x →1
cos
=e 2
 
0

 
x
 x  
lim – sin  
=e x →1  2  2
= e−2 Ans.

1
  x
(iv) lim  tan  + x  
x →0
 4 
1  
tan  + x  −1
   x lim 4 
0
Sol. lim  tan  + x   (1 ) = e x→0 x
 
x →0
 4  0
sec2 (  /4 + x)
lim
=e x →0 1
= e( 2 )2
= e 2 Ans.

114
tan x
5. The value of lim([x])

is equal to (where [.] denotes the greatest integer
x→
2

function)
(A) 0 (B) 1 (C) e (D) e–1

Sol. lim([x]) tan x
= (Exact1) = 1 Ans.

x→
2

 x
n
x
1/n

6. lim  sin + sin  , (x  0) is
n →  n n 
 
− x2
(A) e − x2
(B) e 2 (C) 1 (D) e–1
n 1/n
x x
lim sin + sin
Sol. n → n n
0 0
 1

 x n
 = y (Let)
lim 0 + sin
n →  n 
 
x
n sin
n
 n y = lim (  / )
n → n
Apply L-H Rule
1 x  x 
 ln y = lim  cos   − 3/2 
n → sin x / n n  2n 
–1 x / n x
= lim   cos =0
n → 2n sin x / n n
y = e0 = 1

7. Evaluate the following limits :


lim(| x |)x ( 0o )
2
(i)
x →0

Let y = lim(| x |) x
2
Sol.
x →0

 ln y = lim x 2  ln (| x |) (0 × )
x →0
ln| x |
= lim (/)
1/ x 2
x →0

Apply L-H Rule


1/ x – x2
ln y = lim = lim =0
x →0 –2 / x 3 x →0 –2

 y = e0 = 1 Ans.

(ii) lim− (tan x)cos x (o)



x→
2

115
Sol. Let y = lim− (tan x)cos x

x→
2

    
 ln y = lim cos  – h   ln  tan  – h  
x → −h 2   2 
2

ln (cot h)   
= lim  
cos ec h   
h →0

Apply L-H Rule


1
 − cos ec 2 h
cot h
lim
h → 0 − cos ec h  cot h

tan 2 h h
= lim 2
 h = 0
h →0 h sin h
 y = eo = 1 Ans.

(iii) lim([x])1− x , where [.] denotes greatest integer function


x →1−

Sol. lim ([1 − h])1−1+h = lim ([1 − h]) h [Eact 0]→ 0


x →1–h h→0 1

= 0 Ans.
(iv) lim e tan x
x→
2

Sol. RHL = lim+ e tan x = e→ –  = 0



x→
2

LHL = lim− e tan x = e→  = 



x→
2

 lim e tan x does not exist.



x→
2

[1.3x] + [2.4x] + .... + [n.(n + 2)x]


8. Evaluate lim , where [.] denotes greatest
n → n3
integer function.
[1.3x] + [2.4x] + .... + [n.(n + 2)x]
Sol. Let lim =
n → n3
 1  3x – 1 < [1  3x]  1  3x
2  4x – 1 < [2  4x]  2  4x
n(n + 2)x − 1  [n(n + 2)x]  n(n + 2) x
Add & Divide by n 3and take lim
n →
n 

 n(n + 2)x − n  n(n + 2)x


 lim n =1
  lim n =1
n → n3 n → n3

116
 (n 2
+ 2n)x − n  (n 2
+ 2n)x
 lim   lim
n → n3 n → n3
 n(n + 1) (2n + 1) 2n(n + 1)   n(n + 1) (2n + 1) 2n(n + 1) 
 + x −n  + x
 lim  6 2    lim  6 2 
n → 3 3
n n → n

  1  1     1  1 
 1 +  2 +   1 1     1 +  2 + 
  
lim   n  n+  n  n+
1 1 1
 +   x –    lim   +  x
n → 
 6  n n2   n2  n →  6  n n2  
 
x x
  
3 3
x
 By sandwich theorem, = Ans.
3

9. If [x] denotes greatest integer less than or equal to x, then


1 2
lim 3
([1 x] + [22 x] + ....... + [n 2 x]) is equal to
n → n
x x x x
(A) (B) (C) (D)
2 3 6 4
1 2
Sol. lim 3 ([1 x] + [22 x] + ....... + [n 2 x])
n → n

12 x − 1  12 x   12 x
22 x − 1   22 x   22 x

n 2 x − 1   n 2 x   n 2 x
add & divide by n 3
n n

 n2x − n n x 2

lim n =1
  lim n =1 3
n → n3 n → n
 n ( n + 1)( 2n + 1) n   n ( n + 1)( 2n + 1) 
lim  x −    lim  x
n →
 6n 3 n3  n →
 6n 3 
1 1
x  x
3 3
x
 By sandwich theorem, = Ans.
3
x −1
2n
10. If f (x) = lim
n → x 2n + 1
, n  N find range of f(x).

(x 2 )n − 1
Sol. f (x) = lim 2 n
n → (x ) + 1

117
Case i) x2 =1  x = ± 1
(1)n − 1 1 − 1
 f (x) = lim n = =0
n → (1) + 1 1+1
Case ii) x2 < 1  x  (– 1, 1)
(x 2 )n − 1 0 − 1
 f (x) = lim 2 n = = –1
n → (x ) + 1 0 +1
{(Value of x  ( −1,1) ) →  = 0}
case iii) x2 > 1  x > 1 or x < – 1
(x 2 )n − 1   
 f (x) = lim 2 n  
n → (x ) + 1   

 
(x )
2 n 1 − 1 n 
 ( x2 ) 
lim   =1
n →  
( x ) 1 + 2 n 
2 n  1 

 (x ) 
f(x) = 1
 Range : - f(x)  {– 1, 0, 1} Ans.

118
EXERCISE-1
x2 − x
Q.1 Lim
x →1 x −1

( x )4 − x  0  x ( x − 1)( x + 1 + x )
Sol. Lim   = Lim
x →1 x − 1  0  x →1 ( x − 1)
= 3 Ans.

13
x−7 x
Q.2 Lim
x →1 5
x−3x

x1/13 − x1/7  0 
Sol. Lim  
x →1 x1/5 − x1/3  0 
Apply L-H Rule
1 −12/13 1 −6/7 1 1
x − x −
45
= Lim 13 7 = 13 7 = Ans.
x →1 1 −4/5 1 − 2/3 1 1 91
x − x −
5 3 5 3

x 2 − x.1nx + 1nx − 1
Q.3 Lim
x →1 x −1
(x 2 − 1) – 1nx(x − 1)  0 
Sol. Lim  
x →1 (x − 1) 0
(x − 1)[x + 1 − ln x]
= Lim
x →1 (x − 1)
= 2 Ans.

 100 k 
  x  − 100
Q.4 Lim  K =1 
x →1 x −1
 100 k 
  (x)  − 100 0
 
Sol. lim  k =1 
  ;
x →1 (x − 1) 0

x + x 2 + x 3 + .... + x100 − (1 + 1 + ....1)


lim
x →1 x −1

119
(x − 1) + (x 2 − 1) + .... + (x100 − 1)
= lim
x →1 (x − 1)
100 101
= 1 + 2 + 3 + .... + 100 = = 5050 Ans.
2

2 x + 3x1/3 + 5x1/5
Q.5 Lim
x → 3x − 2 + (2x − 3)1/3

2 x + 3x1/3 + 5x1/5   
Sol. Lim  
x → 3x − 2 + (2x − 3)1/3   

 1 1 
x1/2  2 + 3  1/6 + 5  −3/10 
= Lim  x x  = 2 Ans.
x →  2 1  3 
1/3
3
x1/2  3 − + 1/6  2 −  
 x x  x 

1 + 3 tan x
Q.6 Lim
x→
 1 − 2 cos 2 x
4

1 + (tan x)1/3  0 
Sol. Lim  
 1 − 2 cos 2 x  0 
x→
4

Apply LH Rule
1/ 3(tan x) −2/3  sec 2 x
Lim
 4sin x cos x
x→
4

1 (−1) −2/3  (– 2) 2
=
3  1  1 
4  − 
 2  2
−1
= Ans.
3

sec 4x − sec 2x
Q.7 Lim
x→0 sec 3x − sec x
sec 4x – sec 2x  0 
Sol. Lim  
x→0 sec3x − sec x  0 
cos 2x − cos 4x cos x  cos 3x
= Lim 
x→0 cos x − cos 3x cos 2x  cos 4x

120
sin 3x  sin x 3
= Lim = Ans.
x→0 sin 2x  sin x 2

 p q 
Q.8 Lim  −  p, q  N
 1− x 1− x 
x→ 1 p q

 p q 
Sol. Lim  − q 
( – )
 1− x 1− x 
x→ 1 p

 p(1 − x q ) – q(1 − x p )  0


Lim  p+q   
x→ 1
 1− x − x + x
p q
 0
Apply L–H Rule
−pqx q −1 + pqx p −1
lim
x →1 −qx q −1 − px p −1 + ( p + q ) x p + q −1

pq ( x p − x q ) 0
lim , 
x →1 (p + q) x p+q
− px − qx  0 
p q

pq ( px p −1 − qx q −1 )
lim
(p + q) x p+q −1 − p2 x p−1 − q 2 x q −1
x →1 2

pq ( p − q ) p − q
= Ans.
2pq 2

Q.9 Find the sum of an infinite geometric series whose first term is the limit of the
tan x − sin x
function f(x) = as x → 0 and whose common ratio is the limit of the
sin 3 x
1− x
function g(x) = as x → 1.
(cos −1 x) 2
tan x – sin x 0
Sol. First term of G.P., a =  
sin 3 x 3 0
x
x3
 x3   x3 
x + −x −  1 1 1
 lim 
3  3!  = + =
x →0 3
x 3 6 2
0
Common ratio of G.P. r = lim  
x →0  0 

Apply L-H Rule

121
1

2 x 1− x2
r = lim = lim
x →1 −1 x →1 4 x  (cos −1 x)
2 cos −1 x 
1− x2

1 1− x2
 r = lim
4 x →1 cos−1 x
Again apply L-H Rule
− 2x

r = lim 2 1 − x =  r = Ans.
1 2 1 1
4 x →1 −1 4 4
1− x 2

a 1/ 2 2
 S = = = Ans.
1− r 1− 1 3
4

et + e− t
Q.10 Lim (x − l n cosh x) where cosh t = .
x → 2

  ex + e− x 
Sol. Lim  x − n    ( – )
x →
  2 

 e2x + 1   2e2x 
= Lim n e − n  x
 = Lim n  2x 
x →  2ex  x →  e +1 
 2 
= Lim n   = ln 2 Ans.
x →  1 + 1/ e 
2x

Q.11 (a) Lim


(
cos −1 2x 1 − x 2 );
1 1
x→
2 x−
2
Sol. (a) Put x = sin 
cos −1 ( sin 2 )  / 2 − sin −1 ( sin 2 )
 Lim = Lim
→  /4 1 →  /4 1
sin  − sin  −
2 2
 / 2 − (  − 2 ) 2 ( / 4 + h) −  / 2
RHL= Lim = Lim
→ /4 + h 1 h → 0 sin (  / 4 + h) − 1/ 2
sin  −
2

122
2h 0
= Lim  
h→0 sin ( / 4 + h) − 1/ 2  0 
Apply L-H Rule
2
 RHL = Lim =2 2
h→0 cos ( / 4 + h)
 / 2 − 2  / 2 − 2( / 4 − h)
LHL = Lim = Lim
→ /4 – h 1 h → 0 sin (  / 4 – h) − 1/ 2
sin  −
2
2h 0
= Lim  
h→0 sin ( / 4 – h) –1/ 2  0 
Apply L-H Rule
2
 LHL = Lim = –2 2
h→0 cos ( / 4 – h)  (–1)
 LHL  RHL

cos −1 (2x 1 − x 2 )
 = Lim does not exits.
1 1
x→
2 x −
2

1− sin 2x
(b) Lim
x→ 4  − 4x
1 − sin 2x 1 + sin 2x
Sol. Lim 
x→ 4  − 4x 1+ sin 2x

1 − sin 2x 1
= Lim 
x→ 4  − 4x 2
2 | sin (  / 4 − x) |
= Lim
2 x → 4 4 ( / 4 − x)
1 −1
 RHL = & LHL =
4 4
1− sin 2x
 Lim does not exist.
x→ 4  − 4x

[x]2 + 15[x] + 56
(c) Lim where [] denotes the greatest integer function
x →−7 sin(x + 7)sin(x + 8)

[x]2 + 15[x] + 56
Sol. Lim
x →−7 sin(x + 7)sin(x + 8)

Exact 0
= = 0 Ans.
tends to 0

123
1 − tan x  0 
Q.12 Lim  
x→ 4
1 − 2 sin x  0 
Sol. Apply L-H Rule
− sec2 x
 Lim = 2 Ans.
x→ 4
− 2 cos x

8  x2 x2 x2 x2   0 
Q.13 Lim 1 − cos − cos + cos cos   
x→0 x8  2 4 2 4  0

8  x 2  x2 
Sol. Lim 1 − cos 1 − cos 
x→0 x8  2  4 
2sin 2 x 2 / 4 2sin 2 x 2 / 8 1 1
= Lim 8  2 2
 2 2
 2 2
x→0 (x / 4) (x / 8) (4) (8)
8 2 2 1
= = Ans.
16  64 32

2 − cos  − sin   0 
Q.14 Lim  
→ 4 (4 − ) 2 0
Sol. Apply L–H Rule
sin  − cos  0
 Lim  
→ 4 2 (4 − )  4 0

cos  + sin  2 / 2 1
= Lim = = Ans.
→ 4 8 4 32 16 2

2− cos x − 1  0 
Q.15 Lim  
x → 2 x(x −  )
2 0

2− cos x − 1  0 
Sol. Lim  
x→ 2 (x 2 − x / 2)  0 
Apply L-H Rule
2− cos x  (sin x)  ln 2 2
= Lim = ln 2 Ans.
x→ 2 (2x −  / 2) 

a sin x − sin 2x
Q.16 If Lim is finite then find the value of 'a' & the limit.
x→0 tan 3 x

124
a sin x − sin 2x  0 
Sol. Lim   = Finite
x→0 tan 3 x 3 0
3
.x
x
a(x − x 3 / 3!) − (2x − (2x)3 3!)
 Lim = Finite
x→0 x3
 a − 2   − a 8  
 Lim  2  +  +   = Finite
x→0
 x   6 6  
−a 8
 a – 2= 0 and limit = + = 1 Ans.
6 6

a
Q.17 (a) Lim tan −1 , where a R ;
x →0 x2
 2x x
(b) Plot the graph of the function f(x) = Lim  tan −1 2 
t →0
  t 

 tan −1 (+ ) ; a  0
 a  
Sol. (a) Lim tan −1  2  =  tan −1 (– ) ; a  0
x →0  x   −1
 tan (0) : a = 0


2 ; a  0

−1  a  
Lim tan  2  =  – ; a  0 Ans.
x →0 x  2

0 ; a = 0
2x x
(b) f(x) = Lim tan −1  2 
t →0  t 
 2x −1
  tan (+ ) = x ; x  0

= 0 ; x=0
 2x
 tan −1 (– ) = – x ; x  0

 f(x) = | x |

125
Q.18 Lim [ln (1 + sin²x). cot(ln2 (1 + x))]
x→0

l n (1 + sin 2 x) 0
Sol. = Lim  
x→0 tan (ln 2 (1 + x)) 0

n(1 + sin 2 x) n 2 (1 + x) 1
= Lim  sin 2
x  2
x→0 2
sin x tan n ((1 + x))
2
n (1 + x)

sin 2 x x2
= Lim  = 1 Ans.
x→0 x2 n 2 (1 + x)

(ln(1 + x) − ln 2)(3.4x −1 − 3x)


Q.19 Lim
[(7 + x) 3 − (1 + 3x) 2 ].sin(x − 1)
1 1
x →1

  1+ x  1   4x −1 − x 
Sol. Lim ln    .3.  
  2  sin (x − 1)   (7 + x) − (1 + 3x) 
x →1 1/3 1/2

0
Here  
0

2 1 1  n 4  (4 x −1 ) − 1 
=   .3  
1 + x 2 cos(x − 1)  1 (7 + x) −2/3 − 3 (1 + 3x) −1/2 
3 2 
1  n 4 −1 
 .3
2 1.1 − 3.1 
 
3 4 2 2
1  n 4 –1  –9 4
 .3.    ln
2 1− 9 4 e
 
 12 

n
  
Q.20 If l = Lim
n →
  (r + 1) sin r + 1 − r sin r 
r =2
then find { l }. (where { } denotes the

fractional part function)


          
Sol Lim  3sin − 2sin + 4sin − 3sin + 5sin − 4sin + ... + (n + 1)sin   − nsin 
n →
 3 2 4 3 5 4  n +1  n

    
Lim  −2sin + (n + 1) sin    − 2
n →
 2  n +1  

126
 − 2 =  − 3 Ans.

(3x 4 + 2x 2 )sin 1x + | x |3 +5
Q.21 Lim
x →− | x |3 + | x |2 + | x | +1
 1
(3x 4 + 2x 2 )  sin  + | x |3 +5
Sol. Lim  x
x →− | x | + | x | + | x | +1
3 2

3x 3 + 2x 2 − x 3 + 5
Lim
x →− (x)3 − (x) 2 − (x) + 1

2 5
3++
Lim x x3
x →− 1 1 1
−1 − − 2 − 3
x x x
= – 2 Ans.
(x 3 + 27 )1n (x − 2)  0 
Q.22 Lim  
x →3 x2 − 9 0
(x 3 + 27 ) l n (1 + (x − 3) )
Sol. Lim 
x →3 (x + 3) (x – 3)
(27 + 27) 1
= =9
3+3

27 x − 9x − 3x + 1  0 
Q.23 Lim  
x→0 2 − 1 + cos x  0 

(9x − 1) (3x − 1) 2 + 1 + cos x


Sol. Lim 
x→0 2 − 1 + cos x 2 + 1 + cos x

(9x − 1) (3x − 1)  ( 2 + 1 + cos x )


= Lim
x→0 (1 − cos x)

 9 x − 1   3x − 1 
  x
2

= 2 2 Lim  x   x  = 4 2 × ln 9 × ln 3
x→0 sin 2 x x 2
2 2 
x /4 4
= 8 2 (ln 3)2 Ans.

127
x
Q.24 Let f (x) = , x  0 and g(x) = x + 3, x 1
sin x
= x 2 − 2x − 2, 1  x  2
= 2 − x, x  0
= x − 5, x2

find LHL and RHL of g ( f (x) ) at x = 0 and hence find Lim g ( f (x) ) .
x →0

Sol. LHL = lim g(f (0 – h)) =lim g(2 + h)) =lim (2 + h − 5)


x →0–h h →0 h →0
0 2

LHL = – 3
 h 
RHL = lim g(f (0 + h)) =lim g   {sin x < x}
x →0 + h h →0  sin h 
0

 h  2  h  
=lim   − 2  − 2 = 1 − 2 − 2 = – 3
h →0  sin h   sin h  

RHL = – 3
 limg(f (x)) = – 3 Ans.
x→0

Q.25 Let Pn = a Pn−1 − 1 ,  n = 2, 3,.......and Let P1 = ax – 1 where a  R+ then evaluate

Pn
Lim .
x →0 x
P1 a x −1  0 
Sol. Pn = a Pn−1 − 1  Lim = Lim   = ln a
x →0 x x →0 x  0 
−1)
* P2 = a P1 − 1  P2 = a (a −1
x

a (a −1) − 1 a x − 1
x
P
 Lim 2 = Lim x 
x →0 x x →0 (a − 1) x
= ln a × ln a = (ln a)2
* P3 = a P2 − 1

P3 a P2 − 1 a P2 − 1 P2
 Lim = Lim = Lim 
x →0 x x →0 x x →0 P2 x

a P2 − 1 P
= Lim  Lim 2 = ln a × (ln a)2 = (ln a)3
x →0 P2 x →0 x

Pn
 In similar way, Lim = (ln a)n Ans.
x →0 x

128
1  1 1 + ax 

3 
x  1 + x 1 + bx 
Q.26 If the Lim exists and has the value equal to , then find the
x →0

1 2 3
value of − + .
a b
1 1 + bx − (1 − ax) (1 + x)1/ 2 
Sol. Lim  
x →0 x3  1 + x (1 + bx) 

  11  1  1  1   
   − 1  − 1 − 2   
 1 2  2  2  2  2  
 (1 + bx) − (1 + ax) 1 + x + x +
2
x + ....
3

1
 Lim 3   2 2! 3!  
x →0 x  1 + x (1 + bx) 
 1 1 

1  1 a   1 a
1 + bx − 1 −  + a  x +  −  x 2 +  − +  x 3 ....
 Lim 2  8 2  16 8 
x →0 3
x
 b − a − 1/ 2   1/ 8 − a / 2   a 1 
 Lim 
x →0 
+  +  −  = Finite
x2   x   8 16 
1 1 a 1 1
 b−a − = 0, − = 0 & l = −
2 8 2 8 16
1 3 1
 a= , b= & l= −
4 4 32
1 2 3
 − + = 4 + 64 + 4 = 72 Ans.
a l b

Q.27 Let {an}, {bn}, {cn} be sequences such that


(i) an + bn + cn = 2n + 1 ; (ii) anbn +bncn + cnan = 2n – 1 ; (iii) anbncn = – 1 ;
(iv) an < bn < cn

Then find the value of Lim na n .


n →

Sol. Let the cubic equation roots are an , bn & cn be


x3 − (2n + 1)x 2 + (2n −1)x + 1 = 0 : an < bn < cn
 (x – 1) (x2 – 2nx – 1) = 0

2n  4n 2 + 4
 x = 1 or = n  n2 +1
2

The root of this equation are 1, n − n 2 + 1 & n + n 2 + 1

129
 a n = n − n 2 + 1 { an < bn < cn}

n(n 2 − n 2 − 1) 1
 lim = − lim
n →
n + n2 +1 n →
1 + 1 + 1/ n 2
−1
= Ans.
2

Q.28 If n  N and an = 22 + 42 + 62 + ....... + (2n)2 and bn = 12 + 32 + 52 + ..... + (2n – 1)2.

a n − bn
Find the value Lim .
n → n
 n(n + 1) (2n + 1) 
Sol. a n = 22 [12 + 22 + 33 + ...... + n 2 ] = 22   ….(1)
 6 
and an + bn 12 + 22 + 32 +……+ (2n – 1)2 + (2n)2
(2n) (2n + 1) (2(2n) + 1)
an + bn = ….(2)
6
2n (2n + 1)(4n + 1) − 4n(n + 1) (2n + 1)
(2) – (1)  bn =
6
2n(2n + 1)[4n + 1– 2n − 2]
bn =
6
n(2n + 1) (2n − 1)
 bn =
3
4n(n + 1) (2n + 1) n(2n + 1) (2n –1)

a n – bn 6 3
 lim = lim
n → n n → n
1 (
= lim 4n 2 + 6n + 2 − 4n 2 − 1 ) ; Rotationalise
n → 3

1 (6n + 3) 1 6+3/ n
= lim = lim
n → 3
4n + 6n + 2 + 4n –1 n → 3 4 + 6 / n + 2 / n 2 + 4 –1/ n 2
2 2

1 6 3
=  = Ans.
3 (2 + 2) 2

Q.29 At the end points A, B of the fixed segment of length L, lines are drawn meeting at C
and making angles  and 2 respectively with the given segment. Let D be the foot of
the altitude CD and let x represents the length of AD. Find the value of x as  tends to
zero i.e. Lim x .
→0

Sol.

130
C

A  2
B
x D L– x

y y
tan  = & tan 2 =
x L−x
 x tan  = (L – x) tan 2
2 tan 
 x tan  = (L − x) 
1 − tan 2 
2L
 x=
3 − tan 2 
2L 2L
 lim x = lim = Ans.
→ 0 → 0 3 − tan 
2
3
Q.30 At the end-points and the midpoint of a circular arc AB tangent lines are drawn, and
the points A and B are joined with a chord. Prove that the ratio of the areas of the two
triangles thus formed tends to 4 as the arc AB decreases indefinitely.
Sol.

O
r r
 
A P B

D R E

 
− −
2 C 2

Let radius of circle be r.


In  AOP, AP = r sin   AB = 2AP = 2r sin 
  AP
In APC, tan  −   =
2  PC

131
r sin 
PC =
cot 
r sin 2 
PC =
cos 
1
 Area of ABC = AB  PC
2
= r2tansin2
   DR
In DRC, tan  −   =
2  RC
 DR = RC cot 
DR = (OC – OR) cot 
 r  r(1 − cos )
DR =  − r  cot  =
 cos   sin 
1 1
 Area of DEC =  DE  RC =  (2DR)  RC
2 2

 r (1 − cos  )
2
(1 − cos )  r
2
1
=  2r  −r =
2 sin   cos   sin  cos 
Now,  AB = 2r sin  ,  If AB → 0   → 0

Area ( ABC) sin 2  tan 


 lim = lim  sin  cos 
AB →0 Area (  CDE)
(1 − cos  )
→ 0 2

sin 4  (1 + cos  )
2

= lim = 22 = 4 . Hence Proved Ans.


→0 sin 
4

a tan x − a sin x
Q.31 Evaluate Lim , a > 0. [REE 2001, 3 out of 100]
x →0 tan x − sin x
 a (tan x –sin x) − 1   0 
Sol. lim a sin x     = n a Ans.
x →0  tan x – sin x  0 

2 −1  1  
Q.32 Find the value of Lim   (n + 1) cos  n  − n  . [ JEE ' 2004, 2 out of 60]
n →
   
2 1 
Sol. Lim  (n + 1) cos −1   − n  ( – )
n →
 n 

132
Put n = 1/t
2 1  1
 lim   + 1 .cos −1 (t) − 
t →0    t  t
2( ) −1 
  1 + t .cos (t) − 1   0 
= lim    
t →0  t  0
Apply LH Rule
  −1 –1 
cos (t) + (1 + t)  
 1+ t2 
= lim
t →0 1

=  
2 
 2
2
− 1 = 1 − Ans.

133
SOLUTIONS
EX#2
2 8 x 2 3
1. Lim  2 x  3 
x  2
 2x  5 
Ans. e –8

8 x2 3
3
 2x 2  3   2x 2  3 2  x 2
Sol. lim  2  {1 } at x  ,  1 & 8x2 + 3  
x   2x  5
  2x 2  5 2  5
x2

 2 x2 3 
lim (8x 2  3)  2 1 
x   2x  5 
  e
6
–16–
 –2  lim x2
lim (8x 2  3)  2  x 5
2 2
x   2x  5 
 e  e x
 e–8

x
2. Lim  x  c   4 then find c
x 
 xc
Ans. c = ln2
x
 xc
Sol. lim 
x  x – c
 4 {1}
 
 x c 
lim x  –1
 x–c 
e x
4
2c
 2c  c
lim x   lim 1–
e x x–c
4  e x  x
 4  e 2c  4
 2c  ln e = ln 4
ln4
c ln2
2

1/ x
 1  x 1/ x 
3. Lim  
x 0  e 
 

134
1

2
Ans. e
 
1
1/ x  
 (1  x)1/ x   x
(1  x) 
Sol. lim    ln (1 x ) 
x 0
 e   
e x
 
1/ x
  x 11 2   x2 x3
x –  ...... 
2 3
 e  1 – 2  24 x ......    e x

 lim    {1 }  
x 0
 e   x x2
1–  ...... 
   e 2 3 
 2 
– 
x x 
 x 11 2   1 2 3 
 1–  x ...... –1 e  e   e  ......
lim 
2 24   
 e x 0 x
     
  x x2   x2  
–1 11
lim – x .......  e 1 –   1   ..... .....
 e x 0 2 24
  2 8  3  
 

1
  x 11 2  
 e 2 e 1 –  x  ....... 
  2 24  

2 n 2  n 1
 n 2  n 1 
4. Lim  
n  n 
 
Ans. e–1

2 n 2  n –1
at n   
 n 2  n –1   
1 1
Sol. lim 
n   n


, 1 

 2 1 – 
   n  n –1  n n  1
 n 1 

(2 n 2  n –1)  n 2  n –(n 1) 


 
lim
 e n n

(2 n2  n –1)(n 2 n –(n 1)2 )


lim
[ n2  n (n 1)]n
 en 

 1 1 
n  2 1 – 2 (– n –1)
 n n 
 1 1
lim n  1 1 .n
n n
 e n 

2 
1 0 –0  –1–0 

  1 0 1 0 
e
–2
–1
 e 2  e

135
π
5. Lim x 2 sin n cos
x  x

– 2
Ans.
4

 1
 sin n cos z Let, x 
Sol. lim x sin n cos  lim
2  z
x  x z  0 z2 If x  , z  0

sin  n cos z n cos z


 lim
z 0  n cos z z2

n cos z  sin x 
 lim  lim  1
z 0 z2  x 0 x 

n cos z
 lim
z 0 2z 2

 sin  z   
 lim  {by L’ Hospital Rule}
z 0 cos  z  4z 

–  2 sin z – 2 – 2
 lim  lim 
z 0 4 (cos z)  z z 0 4cos z 4

x2
  a 
6. Lim  cos  2   x    aR
x     1  x   
  
2 2
Ans. e –2 a
x
Sol. Put = y  y  1 as x 
1 x

1 1 1 y
1– y 1+x= x= 1  x =
1 x 1 y 1 y 1 y
2
 y   y2 
Lim cos 2 ya 1 Lim( 2sin 2 ya )
y1  1 y  = y1

 (1 y) 2 
e e
 sin 2  (1 ya ) 2 (1 ya )2 2 
Lim  2 2 a 2
. .y 
y1
  (1 y ) (1 y)2 
e

136
2
 1 ya  2  xn 1 
lim 2 
x1
 1y 
 .y 2 2  Lx
im  n
e  e–2 a  1 x 1

tan 2x
7. Lim  tan x 
x 1 4 

Ans. e–1
tan 2x
Sol. Lim  tan x  1 

x 1 4 

 x    x  
lim tan   tan   1
x 1  2   4  
e
  x  
 tan  4 1
  
lim 
x 1  x 
cot 
 2 
e

 2  x 
sec  
4  4 
lim
x 1   x  1
cos ec2    2
e 2  2 
e 2
 e1
1
Lim  x  1  cos x 
x
8. x 0
 x 
1

Ans. e 2

 x  1  cos x 0
if x  0, x
 
0
1  
 x –1  cos x  x Using L'Hospital Rule 
Sol. lim   {1} if x  0, 1  sin x  1 
x 0
 x     
 

 x–1cosx  1
–1.
  xlim
0 x x
e
 1 cos x  1
lim  .
 x 0  x x
e
–1 cos x
lim
 x 0 x2
e

137
x
–2sin 2  
lim  2
x0  x2 
 4 
 4 
 
e
1

 e 2

nx
 x1 1 1 1
x 
Lim  a 1  a 2  a 3 ..... a n 
x x

9. x   n  where a1,a2,a3,......an > 0


 

Ans. a1 a2.........an
nx
 x 
x

x

x

a  a  a  .......  a
Sol. lim  1 2 3 n 
x   n 
 
 
n
t t t
 a  a  .......  a 
1  , x  1t
t
1 2 n 
 lim  
t 0
 n 
n  a 1t  a 2t  ....... a nt 
lim  –1
 t0 t  n 
e

 a1t  a t2  ....... a nt –1–1–1.......–1 


 
n n times
lim  
t0 t  n 
 e 


 a t –1 a t –1 a t –1 
lim  1  2  ...... n   a X –1 
 e  t
t0  t t 
 lim  n a 
 x0 x 

 e[ln a1 ln a 2 ...... ln a n ]


 a1 a2.........an
sin 1 (1 {x}).cos 1 (1 {x})
10. Let f(x) = then find xLim
0
f(x) and xLim
0
f(x), where {x} denotes the frac-
2{x} . (1  {x})
tional part function.

 
Ans. ,
2 2 2

sin –1 (1 – {x}) cos –1 (1 – {x})


Sol. f (x) 
2{x}  (1 – {x})

138
sin –1 (1– x)  cos –1 (1– x) at x  0 
lim f (x)  lim  
x 0 x 0 2x (1– x) {x}  x 

sin –1 (1– x) cos –1 (1– x)


 lim 
x0 1– x 2x

 sin –1 (1) t [Let  cos –1 (1 – x)  t


lim .
t 0 1 t
2  2 sin 1 – x  cos t
2
x  1 – cos t
–1
sin (1) t t
lim . x  2sin 2 ]
 t 0 1 t 2
2  2 sin
2

 t/2 
 lim . 
t 0 2 t 2
sin
2

sin–1  x  cos–1  x  at x  0 – 


lim f(x)  lim–  
x0– x0 2  1 x   x  {x}  x  1
–1
sin –1 x cos   x    cos 1 x 
 lim–   lim 1 
x 0 x 2  1  x x 0 2 2 2

ae x  b cos x  ce  x
11. Find the values of a, b & c so that Lim
x 0 2
x. sin x
Ans. a = c = 1, b = 2

ae x – b cos x  ce – x
lim 2
2  sin x 
x 0
Sol. x  
 x 

0
 at x  0 denominater is 0, so it should be form
0
 a – b + c = 0  a + c = b .......(1)
by L’ Hospital Rule 

a  ex  b  sinx –c  e–x 0


lim 2   from
x0 2x 0

0
For from  a – c = 0 ......(2)
0
by again L’ Hospital Rule 

139
aex  bcos x  ce–x a  b c
lim  2 2
x0 2 2
a + b + c = 4 .......(3) by equation (1), (2) & (3)
a = c = 1, b = 2

1  a2  x2  a   x  
12. Lim   2 sin   sin    where a is an odd integer
x a (a  x 2 ) 2
2  ax  2   2 

2 a 2  4
Ans.
16a 4

1  a2  x2  a   x  
Sol. lim 2 2 2  – 2sin    sin  
x  a (a  x )
 ax  2  2 

1  (a – x)2  2ax   
 lim 2 2 2   cos  a  x  – cos  a – x  
x  a (a  x ) ax 2 2
 

1  (a – x)2   
 lim 2 2 2   1  cos  a  x   1– cos  a – x  
x  a (a  x )
 ax 2 2 

 
2 cos2 (a  x) 2sin 2 (a– x)
1 4 4
 lim  
x  a (a  x) 2  ax (a 2 – x 2 )2 (a 2 – x 2 )2

   a (a  x) 
 2
4sin (a– x)  cos2 (a  x)  sin 2  –
4 4 

1 4  2
  lim  
4a 4 x a (a 2 – x 2 )2  2  
 sin  a  x 
 4 


4sin 2 (a– x)
1 4 2
  lim 
4a 4 x a 2 16
(a– x) 2  (a  x) 2
16

1 4  1  4  a 2 2
 lim    
x  a 4a 4 16(a  x) 2 4a 4 16 a 2 16a 4

2 2
Lim tan x  x
13. x 0
x 2 tan 2 x

140
2
Ans.
3

tan 2 x – x 2 tan 2 x – x 2 tan 2 x – x 2


Sol. lim 2  lim 2
 lim
x  0 x  tan 2 x x4
4 tan x
x 0 x 0
x 
x2

lim
 tan x  x  tan x  x 
x 0 x4

 x3  x3 
 x   ...  x  x   ...  x 
3 3
lim  
4

x 0 x

1 4 x2 
x   ...  2   ... 
3  3 
lim 4
x 0 x

2

3

(1  x )(1  x 2 )(1  x 3 )......(1  x 2n )


14. If L = Lim then show that L can be equal to
x 1 [(1  x )(1  x 2 )(1  x 3 ).........(1  x n )]2
n
nr 1 n
(a)  r (b)  ( 4r  2)
n! r 1
r 1

(c) The sum of the coefficients of two middle terms in the expansion of (1 + x)2n – 1.
(d) The coefficient of xn in the expansion of (1 + x)2n.
Ans. (a,b,d)
Sol. divide by each term of numerator & denominator by (1– x)

1  x n 1 1  x n  2 1  x n  3 (1  x 2n )
. . ......
1 x 1 x 1 x (1  x)  by  
 lim  
x 1 1 x 1 x 1 x2 3
(1  x n )  1– xn 
. . ....... lim  n
1 x 1 x 1 x (1  x)  x 1 1 – x 

n
n  r n 1 n  2 n  3 nn
(a) r 1 r

1

2

3
.....
n

141

 n  1 n  2  .... 2n   n! n  1 n  2  ...  2n 
1 2  3.....  n  n! n!

2n! 2n
  Cn
n! n!

1 n
(b)  (4r  2)
n! r 1

1
  4  2 8  2 12  2  ...  4n  2 
n!

1 2n
 2  6  10...  4n  2   1  3  5...  2n  1 
n! n! 

2n
 1 2  3...n 1 3 5... 2n 1
n!n!

 2  4  6...(2n) 1 3  5...


   2n  1
n!n!

 2n !

n!n!

2n
 Cn

(c) 2n–1Cn + 2n–1Cn + 1 = 2nCn + 1


(d) 2nC
n

15. Lim [1.x ]  [ 2 .x ]  [ 3.x ]  .....  [ n .x ] , Where [.] denotes the greatest integer function.
n 
n2

x
Ans.
2

[1  x]  [2  x]  ......  [n  x]
Sol. k  lim
n n2
 x  [x] > x – 1

142
x  2x  .....  nx x  2x  .......  nx – n
lim 2
 k  lim
n  n n  n2

x  n(n  1) x  n(n  1) n
lim 2
 k  lim – 2
n  2n n  2n 2 n

x x
k 
2 2

x
 by sandwhich theorem  k 
2

1  x  ln x
16. Evaluate, Lim
x 1 1  cos x
1
Ans. 
2

1xlnx  0
Sol. Lim ,  
x1 1cosx
0

1
1 
x 0
lim ,  
x 1  sin  x 
0

 1 
 2  1
x 
lim 2  2
x 1  cos  x  

  ay   by  
 exp x ln(1  )  exp x1n(1  ) 
 x   x 
17. Lim  Limit
y0  x y 
 
 
Ans. a–b

 ay   by 
 x n 1 
 x
x n 1  
 x 
 e – e 
Sol. lim  lim 
y 0 x  y
 

x x
  ay   by  
 1   – 1   
 x   x 
lim  lim  {1 }
 y 0  x 

y 


 For 1 lim f (x)g(x )  lim e g(x )[f (x ) –1]
x a x a 
 

143
 ay   by 
 x 1 –1
 x 
x 1 –1 
 x 
 e – e 
lim  lim 
 y 0 x  y
 

eay – e by (e (a –b)y – 1)
 lim  lim e by
(a – b)  a – b
y 0 y y 0 y (a – b)


18 Let x0 = 2 cos and xn = 2  x n 1 , n = 1, 2, 3, .........., find Lim 2 (n 1) · 2  x n .
6 n


Ans.
3
Sol. Let  = /6  x0 = 2cos  & x n  2  x n 1

 x1  2  x 0  2  2 cos  = 2cos/2

x 2  2  x1  2  2 cos  / 2 = 2cos/22

x 3  2  x 2  2  2 cos  / 4 = 2cos/23


x n  2 cos
2n

 lim 2 n 1 2  x n = lim 2 n 1  2 sin   n 1 


n  n   2 

sin   n 1 
lim 2   2    2  
= n    = Ans.
 n 1  3
 2 

Lim  n (1  x)  1 
1 x
 
19. x 0
 x2 x

1
Ans.
2

144
 ln (1  x)1 x 1  (1  x) ln (1  x) – x
Sol. lim  2
–   lim
x 0
 x x  x 0 x2

 x 2 x3 
(1  x)  x –  ......  – x
 2 3 
 lim 2
x 0 x

 x 2 x3   2 x3 x 4  
  x    ...    x    ...   x 
2 3 2 3
lim    
2
 
 x 0  x 
 
 

x2 x3
 ... 1
 lim 2 6 
x 0 x2 2


 4 
  n3 1  
(1  n 1 ) 2
20. Let L =  1  2  ; M =   n 3  1  and N =  1  2n 1
, then find the value of
n 3  n  n 2   n 1

L –1 + M–1 + N–1.
Ans. 8


 4  (n – 2) (n  2)
Sol. L   1 – 2   
n 3  n  n 3 n2

(3 – 2) (3  2) (4 – 2) (4  2) (5  2) (5  2) (6 – 2) (6  2)
 L 
33 44 5 5 66

1 5 2 6 3 7 1 2 1
 . . . . . ......  
3 3 4 4 5 5 3 4 6

n 3 –1

M 3
n 2 n  1

 n 2 – n  1 

(n – 1) (n 2  n  1)  2 
   n – 2n  1  n 
2
n  2 (n  1) (n – n  1)  (n –1) 2  (n–1)  1
 

145
(2 – 1) (22  2  1) (3 –1) (32  3  1) (4 –1) (42  4  1)....... 1 2 2
 M  
(2  1) (22 – 2  1) (3  1) (32 – 3  1) (4  1) (4 2 – 4  1)...... 3 3


(1  n –1 )2 
(1  n)2 22  32  42......
 N     2
n 1 1  2n –1 n 1 n (n  2) 1  3  2  4  3  5.......

1 1 1  1
    6  8
L M N 2 2


21. A circular arc of radius 1 subtends an angle of x radians, 0 < x < as shown in the figure. The point C
2
is the intersection of the two tangent lines at A & B. Let T(x) be the area of triangle ABC & let S(x) be
the area of the shaded region. Compute:
T (x )
(a) T(x) (b) S(x) & (c) the limit of as x  0.
S(x )
1 x x sin x 1 1 3
Ans. T(x)  tan 2  sin x or tan  , S(x)  x  sin x, limit 
2 2 2 2 2 2 2
Sol. Let BC = AC = a, AB = b
1
(a) T  x    AB  CM
2

1
  2  BM  CM
2

  x x
BM  a sin  90  2   a cos 2
1   cos x    

 a 2  2 sin  
2  2 2  CM  a cos  90  x   a sin x

  2  2

1 x
 tan 2  sin x
2 2

x x cos x  x a
 tan 2  sin  tan   a
2 2 2  2 1

x x
T  x   tan  sin 2
2 2

(b) S(x) = ar(sector) – ar(AOB)

146
r2 1
  x   r2 sin x ; r  1
2 2

x 1
  sin x
2 2

x x
2 tan  sin 2
(C) T(x) 2 2
lim  lim
x  0 S(x) x 0 x – sin x
x x
2  tan sin 2 2
x
2  2 x
x 2 x2 4
2 4  sin x tan x 
 lim  lim  lim  1
x 0 x – sin x  x 0 x x0 x 

x3 3x 2 6x 3
 lim  lim  lim  {By L Hospital Rule}
x  0 4(x– sinx) x 0 4(1 – cos x) x  0 4(sin x) 2

n
x cot x
Q.22 Let f (x) = Lim 3 n1
sin3  n  and g (x) = x – 4 f (x). Evaluate Lim 1  g( x )  .
n
n1 3  x 0

Ans. g(x) = sinx and l=e

Sol.  sin3x = 3sinx – 4sin3x

3 sin x  sin 3x
 sin 3 x 
4

 x   x 
n
x n  3sin  n   sin  n 1  
  3n 1 sin 3  n    3n 1  3   3 
n 1  3  n 1  4 
 

1 n  n  x  n 1  x  
  3 .sin  3n   3 .sin  3n1  
4 n 1 

1 x  
  3n.sin  n   sin x 
4 3  

1 n x 
 f (x) = lim  3 sin n  sin x 
n  4  3 

147
1  x sin
x 
3n
lim
= n  4  x  sin x 

 3n 

1
f (x) = x  sin x 
4

 g(x) = x–4f (x) = x – x + sinx

 g(x) = sinx

cot x cot x
 lim 1  g  x  
x 0
 lim 1  sin x 
x 0
1 

lim sin x cot x


 e x 0  e Ans.

n
 
Q.23 I f f (n,)=  1  tan 2 2r  , then compute Lim
n 
f (n , )
r 1


Ans.
tan 

n
 2  
Sol. f (n,) = 1  tan r 

r 1 2 

 2   2   2    2  
= 1  tan 1  tan 2 1  tan 3  ……… 1  tan n 
 2  2  2   2 

2 tan  2 2 tan  22 2 tan  23 2 tan  n


2
= tan  . . ………

tan 2 tan 2  
tan n 1
2 2

 2 tan  
use tan 2  
 1  tan 2  

=
2 n. tan  

2n
tan 

lim f (n, ) = lim


. tan  

2n
 n  n  tan   tan 

2n

148
cos 2x  (1  3x )1 3 3 4 cos3 x  ln (1  x ) 4

Q.24 L = Lim 2 4
x 0 x
If L = a b where 'a' and 'b' are relatively primes find (a + b).
Ans. 19

cos 2 x  1  3x 1 / 3
Sol. Consider,
2
 2 x 2 2 x 4   1  1  1 
  1 x  3 3 2
1    3 x 

 2! 4!   2! 
 
2

1
 2  x  3x 2 2
 
 2 
 

1
3
 4 cos x  4 ln 1  x  3
Consider,  

 4 

1
1
 2 3 3 
  x   x 2    3x 2 x 2  3
=  1   x   1   x 
 2!   2   
 2 2 
 

1

= 1 x  x 2 
3

1
1
 2  x  3x 2  2

 2
  1 x  x2

 
3
 0
L  lim  
 
x 0 x 0

 1  x  3x 2    1 2 
1      1   x  x  
2  2   3 
 lim 
x0 x

149
x 3x 2 x x 2
  
 lim 4 4 3 3  11 7  a
x0 x 4 3 12 b

 a +b = 19 Ans.

x2
 cosh (  x )  et  et
Q.25 Lim   where cosh t =
x    cos (  x ) 
2

2
Ans. e


Sol. Put t
x

2
x2
 cosh  / x    cos ht  t 2
 lim    lim  
x    cos(  / x )  t  0 cos t 

2 t 2 t3
 e t  e t  t 2 et  1  t   
= lim   ; 2! 3!
t 0 2 cos t  t 2 t3
  e t  1 t       
2! 3!

2
 2 4  t2
1  t  t 
lim  2! 4!  (l  )
= t  0 cos t 
 
 

 t2 t4 
 1   2
 
lim 2! 4!  1  2
= t 0  cos t  t
 
e  

 
 2 t 
 2 sin 2  2
lim  2    
1 t
= t 0   t 2  2! 4!  cos t
 4   
4
e    

1 1 2
   2 Ans.
e 2 2   e

150
f (x) x (1  a cos x )  b sin x
Q.26 f (x) is the function such that Lim  1 . If Lim  1 , then find the value of
x 0 x x 0 f ( x) 3
a and b.
Ans. a = –5/2, b = –3/2

x 1  a cos x   b sin x
Sol. Lim 1
x 0 f x 3
  x 2   3
x1  a 1     b x  x 
  2    3! 
 
Lim 1
 x  0  f x  
3
3
  x
 x 

3
x3
a  b  1x  a x b  f x  
 Lim 2! 3!  1 Lim  1
 x 0 x 
x 0 x3

 a  b  1  b a 
 lim  2
     1
x0
 x  6 2 

b a
 a – b +1 = 0 &  1
6 2

 On solving , a   5 2 , b   3 2 Ans.

Q.27 Through a point A on a circle, a chord AP is drawn & on the tangent at A a point T is taken such that
AT = AP. If TP produced meet the diameter through A at Q, prove that the limiting value of AQ when P
moves upto A is double the diameter of the circle.
Sol. Let radius of circle be r
In APQ,

–2
2   2 A

T

O

–
P
S

Q 
2 

151
AQ AP

sin   Q  sin  / 2  Q 

AQ 
AP sin 
 ………..(1)
cos 

In APS,

   AP
cos 2   
 2  2r

A
2   2
r
O
r
P

 AP = 2r sin 2

 From (1)

2r sin 2. sin 


AQ 
cos 

 4r sin2

Now, as P  A,  –2 0


 
2

lim  AQ   4r  2  Diameter
 

2

Hence proved

Q.28 Using Sandwich theorem, evaluate

 1 1 1 1 
(a) Lim     ...........  
n  2 2 2 2 
 n n 1 n 2 n  2n 

152
1 2 n
(b) Lim 2 + 2 + ......... +
n  1 n 2n n  n2
Ans. (a) 2, (b) 1/2
1 1 1
 
Sol. 
n 2  2n n2 n2

1 1 1
 
n 2  2n n2 1 n2

1 1 1
 
n 2  2n n2  2 n2

1 1 1
 
2 2
n  2n n  2n n2

Add all and take limt n  

2n  1 1 1 1 2n  1
 nLim  Lim   .......   Lim
 n 2  2n n n2 n2 1 n 2  2n n  n2

1 1 1
2  Lim   ........  2
n  n2 n2 1 n 2  2n

 By sandwitch theorm

1 1 1
Lim   ........  2
n  n2 n2 1 n 2  2n

1 2 n
(b) Lim 2
 2
 ........ 
n  1  n 2n n  n2

1 1 1
 2
 2

nn 1 n 1 n2

2 2 2
2
 2

nn 2n 1 n2

153
3 3 3
2
 2

nn 3 n 1 n2

n n n
2
 2

nn nn 1 n2

Add all and take limt n

n  n  1 1 n n  n  1
 Lim  Lim  .........   Lim
2n  n   
2 n  1  n 2 2 n  2 1  n 2
n  nn

1 1 n 1
  Lim 2
 .........  2

2 n  1  n nn 2
 By Sanwitch theorem,

1 2 n 1
Lim 2
 2
.........  2
 Ans.
n  1 n 2n nn 2

Q.29 Find a & b if :


 2 
(i) Lim  x  1  ax  b  = 0
x  x 1
 
Ans. a = 1, b = –1

x 2  1  ax  b x  1
Sol. Lim 0
x  x 1

1  a x 2  a  b x  1  b   0
 Lim
x  x 1

 1– a = 0 & –(a + b) = 0

a=1 & b = –1

(ii) Lim  x 2  x  1  ax  b  0
x  
 

Ans. a = –1, b = 1/2

Sol. Rationalise

154
 x 2  x  1  ax  b 
Lim  x 2  x  1  ax  b    0
x  
   2 
 x  x  1  ax  b 

x 2

 x  1  ax  b 2
0
 xLim
 2
x  x  1  ax  b

Lim
1  a x 2 2

 1  2ab x  1  b 2 0
 x 
x 2  x  1  ax  b

 1  2ab 
 1 – a2 = 0 & 0
1 a

1
 a=1 & b Ans.
2

 a  1

 
Q.30 If L = Lim 
1

1  then find the value of L  153 .
x 0  ln (1  x )
 ln ( x  1  x 2 )  L

Ans. 307

 
 1 1 
Sol. L  Lim    …………(1)
x 0  ln 1  x   x  1  x 2  
 ln 
  

Put x = –x {  x  0, we can put x = –x}

 
 1 l 
L  Lim   
x 0  ln 1  x    …………(2)
 ln  1  x  x  
2
  

(1) + (2)

 
 1 l 1 1 
 2L  Lim     
x  0 ln 1  x  ln 1  x   2  
 ln  x  1  x  ln  1  x  x  
2
    

155
 2L  Lim

ln 1  x 2  

ln 1  x 2  x 2 
x 0 ln 1  x . ln 1  x 
ln  x  1  x 2 . ln  1  x 2  x 
   

= Lim

ln 1  x 2  0
x 0 ln 1  x . ln 1  x 


ln 1  x 2
  x2
 
2
2L  Lim x
 x 0 ln 1  x   ln 1  x    x 2
 
x x

1
 2L = 1  L
2

1
L  153
 2  153  307
 L 1 Ans.
2

156
Maths IIT-JEE ‘Best Approach’ (MC SIR) LIMIT
EXERCISE # 3

1. Lim x tan 2x  2x tan x is : [ JEE '99, 2 (out of 200) ]


x 0 2
(1  cos 2x)
1 1
(A) 2 (B)  2 (C) (D) 
2 2
Ans. (C)

x  2 tan x
– 2x tan x
Sol. x tan 2x – 2x tan x 1 – tan 2
x
lim  lim
x 0 (1 – cos 2x) 2 x 0 (2sin 2 x) 2

 1 
2x tan x  2
–1 2
lim 1 – tan x   lim 2x tan x tan x
 x 0 4 sin 4 x x  0 (1 – tan 2 x) 4  sin 4 x

x sin 3 x x 1
 lim  lim
x 0 2cos3 x(1– tan 2 x) sin 4 x x 0 sin x 2cos3 x(1– tan 2 x)

1

2

x
 x  3
2 For x  R , Lim
x    = [ JEE 2000, Screening]
 x  2
(A) e (B) e 1 (C) e 5 (D) e5
Ans. (C)
x
 x –3
Sol. lim   {1 }
x  x  2
 

 x –3   x –3– x –2 
lim x  –1 lim x  
x  x2 
  e x  x2   e
–5
–5  lim
 lim x  
x
1
2
e x  x  2 
e x
 e –5

sin(  cos 2 x )
3. Lim equals [ JEE 2001, Screening]
x 0 x2

(A) – (B)  (C) (D) 1
2

Get 10% Instant Discount On Unacademy Plus [Use Referral Code: MCSIR] 157
Maths IIT-JEE ‘Best Approach’ (MC SIR) LIMIT

Ans. (B)

sin ( cos2 x)
Sol. lim
x 0 x2

– cos ( cos2 x)    2cos x sin x


 lim {by L Hospital Rule}
x 0 2x

sin x
 lim– cos (  cos 2 x)   cos x  – (–1)  (1)  (1)  
x 0 x

(cos x  1)(cos x  e x )
4. The integer n for which Lim is a finite non-zero number is
x 0 xn
(A) 1 (B) 2 (C) 3 (D) 4
[JEE 2002 (screening), 3]
Ans. (C)

(cos x –1) (cos x – e x ) (1– cos x) (e x – cos x)


Sol. lim  lim
x 0 xn x 0 x2 x n –2

 x 2 x3   x2 x 4 
x  1  x    ......  –  1–   .......  
1  e – cos x  1 2! 3! 2! 4!
 lim  n –2   lim    
n –2

x 0 2
 x  x  0 2  x 
 
 
3
 2 x 
x  x  ........ 
1 3!
 lim  n–2 
x0 2 x
 
 
 

So power of x for least degree in numerator should be equal to n – 2 inorder to


limit should be finite non-zero number
 n–2=1n=3

sin( n x )[(a  n )n x  tan x ]


5. If Lim  0 (n > 0) then the value of 'a' is equal to
x 0 x2
1 n2 1
(A) (B) n2 + 1 (C) (D) None
n n
[JEE 2003 (screening)]
Ans. (C)
[(a – n)nx – tan x]
Sol. limsin (nx) 0
x 0 x2

Get 10% Instant Discount On Unacademy Plus [Use Referral Code: MCSIR] 158
Maths IIT-JEE ‘Best Approach’ (MC SIR) LIMIT

sin nx  (a – n) nx– tan x 


 lim n 0
x0 nx x
 

 anx – n 2 x – tan x 
 lim n 
x 0 x  0 {by expension of tan x}
 

 2 x3 
 anx – n x – x – ...... 
3
lim n  0
 x 0
 x 
 

n2 1
 2
n (an – n –1)  0  a
n

6. If lim (x–3 sin 3x + ax–2 + b) exists and is equal to zero then :


x 0
(A) a = –3 & b = 9/2 (B) a = 3 & b = 9/2 (C) a = –3 & b = –9/2 (D) a = 3 & b = –9/2
Ans. (A)
sin 3x a
Sol. lim  2 b0
x 0 x3 x

sin 3x  ax
 lim  –b
x 0 x3

 (3x ) 3 
 (3x) –  .......  ax
 3!
lim    –b
x 0 x3
For existence of limit 3 + a = 0  a = – 3

 –9x 3 
  .......  –9 9
 lim  2 3   – b   –b  b 
x 0
 x  2 2
 

1 1
 xn  ex  xn  ex
2   3 
 The value of lim  
n
  (where n  N) is
x  x

2 2
(A) ln   (B) 0 (C) n ln   (D) not defined
3 3
Ans. (B)

Get 10% Instant Discount On Unacademy Plus [Use Referral Code: MCSIR] 159
Maths IIT-JEE ‘Best Approach’ (MC SIR) LIMIT

n
/e x ) n x
2( x  3( x /e ) 1 xn
Sol. lim  x Put x  t
x  (x n / e x ) e e
 As x  , t  0
 2(x n /ex )  1  3( x n /ex )  1  1
 lim  n x    n x    x
x 
 x / e   x / e   e
= (lim 2 – lim 3) × 0 = 0
 xn 
 lim
x  e x
 0
 

x
4  x 2 cos
8. lim 4 is
x 2 sin(x  2)

1
(A) equal to  (B) equal to  (C) equal to   (D) non existent

Ans. (C)

x
4 – x 2 cos
Sol. lim 4
x2 sin (x – 2)


4 – (2 – h)2 cos (2 – h)
x=2–h&h0  lim 4
h 0 sin(2 – h – 2)

  
(h 2 – 4h) sin h h (h  4) sin h  h
lim 2  lim 4 4
 h 0 – sin h h  0 sin h
– h
4


 h 2 (h  4)

 lim 4  2  – 
h 0 –h 4

1  cos(ax 2  bx  c)
9. Let  are the roots of the quadratic equation ax2 + bx + c = 0 then lim equals
x  (x  ) 2

1 a2 a2
(A) 0 (B) ( – )2 (C) ( – )2 (D)  ( – )2
2 2 2

Get 10% Instant Discount On Unacademy Plus [Use Referral Code: MCSIR] 160
Maths IIT-JEE ‘Best Approach’ (MC SIR) LIMIT
Ans. (C)

Sol. ax2 + bx + c  0   ax2 + bx + c = a (x – ) (x – ) ......(1)


1– cos (ax 2  bx  c) 1– cos (ax 2  bx  c) (ax 2  bx  c)


 lim  lim 
x  (x – )2 x  (ax 2  bx  c)2 (x – )2

1 a 2 (x – ) 2 (x – )2 a 2 ( – ) 2
 lim  
x  2 (x – )2 2

10. ABC is an isosceles triangle inscribed in a circle of radius r. If AB = AC & h is the altitude fromA to BC

and P be the perimeter of ABC then lim equals (where  is the area of the triangle)
h 0 P3

1 1 1
(A) (B) (C) (D) none
32r 64r 128r
Ans. (C)

Sol. BD  r 2  (h  r) 2

 2hr  h 2

A
AD = h
r
O
r r
 AB  h 2  (2hr  h 2 ) h–r
B C
D

 2hr
1
   BC  h
2
1
  2 2hr  h 2  h
2

 h 3/2 2r  h h 3/ 2 2r  h
 lim  lim  lim 3
h  0 p3 h0
(2 2hr  2 2hr  h 2 )3 h 0 h 3/ 2  2 2r  2 2r  h 
 

2r 1
 3

2 2r  2 2r  128r

Get 10% Instant Discount On Unacademy Plus [Use Referral Code: MCSIR] 161
Maths IIT-JEE ‘Best Approach’ (MC SIR) LIMIT
 
 (1  cos x )  (1  cos x )  (1  cos x )  .........    1
11. Lim  2

equals
x 0 x
1
(A) 0 (B) (C) 1 (D) 2
2
Ans. (B)

 
 (1  cos x )  (1  cos x )  (1  cos x )  .........    1
Sol. Lim  2

x 0 x

1 1 1
 2
 .....
23
(1  cos x) 2 2 1
Lim
x 0 x2

(1  cos x)  1
lim
x 0 x2

 cos x
 lim
x 0 x2

sin x
 lim
x 0 2x

cos x 1
 lim 
x 0 2 2

ln(3  x)  ln(3  x)
12. If lim = k, the value of k is
x 0 x
2 1 2
(A) (B)  (C)  (D) 0
3 3 3
Ans. (A)
ln (3  x) – ln(3 – x)
Sol. lim k
x 0 x

 3 x   2x 
ln   ln  1  
3– x 
 lim  k  lim  3– x 
x 0 x x 0 x

 2x 
ln 1  
3– x  2x
 lim   
2
k
x 0 2x (3 – x)  x 3
3– x

Get 10% Instant Discount On Unacademy Plus [Use Referral Code: MCSIR] 162
Maths IIT-JEE ‘Best Approach’ (MC SIR) LIMIT

x 2n  1
13. The function f (x) = Lim 2 n is identical with the function
n  x 1
(A) g (x) = sgn(x – 1) (B) h (x) = sgn (tan–1x)
(C) u (x) = sgn( | x | – 1) (D) v (x) = sgn (cot–1x)
Ans. (C)

 x 2n  1
Sol. f (x)  lim  2n 
n   x  1

 11
 0; x2  1
 1  1
 0 1
f (x)    1 ; x2  1
 0  1
 x 2n  1    2
n  x 2n  1    ; x  1
lim

0; x  1

f (x)  1 ; 1  x  1  5gm ( x  1)
 1 ; x  1 or x  1

14. Which one of the following statement is true?


(A) If Lim f ( x ) ·g( x ) and Lim f ( x ) exist, then Lim g ( x ) exists.
xc xc xc

(B) If Lim f ( x ) ·g( x ) exists, then Lim f ( x ) and Lim g ( x ) exist.


xc xc xc

(C) If Lim f (x )  g ( x )  and Lim f ( x ) exist, then Lim g ( x ) exist.


x c xc xc

(D) If Lim f (x )  g ( x )  exists, then Lim f ( x ) and Lim g ( x ) exist.


x c xc xc
Ans. (C)
Sol. (A) let f(x) = x & g(x) = cosec x
Now, lim f (x)  g(x) exist  1
x 0

also lim f (x)  0 (exist)


x 0

but lim g(x) does not exist


x 0

 Option (A) is False


1 ; x  0
(B) let f (x)   and g(x) = 0
2 ; x  0
then f(x) · g(x) = 0 and lim f (x )  g (x ) exist,
x 0

while lim f (x) does not exist


x 0

Get 10% Instant Discount On Unacademy Plus [Use Referral Code: MCSIR] 163
Maths IIT-JEE ‘Best Approach’ (MC SIR) LIMIT
 Option (B) is False
(C)  g(x) = f(x) + g(x) – f(x)
 By theorem,
lim g(x)  lim  f (x)  g(x)  lim f (x)
x C xC x C

 Both lim
x C
 f (x)  g(x) & lim
xC
f (x) both exist

 lim g(x) also exist


xC

 Option (C) is True


(D) let f(x) = [x] & g(x) = {x}
 lim f (x)  g(x)  lim x  0 (exist)
x0 x0

but lim
x 0
g(x) & lim f (x) both does not exist
x0

 Option (D) is False

xf (3)  3f (x)
15. If f(3) = 6 & f '(3) = 2, then xlim is given by
3 x 3
(A) 6 (B) 4 (C) 0 (D) none of these
Ans. (C)
Sol. f (3)  6 & f (3) = 2

x f (3) – 3f (x) (3  h)f (3) – 3f (3  h)


lim  lim
x 3 x –3 h 0 3 h – 3

–3f (3  h)  3f (3) hf (3)


 lim 
x 0 h h

 lim – 3f (3)  f (3)  – 6 +6 = 0


x 0

16. If Lim [ f ( x )  g ( x )]  2 and Lim [ f ( x )  g ( x )]  1 , then Lim f (x ) g ( x )


x a x a x a

3 3 4
(A) need not exist (B) exist and is (C) exists and is – (D) exists and is
4 4 3
Ans. (B)

Sol. lim f (x)  lim g(x)  2


xa x a

lim f (x) – lim g(x)  1


x a xa

3
add 2 lim f (3)  3  lim f (x) 
xa x a 2

3 1
lim g(x)  2 – 
x a 2 2

Get 10% Instant Discount On Unacademy Plus [Use Referral Code: MCSIR] 164
Maths IIT-JEE ‘Best Approach’ (MC SIR) LIMIT
3 1 3
 lim f (x)  g(x)   
x a 2 2 4

17. Given 0 < a < b, the value of lim (an + bn)1/n is


n 

(A) a (B) b (C) 1/a (D) 1/b


Ans. (B)
1  
 
 a  b
n
1  a   n

Sol. lim (a n  b n ) n  lim b    1  
n   b  a
 
n 
  1 
 b 
 b (1 + 0)0  a
n

 
 b  lim    0 
n  b
   

12 n 2 2 (n1)32 (n2).....n 2.1


18. Lim is equal to :
n 13 23 33 ......n 3
1 2 1 1
(A) (B) (C) (D)
3 3 2 6
Ans. (A)

n n
2 2
r (n  r  1)  (n  1)r  r3
r 1 r 1
lim n  lim n
Sol. n 
3
n 
3
r r 1
r
r 1

2
n(n  1)(n  2)  n(n  1) 
(n  1)  
6  2 
 lim 2
n 
 n(n  1) 
 
2 

2n  1 n

 lim 3 2    1
n  
n   3
2

Limit

cot  1 x  a log a x 
19. The value of (a > 1) is equal to
x 
sec 1 a x
log x a
(A) 1 (B) 0 (C) /2 (D) does not exist
Ans. (A)

Get 10% Instant Discount On Unacademy Plus [Use Referral Code: MCSIR] 165
Maths IIT-JEE ‘Best Approach’ (MC SIR) LIMIT

 log x 
cot 1  aa 
 x 
Sol. lim ....... (1)
x   ax 
sec 1 
 log a x 

Now,
log a x   
lim  
x  xa   
Apply LH Rule
1/ x ln a 1
 lim n 1
 lim
x  a x x  a ln a  x n

=0
and
ax  
lim  
x  log x   
a

a x ln a 2
 lim  lim a x  x  ln a   
x  1 x 

x ln a

 log x 
cot 1  aa 
 x  cot 1 (0)
lim  1
 form (1), x 
1  a x  sec ()
sec 
 log a x 

/2
 1
/2

20. Let f : (1, 2)  R satisfie the inequality


cos(2x  4)  33 x 2 | 4x  8 |
< f(x) <  x  (1, 2). Then lim f(x) is equal to
2 x 2 x  2
(A) 16
(B) –16
(C) cannot be determined from the given information
(D) does not exists
Ans. (B)

cos(2x  4)  33 x 2 4x  8
Sol.  f (x)  x  (1, 2)
2 (x  2)

cos(2x  4)  33 x 2 4x  8
lim  lim f (x)  lim
x  2 2 x 2 x 2 x2

Get 10% Instant Discount On Unacademy Plus [Use Referral Code: MCSIR] 166
Maths IIT-JEE ‘Best Approach’ (MC SIR) LIMIT
cos  2x  4   33  4x  8 
lim  lim f (x)  lim (4x 2 )  
x 2 2 x 2 x 2
 4 x  2 
 4(x  2) 
 
1  33 for x  2
 lim f (x)  16
2 x2

 lim f (x)  16 {by sand which rule}


x 2

sin x cos x n
21. Let a = min [x2 + 2x + 3, x  R] and b = Lim
x 0 e x  e x
. Then the value of  a r b n r is
r 0

2 n 1  1 2n 1  1 2n  1 4n 1  1
(A) (B) (C) (D)
3 ·2 n 3 ·2 n 3 ·2 n 3 ·2 n
Ans. (D)
Sol. a  min[x 2  2x  3], x  R
 min [(x + 1)2 + 2], x  R
2
sin x cos x
b  lim
x0 ex  e x
sin 2x
 lim

x 0 2 e x  e  x

2  cos 2x
 lim {by L’Hospital Rule}

x 0 2 e x  e  x

1 1
 
11 2

n n
1
  a r  bnr  2 r

r0 r0 2nr
n n
1
  22r  n  4 r

r 0 2n r 0

1 0 1 2 n 1
 n  4  4  4  ....  4   n 1
 4n 1  1 4 n  1  1

2 2 4 1 3  2n

    1    
22. Given l1 = Lim cos 1 sec x   ; l2 = Lim sin cosec x    ;
x  4   4  x  4   4 

Get 10% Instant Discount On Unacademy Plus [Use Referral Code: MCSIR] 167
Maths IIT-JEE ‘Best Approach’ (MC SIR) LIMIT

1     1    
l3 = Lim tan cot  x   ; l4 = Lim cot  tan  x  
x  4   4  x  4   4 
where [x] denotes greatest integer function then which of the following limits exist?
(A) l1 and l2 only (B) l1 and l3 only (C) l1 and l4 only (D) All of them
Ans. (A)

   
Sol. 1  RHL  lim cos 1 sec  x     lim cos 1 sec 0  cos 1 1  0
x

  4  x  
4 4

   
LHL  lim cos 1 sec  x     lim cos 1 sec 0   cos 1 1  0
x

  4  x 
4 4

 1 exist

     
 2  RHL  lim sin 1  cos ec  x     lim sin 1 cos ec 
x

  4  x    2
4 4


 sin 1 
2

  
LHL  lim sin 1  cos ec  
x
  2 2
4

 2 exist

   
 3  LHL  lim tan 1  cot    tan 1  0  0
x

4
  4 


   
RHL  lim tan 1 cot   tan 1 (1) 
x

 2  4
4

limit does not exist


 4  LHL  lim cot 1  tan  0    cot 1  1  
x
 4
4


RHL  lim cot 1  tan 0   cot 1 (0) 
x
 2
4

limit does not exist

Get 10% Instant Discount On Unacademy Plus [Use Referral Code: MCSIR] 168
Maths IIT-JEE ‘Best Approach’ (MC SIR) LIMIT
23. Suppose that a and b (b  a) are real positive numbers then the value of
1/ t
 b t 1  a t 1 
Lim  
 has the value equals to
t 0
 b  a 
1
a ln b  b ln a b l n b  a ln a  b b  ba
(A) (B) (C) b ln b – a ln a (D)  
ba ba  aa 
 
Ans. (D)
1/ t
 b t 1  a t 1 
Sol. lim   1 
form
t0
 ba 
1  b t 1  a t 1 
lim  1
 b a
t 0 t 

e
1  bt 1  b a t 1  a 
lim   
t 0 b  a 
 t t 
e
1  t 1
lim b nb  a t 1na 
t 0 b  a  
e
1
 bnb  ana 
b a
e
b a
nb ba  na ba
e
1
 bb  (ba)
 a 
a 

x 1 
24. If xlim x.n 0 1/ x  = –5, where  are finite real numbers then

1 0 1/ x

(A)  = 2,  = 1,  R (B)  = 2,  = 2, = 5


(C)  R,  = 1,  R (D)   R,  = 1, = 5
Ans. (D)
 1   r 
Sol. lim x  ln   2   1      5
x  xx   x
  r
 lim x  ln    3      ln      1
x   x x
  r
 lim x  ln  1  3   (  0)  5
x   x x

ln(1  t 3  rt)
 lim (0 / 0)  5
t 0 t
3t 2  r
 lim  5  r  5 &   R
t 0 (1  t 3  rt)

Get 10% Instant Discount On Unacademy Plus [Use Referral Code: MCSIR] 169
Maths IIT-JEE ‘Best Approach’ (MC SIR) LIMIT
 a  R,  = 1, r = 5

 1  1
25. Range of the function f(x) =  2  is, where [*] denotes the greatest integer function
 n(x  e)  1  x2
and e = lim (1 + )1/
0

 e 1 
(A)  0,   {2} (B) (0, 1) (C) (0, 1]  {2} (D) (0, 1)  {2}
 e 
Ans. (D)

 1  1
Sol. f (x)    
  n(x  e) 
2
1  x2

 1  1 
for f(x) to be Min  lim f (x)  lim     
   n (x  e) 
2
x  x 
1 x2 
 0 + 0 0
for Max value of f(x) 

 1  1
lim f (x)  lim    lim
x 0  n (x  e)
2
x 
  x 0 1  x 2
=0+1=1

 1  1
at x = 0 f(0) =     11  2
 n e 1 0
 Range of f(x) = (0, 1)  {2}

26. lim sin–1[tan x] = l then {l} is equla to


x 0

  
(A) 0 (B) 1 – (C) –1 (D) 2 –
2 2 2
where [] and {} denotes greatest integer and fractional part function.
Ans. (D)

Sol. l  lim– sin –1[tan x]  lim– sin –1[0 ]  lim– sin –1 (–1)
x 0 x 0 x 0


=–
2

  
 {l} =  –   2 –
 2 2

Get 10% Instant Discount On Unacademy Plus [Use Referral Code: MCSIR] 170
Maths IIT-JEE ‘Best Approach’ (MC SIR) LIMIT

27. Which one of the following limit does not tends to unity ?
sin(tan t) sin(cos x)
(A) lim (B) lim
t 0 sin t x 0 cos x

1  x  1 x x2
(C) lim (D) lim
x 0 x x 0 x
Ans. (B,D)
sin(tan t) sin(tan t) tan t tan t
Sol. (A) lim  lim   lim(1) 
t 0 sin t t  0 tan t sin t t  0 sin t

1
 lim 1
t0 cos t
sin(cos x)
(B) lim 1
x
 cos x
2

1 x  1 x
(C) lim
x 0 x
1 1

1 1
 lim 2 1  x 2 1  x    1
x  1 2 2

x2 x
(D) lim  lim
x 0 x x 0 x

x
 lim  1
x0 x
cos ecx
 3 
28. Limit   has the value equal to :
x0 
1  4  x 

(A) e 1/12 (B) e 1/6 (C) e 1/4 (D) e 1/3


Ans. (A)
cosec x
 3 
Sol. lim 
x 0 1  4  x 
1 
form
 

3 x  2– 4  x  1
  lim  
lim cosec x  –1 x0 sin x  1
4  x  x
  e
x 0 1 4 x   e 

1
0–
2 4 x
lim
2– 4  x x0  1 
lim (1 4  x )1 x  0  
x0 x (14 x )  2 4 x 
 e e {By L’Hospital Rule}

Get 10% Instant Discount On Unacademy Plus [Use Referral Code: MCSIR] 171
Maths IIT-JEE ‘Best Approach’ (MC SIR) LIMIT
–1 1

22(1 2  0)
 e e 12

29. Limit

cot 1 x  1  x  is equal to
x   2 x  1  x 
sec 1   
 x  1  

(A) 1 (B) 0 (C) /2 (D) non existent


Ans. (A)

 (x  1) – x 
–1 cot –1  
cot ( x  1 – x )  x 1  x 
Sol. lim x
 lim
x 
–1  2x –1 
x    2x  1  
sec   sec –1 e x  –1 
 x –1    x –1 

 1  
cot –1   –1
 x 1  x   cot (0)  2  1
 lim sec –1 () 
x   x(x  2) 
sec –1 e x –1 
2
 

1 x 1 x 1 1 x 
30. lim  tan  2 tan 2  3 tan  .....  n tan n  is equal to
n   2 2 2 2 2 2 2 
1 tan x tan x  x
(A) – tan x (B) (C) (D) x – cot x
x x x tan x
Ans. (C)
2 tan x / 2
Sol. Consider, tan x 
1  tan 2 x / 2
2 tan x / 2  cot x
 cot x  tan x 
1  tan 2 x / 2
2 cot x
 1  tan 2 x / 2 
cot x / 2

Get 10% Instant Discount On Unacademy Plus [Use Referral Code: MCSIR] 172
Maths IIT-JEE ‘Best Approach’ (MC SIR) LIMIT
x x
 cot  tan  2 cot x
2 2
1 x 1 x
 tan  cot  cot x
2 2 2 2
1 x 1 x
 tan  cot  cot x
2 2 2 2
1 x 1 x 1 x
2
tan 2  2 cot 2  cot
2 2 2 2 2 2
1 x 1 x 1 x
3
tan 3  3 cot 3  2 cot 2
2 2 2 2 2 2

1 x 1 x 1 x
n
tan n  n cot n  n 1 cot n 1
2 2 2 2 2 2
--------------------------------------------
Add & take lim
n 

1 x 1 x 1 x
 lim  tan  2 tan 2  ....  n tan n 
n   2 2 2 2 2 2 
1 x
 lim n
 cot n  cot x
n  2 2
x / 2n 1 1 1
 lim n
  
n  x  tan x / 2 tan x x tan x
tan x  x

x  tan x

x 2  1 , x  0, 2
sin x x  n, n  0, 1, 2, 3.... 
31. f(x) =  and g(x) =  4 , x  0 then lim f [f(x)]
 2 otherwise  5 , x  2
x 0

(A) 1 (B) 4 (C) 5 (D) non existent
Ans. (A)
Sol. let P(x) = g(f(x))
RHL  lim P(x)  lim g(sin h)
x0 h h0

2
 lim (sin h  1)  1
h0

LHL  lim P(x)  lim g(sin(  h))


x 0h h0

Get 10% Instant Discount On Unacademy Plus [Use Referral Code: MCSIR] 173
Maths IIT-JEE ‘Best Approach’ (MC SIR) LIMIT
 lim ( sin 2 h)  1
h0

 lim g(f (x))  1


x0

n(x 2  e x )
32. Let f(x) = . If lim f(x) =  and lim f(x) = m then
n(x 4  e2x ) x  x 

(A)  = m (B)  = 2m (C) 2 = m (D)  + m = 0


Ans. (A)

 n(x 2  ex )
Sol. f (x) 
 n(x 4  e2x )

 n(x 2  e x )
l  lim
x   n(x 4  e2x )

 x2 
 n e x  x  1
 lim e 
4
x  x 
 n e 2x  2x  1
e 

 x2 
 n ex   n  x  1
e 
 lim 4
x   x 
 n e2x   n 1  2x 
 e 

 x2 
x   n  x  1
e 
 xlim 4
  x 
2x   n  1  2x 
 e 
1  x2 
 n 1
1
x  e x  1  0 1  x2 
 xlim    lim  0
 1  x4  2  0 2  x  e
x
2   n 1  2x  
x  e 

Similarly

Get 10% Instant Discount On Unacademy Plus [Use Referral Code: MCSIR] 174
Maths IIT-JEE ‘Best Approach’ (MC SIR) LIMIT
1  x2 
1  n 1  x 
x  e  1
m  lim   x2 
x  1  x4  2 
 x  x  0
lim
2  n 1  2x   e 
x  e 

33. Lim cos   n 2  n  when n is an integer :


n  
(A) is equal to 1 (B) is equal to  1 (C) is equal to zero (D) does not exist
Ans. (C)

Sol. 
lim cos  n 2  n
n 

 1
 lim cos  n 1  
n 
 n
1
 
 1  2

 lim cos n  1   

n    n 
 
  1 1     
 lim cos  n  1  – 2  ....    lim cos n  –  ....
n 
  2n 8n  n 
 2 8n 

  
 lim sin  n –  ....  0
n 
 8n 

[x 2 ] [x 2 ]
34. Let lim = l & xlim = m, where [] denotes greatest integer, then :
x 0 x 0 x 2

(A) l exists but m does not (B) m exists but l does not
(C) l & m both exist (D) neither l nor m exists
Ans. (C)

[x 2 ] Exact 0
Sol.   lim  0
x 0 x Approaching to 0

[x 2 ] Exact 0
m  lim 2
 0
x 0 x Approaching to 0
 Both  & m exist.

2
 
35. lim  1  log 2 x cos x 
x 0  cos 
 2 
(A) is equal to 4 (B) is equal to 9 (C) is equal to 289 (D) is non existent

Get 10% Instant Discount On Unacademy Plus [Use Referral Code: MCSIR] 175
Maths IIT-JEE ‘Best Approach’ (MC SIR) LIMIT
Ans. (C)
2
 
Sol. lim 1  log 2 x cos x 
x 0 cos
 2 

 x2 
1 – 2!  ....
log cos x  
lim log  cos x  lim  lim x
x 0 cos x0 x x  0  x 2
 [by expansion of cos x & cos ]
2 log cos log 1 –  .... 2
2  4  2! 

  x2   x2 
log 1 –   ....   –   ....  ......
lim   2! 
 lim  2
2! 
2
 x 0  x   x  0  x  {by expansion log (1 – x)}
log 1 –   ....   –   ....  ......
  8!   8 

1

 lim 2!  4
x0 1

8

2
 
 lim  1  log  2cos x   (1  42 ) 2  289
x 0 cos
 2 

36. The value of Limit


 tan  {x }  1   sin {x }
where { x } denotes the fractional part function:
x0 {x }  {x }  1 
(A) is 1 (B) is tan 1 (C) is sin 1 (D) is non existent
Ans. (D)
tan(h  1)  sin h
Sol. RHL  lim  tan1
x0 h (h  1)  h

tan( h)  sin (1  h)
LHL  lim  sin1
x0 h (h)  (1  h)
(tan({x}  1)  sin{x}
 lim  Does not exist.
x0 {x}({x}  1)

2  2 x  sin 2x
37. Lim is :
x  (2 x  sin 2x )esin x
(A) equal to zero (B) equal to 1 (C) equal to  1 (D) non existent

Get 10% Instant Discount On Unacademy Plus [Use Referral Code: MCSIR] 176
Maths IIT-JEE ‘Best Approach’ (MC SIR) LIMIT
Ans. (D)
2  2x  sin 2x
Sol. lim
x  (2x  six2x)  esin x

2 sin 2x
 2
2
 lim x x  lim sin x
x   six2x  sin x x  2e
 2   e
x 
= Does not exist
 1 1 
 lim
x  e sin x
oscillates between to e 
 e 

| ax 2  bx  c |
38. Let  &  be the roots of the equation, ax2 + bx + c = 0 where 1 <  < , then lim
x m ax 2  bx  c

when :
(A) a > 0 & m > 1 (B) a < 0 & m < 1
|a|
(C) a < 0 &  < m <  (D) =1&m>
a
Ans. (C)
Sol. Let f(x) = ax2 + bx + c = 0
Case (i) a>0

y
ax 2  bx  c
 lim 1
xm ax 2  bx  c x
O 1  

When m  (, )  (, )


Case (ii) a<0

ax 2  bx  c x
 lim 1
xm ax 2  bx  c
O 1  

When m  (, )

2
lim
39. The value of x 0  cos ax cos ec bx is
 8b2   8a 2   a2   b2 
       
 2 
 2   2  
 a   b   2b   2a 2 
(A) e (B) e (C) e (D) e

Get 10% Instant Discount On Unacademy Plus [Use Referral Code: MCSIR] 177
Maths IIT-JEE ‘Best Approach’ (MC SIR) LIMIT
Ans. (C)
1
cos2 bx
Sol. lim  cos ax   lim  cos ax  sin2 bx {1}
x 0 x 0

(cos ax–1) (1 cos ax) b2 x 2 1


lim lim .(– a 2 x 2 ). 2 . 2 2
sin 2 bx a 2 x2
 e x0
 e x 0 sin bx b x

1 (– a 2 ) – a2
 2 b2 2
e  e 2b

40. If x is a real number in [0, 1] then the value of Limit Limit


m   n   [1 + cos
2m (n !  x)] is given by

(A) 1 or 2 according as x is rational or irrational


(B) 2 or 1 according as x is rational or irrational
(C) 1 for all x
(D) 2 for all x
Ans. (B)
Sol. Case (i) when x is rational in [0, 1]
then n ! x will be integer as n  
 cos2 (n ! x) = 1

 lim cos 2  n!x    1
m


 lim lim 1  cos 2m (n!  x)  1  1  2
m n 

Case (ii) when x is irrational in [0, 1]
then n ! x is not an integer as n  
 cos2 (n ! x ) < 1
 (cos2 (n ! x ))m  0 as m  

 lim lim 1  cos 2m (n!x )  1  0  1
m n 

n
r
41. Lim
n 
 n2  n  r equals
r 1
(A) 0 (B) 1/3 (C) 1/2 (D) 1
Ans. (C)
1 2 n
Sol. Let f (n)  2
 2  ......  2
n  n 1 n  n  2 n nn
1 2 n
Consider, g(n)  2
 2  ......  2
n nn n nn n nn
(n  1)
1  2  .....  n n 2 n 1
g(n)  2
 
n  2n n(n  2) 2(n  2)
1 2 n
and, h(n)  2
 2  ......  2
n  n 1 n  n 1 n  n 1

Get 10% Instant Discount On Unacademy Plus [Use Referral Code: MCSIR] 178
Maths IIT-JEE ‘Best Approach’ (MC SIR) LIMIT
1  2  .....  n n(n  1) n2  n
h(n)   
n2  n  1 2(n 2  n  1) 2(n 2  n  1)
 g(n) < f(n) < h(n)
 lim g(n)  lim f (n)  lim h(n)
n  n  n 

1 1
  lim f (n) 
2 n  2
 By sandwitch theorem,
n
r 1
lim  2

n 
r 1 n nr 2

(sin x  x) 2  1  cos x 3
42. The value of the limit, lim is equal to
x 0 x 5 sin x  (1  cos x) 2 (2x 2  sin x 2 )

19 4 19
(A) (B) (C) (D) 0
45 9 25
Ans. (A)

(sin x  x)2  1  cos x 3  0


Sol. lim 5 2 2 2  
x  0 x sin x  (1  cos x) (2x  sin x )  0 

2
 x3    x 6 x12 
  x   ......  x   1   1    ......
3! 2! 4!
 lim 2
x 0
 x3   x2 x4   x6 
x  x   ......   1  1     2x 2  x 2  
5

 3!   2! 4!   3! 

x6 x 6 x12
 
(3!)2 2! 4!
 lim 2
x 0
 x2   1 x2   x4 
x 1    x 6    1  
6

 3!   2! 4!   3! 

1 1 x6
 
 lim 36 2 4!
x 0 2
 x   1 x 2   x4 
2

 1  3!    2!  4!  1  3! 

Get 10% Instant Discount On Unacademy Plus [Use Referral Code: MCSIR] 179
Maths IIT-JEE ‘Best Approach’ (MC SIR) LIMIT

1 1 19

 36 22  36  19
 1 5 45
1   1 4
 2

43. Lim  n 2  n  1   n 2  n  1   where [ ] denotes the greatest integer function is


n     
(A) 0 (B) 1/2 (C) 2/3 (D) 1/4
Ans. (B)

Sol. lim
n   n 2  n  1 –  n 2  n  1
  
 n  n2  n 1  n 1
2
 lim n  n  1 – n  
n   [ n2  n 1]  n 
1
1
n 1 n
lim  lim
 n 2
n  n 1  n n 1 1
1   2 1
n n
1

2

n
n p n q 
44. Lim   , p, q > 0 equals
n  2 
 
pq
(A) 1 (B) pq (C) pq (D)
2
Ans. (B)
n
npnq 
Sol.. lim   {1}
n   2
 

 n p n q 
  lim n  2 –1
n   
e

1  p n q n – 2    p n –1 q n –1  
1  lim   
lim n  2
 n 0  n n  
 n 0    2  
e e
1
(  n p   n q)
 e2  e n pq
 pq

Get 10% Instant Discount On Unacademy Plus [Use Referral Code: MCSIR] 180
Maths IIT-JEE ‘Best Approach’ (MC SIR) LIMIT

f (x  c) f (x  2c) f (x  3c)
g(x)
45. Let g(x) = f (c) f (2c) f (3c) , where c is constant then lim is equal to
x 0 x
f '(c) f '(2c) f '(3c)

(A) 0 (B) 1 (C) –1 (D) f(c)


Ans. (A)

f (x  c) f (x  2c) f (x  3c)
Sol. g(x)  f (c) f (2c) f (3c)
f '(c) f '(2c) f '(3c)

f '(x  c) f '(x  2c) f '(x  3c) f (x  c) f (x  2c) f (x  3c)


g '(x)  f (c) f (2 c) f (3c)  0 0 0
f '(c) f '(2c) f '(3c) f '(c) f '(2c) f '(3c)

f (x  c) f (x  2c) f (x  3c)
+ f (c) f (2 c) f (3c)
0 0 0

f '(c) f '(2c) f '(3c)


g '(0)  f (c) f (2c) f (3c)  0
{R1 & R3 are same}
f '(c) f '(2c) f '(3c)

 g(x) g '(x)
lim  lim  g '(0)  0
x 0 x x  0 1

1  2 x  n2  2x  e 2g(x)  ef (x)
46. Let f(x) = lim tan  4n  1  cos   and g(x) = lim n cos   then lim
n    n  n  2  n  x 0 x6
equals
8 7 5 2
(A) (B) (C) (D)
3 3 3 3
Ans. (A)

  x 
Sol. f (x)  lim tan 1  8n 2 sin 2   
n    2n  

 2 x 

 sin   
 2n 
 lim tan 1  2  2
 x 2

n 
  x 
  2n  

Get 10% Instant Discount On Unacademy Plus [Use Referral Code: MCSIR] 181
Maths IIT-JEE ‘Best Approach’ (MC SIR) LIMIT
f(x) = tan–1 (2x2)

  2x  
ln 1  cos    1
2
n   n    2x  
g(x)  lim   cos    1
n  2   2x     n 
 cos  n   1

n2   x
 lim   2sin 2   
n  2   n 

 x
 sin 2  
 n
 lim 2
 x 2  g(x)   x 2
n 
 x
 
n
2 1
(2x 2 )
e 2g(x )  e f (x ) e 2x  e tan
 lim  lim
x0 x6 x0 x6
1 2 2 1 2
e tan (2x )  e2x  tan (2x )  1 
 lim 6
 2 1 2 
 (2x 2  tan 1 (2x 2 ))
x 0 x  2x  tan (2x ) 

2x 2  tan 1 (2x 2 )
 lim
x0 x6

 (2x 2 )3 
2x 2   2x 2 
 3  8
 lim 
x0 x6 3

3  4sin A sin C
47. In ABC, angles A, B, C are in A.P. then lim is
AC |A C|
(A) 1 (B) 2 (C) 3 (D) not exist

Ans. (A)

3 – 4 sin A sin C  
Sol. lim Let   A a–d 
AC |A–C|  
B  a 
3 – 4sin (a – d)sin (a  d)  C  a d 
lim  
d 0 | a– d  (a  d) |  A  B  C   
 
a – d  a  d  a   
2 2
3 – 4(sin a – sin d)  3a   
 lim  
d 0 |– 2d |   
 a 
3
 
A  C Code: MCSIR]
Get 10% Instant Discount On Unacademy Plus [Use Referral  182
a – d  a  d  d  0 
 
Maths IIT-JEE ‘Best Approach’ (MC SIR) LIMIT

3
3  4   4 sin 2 d
 4 2 sin d
lim  lim 1
d 0 2d d0 2d

n(x  1)  x
48. The value of lim equal to
x 0 x((1  x)1/x  e)
(A) ee (B) e (C) 1/e (D) 1
Ans. (C)

n(x  1) – x
Sol. lim 1
x 0  
x  (1  x) x – e 
 
We know that

 x 11 
(1  x)  e 1–  x 2  ......
x

 2 24 
x2 x3
 n (1  x)  x –  – ......
2 3
n(x  1) – x
lim 1
x 0  
x  (1  x) x – e 
 

 x 2 x3  –x 2 x 3
 x –   .......  – x   ......
2 3
lim    lim 2 3
 x0  ex 11  x  0 –e 
x  e –  ex 2  ...... – e  x 2   ......
 2 24   2 

1
–  0...... 1
 2 
–e e
 ......
2

 1
49. lim  x   , where {.} denotes the fractional part function.
x 1  x
(A) is equal to 1 (B) is equal to 0 (C) does not exist (D) none of these
Ans. (A)

Get 10% Instant Discount On Unacademy Plus [Use Referral Code: MCSIR] 183
Maths IIT-JEE ‘Best Approach’ (MC SIR) LIMIT

 1 1  1
Sol. lim  x    lim  x     x  
x 1
 x x 1 x  x

1   1
 lim  x     x   
x 1 x   x
1
 For x > 0, x   2 eqnality holdr at x = 1
x
1   1 
 lim x     x    2
x 1 x   x 
1
 lim  x   3  1  1  3  1
x 1 x
k k
1 k r (1  x 2 )1/3  (1  2x)1/4
50. If  cos  r    
for all k  1 and A =  r , then lim =
r 1 2 r 1 x A x  x2
1 A 
(A) (B) 0 (C) (D)
2 2 2
Ans. (A)
k
1 k
Sol.  cos
r 1
r 
2
 k 1


If k = 1  cos 1 1   1  0
2
If k = 2  cos 1 1  cos 1  2  
 
  cos 1  2    cos 1  2    2  0
2 2
3
If k = 3  cos 1 1  cos 1  2  cos 1 3 
2
  3
  cos 1 3 
  3  0
2 2 2
 1 = 1 = 1 = ...... = 0
k
 A   (r )r  0  A  0
r 1

(1  x 2 )1/3  (1  2x)1/4  0 
 lim  
x0 x  x2 0

Get 10% Instant Discount On Unacademy Plus [Use Referral Code: MCSIR] 184
Maths IIT-JEE ‘Best Approach’ (MC SIR) LIMIT
Apply L–H Rule
1 1
 2x(1  x 2 ) 2/3   (2)(1  2x)3/ 4
1
 lim 3 4 
x 0 1  2x 2

 x 
a tan  
 a  2a 
51. If lim  2    e , then a is equal to
x a  x

  2
(A) – (B)  (C) (D) 
2 2 
Ans. (B)
x
a tan
 a 2a
Sol. lim  2 –  e {1}
x a
 x

x x  x – a 
a tan
2a  a  lim a.tan 
x a 2a  x 
lim e  2 – – 1  e
x a
 x 

  h  h
lim a tan
2a
(a  h) a  h – a  lim a  – cot  
h 0  2a  a  h
 eh  0  ah e
 

 ah
lim
h 0 h
tan
(a  h) 2a  h lim
 ah
h 2a h 0 h
(a  h )
2a 2a
 e e

2a 2 2a
2a
 e a
e 
 e1  1 


a
2

1 1 f (x)
52. If f(x) = lim lim (sin (x + h + t) – sin(x + h) – sin(x + t) + sin x), then lim is
h 0 h t 0 t x 0 x
(A) 0 (B) 1 (C) –1 (D) does not exist
Ans. (B)

1 1  2x  2h  t  t  2x  t  t
Sol. f (x)  lim lim  2 cos    sin  2 cos    sin 
h0 h t 0 t
  2  2  2  2

1 sin t / 2   2x  2h  t   2x  t  
 lim lim cos    cos 
h  0 h t 0 t/2    2   2  

Get 10% Instant Discount On Unacademy Plus [Use Referral Code: MCSIR] 185
Maths IIT-JEE ‘Best Approach’ (MC SIR) LIMIT

 2x  h  h
2 sin   sin
cos(x  h)  cos x  2  2
 lim  lim
h 0 h t 0 h

h
2x  h sin
  2   sin x
 lim  sin  
h0  2  h
2
f (x) sin x
 lim   lim 1
x0 x x0 x

n
53. Let Pn =  cos(x . 2–k) and g(x) = nlim P then
 n
lim g(x) is
k 1 x 0

(A) 0 (B) 1 (C) –1 (D) does not exist


Ans. (B)

n
Sol. Pn  cos(x 2–k ) & g(x)  lim Pn
 n
k 1

x x x
Pn  cos  cos ......cos n
2 4 2

x
2n.sin
 2n  cos x  cos x ......cos x
x
2 n sin n 2 4 2n
2

sinx

x
 2sin A cos A  sin 2A
n
2  sin n
2

Get 10% Instant Discount On Unacademy Plus [Use Referral Code: MCSIR] 186
Maths IIT-JEE ‘Best Approach’ (MC SIR) LIMIT

sin x sin x
g(x)  lim P(x)  lim 
n  n  x x
sin n
2 x
2n

sin x
lim g(x)  lim 1
x 0 x 0 x
[COMPREHENSION TYPE]
Paragraph for Question Nos. 54 to 56

   1
n 
 f x  
   n  if
2 3 n–1
Let f(x) = (x + 1) (x + 2) (x + 2 ) (x + 2 ) ..... (x + 2 ) and g(x) =  lim   f (x)  0
n   f (x) 
 0 if f (x)  0
54. The value of g(0) is equal to
f '(0) f '(0) f (0)
f 2 (0) f (0) f '(0)
(A) e (B) ef '(0) (C) e (D) e
Ans. (C)

n
  1 
f x 
  n  
lim   (1 )
Sol. n 
 f (x) 
 

n
  1 
 f  x  n  
lim   n
n   f (x ) 
 
e
1  f (x 1/n)  f (x )   0 
lim    
f ( x) n  1/n 0
e
Apply L–H Rule
1  f '(x 1/n )  ( 1/n 2 ) 
lim  
f (x ) n  ( 1/n 2 ) 
e
f '(x )

 e f ( x)
f '(0)
f (0)
 g(0)  e

55. If x1, x2, x3,....., xn denote the values of x where f(x) vanishes such that x1 > x2 > x3 > ..... > xn, then
n
r
lim  is equal to
r 1 x r
n 

Get 10% Instant Discount On Unacademy Plus [Use Referral Code: MCSIR] 187
Maths IIT-JEE ‘Best Approach’ (MC SIR) LIMIT
(A) –8 (B) –4 (C) –2 (D) –1
Ans. (B)
Sol. f(x) = 0
(x + 1) (x + 2) (x + 22)......(x +2n–1) = 0
x = – 1, – 21, – 22,......,– 2n –1
n
r 1 2 3 n 
lim   lim     ......  
n 
r 1 x r n   x1 x 2 x 3 xn 

1 2 3
   2  ......
–1 –2 –2

 2 3 
 – 1  1  2  ......
 2 2 

1 2 3
Let S     ......
20 21 22
S 1 2
   ......
2 21 22 Subtracting
S 1 1 1
   ......
2 20 21 2 2
S 1
  2 S = 4
2 1 1
2

n
r
 lim   4
n 
r 1 xr
56. Let f(x) = x sin x and g(x) = f(x) f '(x), then number of distinct real roots of equation g(x) = 0 where
x  (–2, 2) is
(A) 5 (B) 6 (C) 7 (D) 8
Ans. (C)
Sol. f(x) = x sin x

Get 10% Instant Discount On Unacademy Plus [Use Referral Code: MCSIR] 188
Maths IIT-JEE ‘Best Approach’ (MC SIR) LIMIT
f (x) = x cos x + sin x

g(x) = f(x)  f (x) = 0
= x  sin x  [x cos x + sin x] = 0
x = 0 sin x = 0  x cos x + sin x = 0
sin x = 0 x = 0, , – 
0
x cos x + sin x = 0
tan x = – x  total 5 points
 7 solutions
Paragraph for Question Nos. 57 to 59
Let a sequence be defined by a1 = 0 and an+1 = an + 4n + 3 for all n  1 (n  N)
57. The value ak in terms of k is (k  N)
(A) (k – 1)(2k + 3) (B) (k – 1)(3k + 1) (C) (k – 1)(k + 3) (D) (k – 1)(2k + 5)
Ans. (A)
Sol. ak = (2k + 3) (k – 1)

a n  a 4n  a  .....  a
42 n 410 n
58. The value of lim
n  a n  a 2n  a  .....  a
22 n 210 n

(A) 685 (B) 683 (C) 687 (D) 690


Ans. (B)

a n  a 4n  a 42 n  ......  a 410 n
Sol. lim
n  a n  a 2n  a 22 n  ......  a 210 n

(n – 1)(2 n  3)  (4 n – 1)(2  4 n  3)  ......


 lim
n  (n– 1) (2n  3)  (2n – 1) (2n  3)  ......

 1  3  1  3
1 – n  2  n    4 – n  2  4  n   ......
     
lim
 n 
 1  3  1  3
 1–  2     2 –   2    ......
 n  n  n  n

1  2  4  2  4  42  2  42  ......

1  2  2  2  2  22  2  22  ......

Get 10% Instant Discount On Unacademy Plus [Use Referral Code: MCSIR] 189
Maths IIT-JEE ‘Best Approach’ (MC SIR) LIMIT

411 –1
1  4  42  ...... 4 –1 411 –1
 
 1  2  22  ...... 211 –1 (211 –1)  3
2 –1
 683

a n  a 9n  a  .....  a  4L 
92 n 910 n
59. If the value of lim = L, then  11  is
n  a n  a 3n  a  .....  a 3 
32 n 310 n

[.] denotes greatest integer function


(A) 4 (B) 3 (C) 2 (D) 1
Ans. (D)

an  a 9n  a 9 2 n  ......  a 910 n
lim
Sol. n 
an  a 3n  a 32 n  ......  a 310 n

(n – 1)(2 n  3)  (9 n –1)(2  9 n  3)  ...... (2  910 n –1) (2  910 n  3)


lim
 n 
(n– 1)(2 n  3)  (3 n– 1) (2  3 n  3)  ......  (310 n– 1) (2  310 n  3)

 1  3  1  3  10 1  10 3
 1–  2     9 –  2  9    ......  9 –  2  9  
 n  n  n  n  n  n
lim
 n 
 1  3  1  3  10 1  10 3
1 –  2     3 –  2  3    ......  3 –  2  3  
 n  n  n  n  n  n

1 2  9(2  9)  92 (2  92 )  ....... 910 (2  910 )



1 2  3(2  3)  32 (2  32 )  ....... 310 (2  310 )

1  911
 L
4(1  311 )

11 22
 4L   9  1   3  1 
  311   (311  1)  311   322  311   1
 
   

Paragraph for Question Nos. 60 to 61

sin x  ae x  be  x  cn(1  x)
Let lim 1
x 0 x2
60. The ordered triplet (a, b, c) is

Get 10% Instant Discount On Unacademy Plus [Use Referral Code: MCSIR] 190
Maths IIT-JEE ‘Best Approach’ (MC SIR) LIMIT
 1 1   1 1   1 1  1 1 
(A)   ,  , 1 (B)   , ,1  (C)   , , 2  (D)  ,  , 2 
 2 2   2 2   2 2  2 2 
Ans. (D)

 x3   x2   x2   x2 
 x   a 1  x   b 1  x   c x 
Sol. 3!   2!   2!   2 
lim 1
x 0 x2

 a  b a  b  c 1 a  b  c
 lim  2    1
x0  x  x 2
a+b=0&a–b+c+1=0&a+b–c=2
1 1
on solving, we get a  , b  ,c  2
2 2

61. The roots of the quadratic equation ax2 + bx + c = 0


(A) both positive (B) both negative
(C) one positive and one negative (D) complex conjugate
Ans. (C)
Sol. ax2+ bx + c = 0

x2 x
– –20
2 2

x2 – x – 4  0

   1 
  One Root positive & one negative
   – 4

Paragraph for Question Nos. 62 to 64

 
 1   n 
Let f(x) = lim   where x  R –   , n  I
y   sin(x  (1/ y))   2
 y  n   
  sin x 

n
a = lim lim ,nI
n  x 0 cot 2 x cot 2 x cot 2 x tan 2 x
(1 2  .....  (2n) )
1
4  2
b = lim  tan 1 x  x 1 (where N represent set of natural numbers, I represent set of integers and R
x 1   
represent set of real numbers)
Paragraph for question no. 62 to 64

Get 10% Instant Discount On Unacademy Plus [Use Referral Code: MCSIR] 191
Maths IIT-JEE ‘Best Approach’ (MC SIR) LIMIT
1/ y
Sol. f (x)  lim  %
y   1
ln  sin  x     ln  sin x 
  y
Apply L–H Rule
1/ y sin  x  1/ y 
 lim  lim
 
y cos x  1/ y  1/ y 2

y  cos  x  1/ y 

sin  x  1/ y

f (x)  tan x

n
a  lim tan 2 x
n 
lim 1
x 0
 cot 2 x
2 cot 2 x
 ......   2n 
cot 2 x

n
 lim cot 2 x cot 2 x
n 
cot 2 x  tan 2 x
 1   2 
 2n      ......
 2n   2n  

n 1 1
 lim   a
n  2n 2 2
1
4  x2 1 
b  lim  tan 1 x  (1 )
n 1   
4 1
tan x 1

lim
x 2 1
e n 1
(%)
Apply L–H Rule
4 1

 1 x 2 1 1
lim
e x1 2x
 e  b  e

62. f(x) = ax has 5 roots in (0, m), m  N, then m =


(A) 5 (B) 6 (C) 8 (D) 10
Ans. (B)
Sol. f(x) = ax has 5 roots in (0, m)
x
 tan x   y(let)
2
x
 y  tan x & y 
2

Get 10% Instant Discount On Unacademy Plus [Use Referral Code: MCSIR] 192
Maths IIT-JEE ‘Best Approach’ (MC SIR) LIMIT

y






O  2 3 4 5 6 x

 m6

63. If f(x) = bx has 6 roots in (0, m), m  N then m =


(A) 5 (B) 6 (C) 8 (D) 10
Ans. (B)
Sol. f(x) = bx has 6 roots in (0, m)
 f(x) = e1/ · x  tanx = e1/ x = y (let)
y = tan x & y = e1/ · x

y 





O  2 3 4 5 6 x

 m6

64. The number of points of intersection of two curves y = |f(x)| and y = b in interval (0, 4) is
(A) 5 (B) 6 (C) 8 (D) 10
Ans. (C)
Sol. y = | f(x) | & y = b ; x  (0, 4)
1/
y = | tan x | & y = e
 Period of | tan x | is 
 Number of points of intersection in (0, 4) & y = 2 × 4
=8
y
y = e1/
 

0 /2  x

Get 10% Instant Discount On Unacademy Plus [Use Referral Code: MCSIR] 193
Maths IIT-JEE ‘Best Approach’ (MC SIR) LIMIT

65. Statement-1 : If  and  are positive reals and [*] denotes greatest integers function then
x   
lim  
x 0    x 

because
{y}
Statement-2 : lim = 0, {.} denotes fractional part function.
y y

(A) Statement-1 is true, statement-2 is true; statement-2 is correct explanation for statement-1.
(B) Statement-1 is true, statement-2 is true; statement-2 is NOT the correct explanation for statement-1.
(C) Statement-1 is true, statement-2 is false.
(D) Statement-1 is false, statement-2 is true.
Ans. (A)

x   x    
Sol. lim    lim  –   
x 0   x  x 0   x  x  

  x   
 lim  –   
x 0
    x 

   x  
 lim  –    
x 0
    x 

 
 
  x 
 – lim   
  x0  
x
{y}
 lim  0 , because {y}  [0, 1) & y  
y  y

66. Lim f(x) does not exist when


x c

(A) f(x) = [[x]]  [2x  1], c = 3 (B) f(x) = [x]  x, c = 1


tan (sgn x)
(C) f(x) = {x}2  {x}2, c = 0 (D) f(x) = ,c =0.
sgn x
where [x] denotes step up function & {x} fractional part function.
Ans. (B,C)
Sol. (A) f(x) = [x] – [2x] + 1 at x = 3
RHL  lim f (x)  3  6  1  2
x 3

LHL  lim f (x)  2  5  1  2


x 3

 lim f (x)  2
x 3

(B) f(x) = [x] – x = –{x} at x = 1

Get 10% Instant Discount On Unacademy Plus [Use Referral Code: MCSIR] 194
Maths IIT-JEE ‘Best Approach’ (MC SIR) LIMIT
RHL = 0, LHL = –1
 lim f (x) does not exist
x 1

(C) f(x) = {x}2 – {–x}2 at x = 0


RHL  lim h 2  (1  h)2  lim h 2  (1  h)2  1
x  0 h x 0

2 2
LHL  lim (1  h)  h  1
x0 h

 lim f (x) does not exist


x0

tan(sgn(x))
(D) f (x)  at x = 0
sgn(x)
tan(1)
RHL  lim  tan1
x0 1
tan( 1)
LHL  lim  tan1
x0 1
 lim f (x)  tan1
x0

x.2 x  x  n2 
67. Let f(x) = & g(x) = 2x sin  x  then :
1  cos x  2 
(A) lim f(x) = ln 2 (B) lim g(x) = ln 4 (C) lim f(x) = ln 4 (D) lim g(x) = ln 2
x 0 x  x 0 x 
Ans. (C, D)

x  2x – x  ln 2 
Sol. f (x)  , g(x)  2 x  sin  x 
1– cos x  2 

2 x –1
x
x(2 – 1) x ln 2
lim f (x)  lim  lim 
x 0 x 0 1 – cos x x 0 1 – cos x 1
2
x 2
= 2 ln 2
= 2 ln 4

 ln2 
sin  x 
 ln2  2 
limg(x)  lim2x  sin  x   lim   .ln2  ln2
x x
2  x  ln2
2x

x 1
68. Let f(x) = . Then :
2x 2  7x  5

Get 10% Instant Discount On Unacademy Plus [Use Referral Code: MCSIR] 195
Maths IIT-JEE ‘Best Approach’ (MC SIR) LIMIT
1 1
(A) lim f(x) =  (B) lim f(x) =  (C) lim f(x) = 0 (D) lim does not exist
x 1 3 x 0 5 x  x 5/2
Ans. (A,B,C,D)
x 1 1
Sol. f (x)  
(2x  5)(x  1) 2x  5

1
(A) lim f (x) 
x 1 3
1
(B) lim f (x) 
x 0 5
(C) lim f (x)  0
x 

(D) xlim f (x)  does not exist { RHL   & LHL  }
5/2

69. Which of the following limits vanish?


1 1
(A) Limit
x   x sin
4 (B) Limit
x   /2 (1  sin x) . tan x
x
2
2x  3 [x]2  9
(C) Limit . sgn (x) (D) Limit
x3 
x x2  x  5 2
x 9
where [ ] denotes greatest integer function
Ans. (A, B, D)

1 1
1 4
sin
4
1 x x
Sol. (A) lim  x  sin  lim 
x  x x  x 1
x
1

4
 lim x 1  0
x

    
(B) lim (1 – sin x)  tanx  lim  1 – sin   h    tan   h 
x

2
h  0
 2  2 

– 1 – cos h  0 
 lim 1 – cos h  – cot h   lim  for 
h 0 h 0 tan h 0 

Get 10% Instant Discount On Unacademy Plus [Use Referral Code: MCSIR] 196
Maths IIT-JEE ‘Best Approach’ (MC SIR) LIMIT
– (0  sin h)
By L’ Hospital Rule  lim  lim– sin h cos 2 h
h 0 sec 2 h h 0

=0

3
2 
x  3 x2
(C) lim .sgn(x)  lim .1  2  0
x  x2  x – 5 x  1 5
1 – 2
x x

[x]2 – 9 [3  h]2 – 9 9–9


(D) lim 2
.  lim 2
 lim 2 0
x 3 x –9 h  0 (3  h)  9 h 0 h  6h  9 – 9

70. In which of the following cases limit exists at the indicated points.
[x  | x |]
(A) f(x) = at x = 0 where [] denotes the greatest integer function
x
(B) f(x) = (x – 3)1/5 Sgn(x – 3) at x = 3, where Sgn stands for Signum function
1
xe x tan 1 | x |
(C) f(x) = 1 at x = 0 (D) f(x) = at x = 0
x
1 ex
Ans. (A,B,C)
[x  | x |]
Sol. (A) f (x)  at x = 0
x
[h  | h |] [2h]
RHL  lim  lim 0
x0 h h h0 h
[  h  |  h |] [0]
LHL  lim  lim 0
x 0  h h h  0 h
 Limit exist.
(B) f(x) = (x – 3)1/5 sgn (x – 3) at x = 3
RHL  lim (h)1/3 sgn (h)  lim (h)1/3  1  0
x 3  h h0

LHL  lim ( h)1/3 sgn (h)  lim ( h)1/3  (1)  0


x  3 h h0

 Limit exist.
xe1/ x
(C) f (x)  at x = 0
1  e1/ x
( h)  e 1/ h (h)  e 1/ h
LHL  lim  lim 0
x0 h 1  e1/ h h  0 1  e
1/ h

Get 10% Instant Discount On Unacademy Plus [Use Referral Code: MCSIR] 197
Maths IIT-JEE ‘Best Approach’ (MC SIR) LIMIT
(h)  e1/ h h
RHL  lim 1/ h
 lim 1/ h
0
x0 h 1  e h  0 1/ e 1
 Limit exist.
tan 1 (| x |)
(D) f (x)  at x = 3
x
tan 1 h tan 1 h
RHL  lim  lim 1
x  0 h h h0 h
tan 1 h  tan 1 h
LHL  lim  lim  1
x0 h h h0 h
 Limit does not exist.
x
 ax  1  2 2
Q.71 Consider the function f(x) =   where a + b  0 then xlim f(x)
 bx  2  

(A) exists for all values of a and b (B) is zero for 0 < a < b
1 1
   
(C) is non existent for a > b > 0 (D) is e a or e b if a = b

Ans. (B,C,D)
x
 ax  1 
Sol. f (x)   
 bx  2 
Case I a=b
x
 ax  1 
lim f (x)  lim   (1 )
x  x  ax  2
 

 ax 1   x 
lim  1 x lim  
= e x   ax  2 
e x   ax  2 

1 1
= e a or e b { a = b}

a
Case II b>a>0  1
b

a
 lim f (x)    0
x 
b
a
Case III a>b>0  1
b

a
 lim f (x)     Does not exist.
x 
b

Get 10% Instant Discount On Unacademy Plus [Use Referral Code: MCSIR] 198
Maths IIT-JEE ‘Best Approach’ (MC SIR) LIMIT

2    2 f () 2  2  1
Q.72 Assume that Lim f () exists and   holds for certain interval
  1 3 2 3
containing the point  = – 1 then Lim f ()
  1
(A) is equal to f (–1) (B) is equal to 1 (C) is non existent (D) is equal to – 1
Ans. (A,D)

2    2 f () 2  2  1
Sol. lim  lim 2  lim
 1 3  1  1 3

lim f () 1  2  1
1  1  2  1
  
1  3 1 1  3

 1  lim f ()  1
1

 by sandwitch theorem,
lim f ( )  1
1

2    2 f () 2  2  1
Also,  2 
3  3
Put   1,

1 1  2 1 2 1
 f (  1) 
1  3 1  3
 – 1  f ( 1)  1
 f ( 1)  1

 lim f ()  1  f (1)


1

a(2x 3  x 2 )  b(x 3  5x 2  1)  c(3x 3  x 2 )


Q.73 If lim = 1 then which of the following relations between
x  a(5x 4  x)  bx 4  c(4x 4  1)  2x 2  5x

a, b and c must hold good ?


(A) a + 2b + c = 1 (B) 5a – b + 4c = 0
(C) 2a + b – 3c = 0 (D) a – 5b + c + 2 = 0

Ans. (B,C,D)

a(2x 3  x 2 )  b(x 3  5x 2  1)  c(3x 2  x 2 )


Sol. lim 1
x   a(5x 4  x)  bx 4  c(4x 4  1)  2x 2  5x

Get 10% Instant Discount On Unacademy Plus [Use Referral Code: MCSIR] 199
Maths IIT-JEE ‘Best Approach’ (MC SIR) LIMIT

(2a  b  3c) x 3  (  a  5b  c)x 2  (a  b)


 lim 1
x  (5a  b  4c)x 4  2x 2  (5  a)x  c
 a  5b  c
 2a  b  3c  0 & 5a  b  4c  0 & 1
2
 2a  b  3c  0 & 5a  b  4c  0 & a  5b  c  2  0

h2 x3 1
Q.74 If lim  x then f(x) can not be equal to
h 0 f (x  2h)  2f (x  h)  f (x) 2 2x
(A) tan–1(x2) + ax + b (B) tan–1(x) + ax2 + b
(C) tan–1(x2) + ax2 + b (D) tan–1(x3) + ax + b
Ans. (A,B,C,D)

Sol. h2 0
LHS  lim 0
h  0 f (x  2h)  2f (x  h)  f (x)
 
Apply L-H Rule

2h 0
 lim  
h  0 f '(x  2h)  2  2f '(x  h)
0

2
 lim
h  0 f ''(x  2h)  4  2f ''(x  h)

1 1
 
2f "(x)  f "(x) f "(x)

1 x3 1  x 4  2x 2  1
   x  
f ''(x) 2 2x 2x

2x
 f ''(x) 
x  2x 2  1
4

2x
 f '(x)   dx  c1
x  2x 2  1
4

Put x2 = t  2x dx = dt
dt dt
  2
 c1     c1
t  2t  1 (t  1)2

1 1
f '(x)   c1  2  c1
t 1 x 1

dx
 f (x)    c1  1.dx
x2 1

Get 10% Instant Discount On Unacademy Plus [Use Referral Code: MCSIR] 200
Maths IIT-JEE ‘Best Approach’ (MC SIR) LIMIT
f (x)  tan 1 (x)  c1x  c 2

(sin(sin x)  sin x) 1
Q.75 If lim 3 5
 , then
x 0 ax  bx  c 12
(A) a = 2 (B) a = –2 (C) c = 0 (D) b  R
Ans. (A,C,D)
sin(sin x)  sin x 1
Sol. lim 
x 0 ax 3  bx 5  c 12
 sin x  x   sin x  x 
2 cos    sin   1
  2   2 
lim 3 5
x 0 ax  bx  c 12

 sin x  x   sin x  x 
2 sin    1
 2   2 
 lim
x  0  sin x  x  12
3 5
   (ax  bx  c)
 2 

 x3 x5  
 x   x
2 3! 5!    1
lim   3
x 0 2  ax  bx  c  12

 
 

x3 x 5

 lim 3 3! 5!  1
x  0 ax  bx 5  c 12
 for limit to exist, c = 0

x3 x5

1
& lim 3!3 5!5 
x  0 ax  bx 12

1 x2

& lim 3! 5!  1
x  0 a  bx 2 12

1 1
 bR &   a=2
6a 12
 a  2, b  R & c = 0

Get 10% Instant Discount On Unacademy Plus [Use Referral Code: MCSIR] 201
Maths IIT-JEE ‘Best Approach’ (MC SIR) LIMIT
1 
Q.76 I f lim n = 1, where x  (2n + 1) ,  n  I, then possible value(s) of  will be :
x 0
 tan 2 x    2
 2  1
 tan x  1 
1 3 1 5
(A)  (B) (C) (D)
2 4 4 4
Ans. (A,B,C)
Sol. For limit to be 1,

tan 2 x  
 1   (1  tan 2 x)  tan 2 x    1  tan 2 x
tan 2 x  1

 1  2 tan 2 x    1
 <1

Q.77 Column–I Column-II


nx
(A) lim (P) 2
x 1 x 4  1

3e x  x 3  3x  3 2
(B) lim (Q)
x 0 tan 2 x 3

  2 tan 1 x 3
(C) lim (R)
x   1 2
n  1  
 x
2sin x  sin 2x 1
(D) lim (S)
x 0 x(cos x  cos 2x) 4

e x  e x  2x 11
(E) lim (T)
x 0 x  sin x 2

1  1  x 2 cos x.cos 3x
(F) lim
x 0 x2
Ans. (A)S; (B)R; (C)P; (D)Q; (E)P; (F)T
n x 0
Sol. (A) lim  
x 1 x4 1 0
Apply L-H Rule

1
1
 lim x 3 
x 1 4x 4

Get 10% Instant Discount On Unacademy Plus [Use Referral Code: MCSIR] 202
Maths IIT-JEE ‘Best Approach’ (MC SIR) LIMIT

3e x  x 3  3x  3 3e x  x 3  3x  3  0 
(B) lim  lim  
x 0 tan 2 x 2 x 0 x2 0
2
x
x
Apply L-H Rule

3e x  3x 2  3  0
 lim  
x 0 2x  0

3e x  6x 3
 lim 
x0 2x 2

  2 tan 1 x 0
(C) lim  
x   1 0
n  1  
 x

Apply L-H Rule

2

1  x2 2
 lim  lim 2
x    x   
 1   1   1  1 
   2    1  2 
1 1
 1   x  1  x 
 x  x 

2sin x  sin 2x 0


(D) lim 0
x  0 x (cos x  cos 2x)
 

 x3   8x 3 
2  x     2x 
 3!   3! 
 lim
x0  x 2  4x 2  
x 1    1  
 2!  2!  

x 3 8x 3 1 8
  
 lim 3 3!  3 62
x 0   x 2 4x 2  1 4 3
x   
 2! 2!  2 2

e x  e  x  2x 0
(E) lim  
x 0 x  sin x 0

Get 10% Instant Discount On Unacademy Plus [Use Referral Code: MCSIR] 203
Maths IIT-JEE ‘Best Approach’ (MC SIR) LIMIT

 x 2 x3   x2 x3 
1  x    1  x    2x
 2! 3!   2! 3! 
 lim
x0  x3 
x x  
 3! 

2x 3
 lim 3!3  2
x0 x

3!

1  1  x 2  cos  cos 3x 0


(F) lim  
x 0 x2 0
Apply L-H Rule

2x
 cos x  cos 3x
2 1  x 2 sin x cos 3x 3 1  x 2 cos x sin 3x
 lim 2 1  x  
x 0 2x 2x 3x

1 1 9 11
   
2 2 2 2

  cos x  cos a 
xlim   , a  n
a  cot x  cot a 

Q.78 Let f(a) = 

2 , a  (2n  1) , n  I and b = lim
n 
 n 2  4n  1  n 2  2 
2 , a  2n



Column–I Column-II
(A) If f(x) is periodic with fundamental period k where k  R, then k  (P) 0
(B) The integers in the range of |f(x)| is/are (Q) 1
(C) Let k where k  R be the root of the equation |f(x)| = b in (R) 2
interval [–4, 4], then possible values of |k| is/are
 b|x| | x | 2
(D) Let g(x) =  then possible values g(x) (S) 4
sgn | 2b  x | 2  | x |
can attain is/are
Ans. (A)R,S; (B)Q,R; (C)P,Q,R,S; (D)P,Q,R,S;

Get 10% Instant Discount On Unacademy Plus [Use Referral Code: MCSIR] 204
Maths IIT-JEE ‘Best Approach’ (MC SIR) LIMIT

 n 2  4n  1  n 2  2 
Sol. b  lim  2
n  4n  1  n 2
 2 
n 
 n 2  4n  1  n 2 2

n 2  4n  1  n 2  2
 lim
n 
n 2  4n  1  n2  2

1
4
n 4
lim 
= x  4 1 2 2
1  2  1
n n n

 b=2
cos x  cos a
Now lim ; a  n
x a cot x  cot a
Apply L-H Rule
 sin x
 lim  sin 3 a
x  a  cos ec 2 x

 sin 3 x ; x  n

(A)  f (x)   2 ; x  (2n  1)  ; n  I
2 ; x  2n

 f(x) is periodic with fundamental period 2

y = |f(x)|

x
0

(B)  Integers in the range of |f(x)| are 1 and 2


(C) |f(x)| = b  |f(x) = 2 |
 x = n ; n  I

 2|x| ; 2  x  2
(D) g(x)  
sgn | 4  x | ; x  2 or x  2

Get 10% Instant Discount On Unacademy Plus [Use Referral Code: MCSIR] 205
Maths IIT-JEE ‘Best Approach’ (MC SIR) LIMIT

 2 x ; 2  x  0
 x
2 ; 0x2
g(x)  
 1 ; 2  x  4or x  4
 0 ; x  4

Get 10% Instant Discount On Unacademy Plus [Use Referral Code: MCSIR] 206
EXERCISE-IV

1 − cos 2x
1. lim is [2002]
x →0 2x
(A) 1 (B) –1 (C) zero (D) does note exist
Ans. (D)
1 − cos 2x | sin x |
Sol. lim = lim
x →0 2x x → 0 x
RHL = 1 & LHL = – 1

1 − cos 2x
 lim does not exist.
x →0 2x

x
 x 2 + 5x + 3  
2. lim  2  (1 ) [2002]
x →  x + x + 3 

(A) e4 (B) e2 (C) e3 (D) 1


Ans. (A)
x
x  x 2 + 5x + 3 
 x 2 + 5x + 3  lim  2 −1
Sol. lim  2  = e x→ x + x +3 
x →  x + x + 3 

 4x 2  
lim   
=e x →  x + x +3    
2
= e4

xf (2) − 2f (x)
3. Let f(2) = 4 and f (2) = 4 . Then lim is given by [2002]
x →2 x−2
(A) 2 (B) –2 (C) –4 (D) 3
Ans. (C)
xf (2) − 2f (x)  0 
Sol. lim  
x →2 x−2 0
Apply L-H Rule
f (2) − 2f '(x)
= lim = f (2) – 2f '(2) = 4 – 8 = – 4
x →2 1
log x n − [x]
4. lim , n  N , ([x] denotes greatest integer less than or equal to x)
x → [x]
(A) has value –1 (B) has value 0 (C) has value 1 (D) does not exist
[2002]

207
Ans. (D)
log x n − [x]  n log x 
Sol. lim = lim  − 1
x → [x] x →0
 [x] 
Now,  x – 1 < [x]  x

1 1 1 log x log x log x


  <   <
x [x − 1] x − 1 x [x] x −1

log x    log x log x   


 lim    lim  lim  
x → x    x → [x] x → x 

Apply L-H Rule


1/ x log x 1/ x
 lim  lim  lim
x → 1 x → [x] x → 1
log x log x
 0  lim  0  lim =0
x → [x] x → [x]

 n log x 
 lim  − 1  = 0 − 1 = −1
x →
 [x] 

f (x) − 1
5. If f(1) = 1, f ' (1) = 2, then lim is [2002]
x →1 x −1
(A) 2 (B) 4 (C) 1 (D)
Ans. (A)
f (x) − 1  0 
Sol. lim  
x →1 x −1  0 
Apply L-H Rule
f '(x)
2 f (x) f '(1)
lim = =2
x →1 1 f (1)
2 x

1 + 24 + 34 + ...... + n 4 1 + 23 + 33 + ...... + n 3
6. lim − lim [2003]
n → n5 n → n5
1 1 1
(A) (B) (C) Zero (D)
5 30 4
Ans. (A)

208
14 + 24 + ...... + n 4 13 + 23...... + n 3
Sol. lim − lim
n → n5 n → n5

 n(n + 1) 
2


1 1 2
4 4 4
  
n  2 
= lim    +   + .... +    − lim
n → n   n  n n  n → n 5

4
1 n r 11 2 1 
= lim    − lim  + 2 + 3 
r =1  n 
n → n n → 4  n n n 
1 1
x5 1
= =  x dx – 0 =
4
= Ans.
0
5 0 5

log(3 + x) − log(3 − x)
7. If lim = k , the value of k is [2003]
x →0 x
2 1 2
(A) − (B) 0 (C) − (D)
3 3 3
Ans. (A)
log(3 + x) − log(3 − x)
Sol. lim =k
x →0 x
Apply L-H Rule
1 1
+
 lim 3 + x 3 − x = k 
2 2
=kk=
x →0 1 3 3

  x 
1 − tan  2   [1 − sin x]
8. lim   is [2003]
x→   x 

2 1 + tan   [ − 2x]
3
  2  

1 1
(A)  (B) (C) 0 (D)
8 32
Ans. (D)
 x
tan  –   (1 − sin x )
Sol. lim 4 2

 3
x→ 
2 8 − x 
2 

  h
tan  − −  (1 − cos h)
RHL = lim 4 4 2

x→ +h 8(– h)3
2

209
h h
2sin 2
tan
= lim+ 2  2= 1 2
2
h→0 h h 16 4
8  2 4
2 4
1
=
32

2x
 a b 
9. If lim 1 + + 2  = e2 , then the values of a and b, are [2004]
x →  x x 
(A) a = 1 and b = 2 (B) a = 1, b  R
(C) a  R, b = 2 (D) a  R, b  R
Ans. (B)
2x
 a b  (1 ) = e2
Sol. lim 1 + + 2 
x →  x x 
 a b 
lim 1+ + 2 −1 2x
 ex →  x x 
= e2
 b
lim 2 a + 
 e x→  x
= e2  a = 1 & b  R

1 − cos(ax 2 + bx + c)
10. Let  and  be the distinct roots of ax2 + bx + c = 0, then lim is
x → (x − )2
equal to [2005]
a2 −a 2 1
(A) ( − ) 2 (B) 0 (C) ( − ) 2 (D) ( − ) 2
2 2 2
Ans. (A)
Sol.  &  are roots of equation ax2 + bx + c =0

 ax2 + bx + c = a(x – ) (x – )
a(x − ) (x − )
2sin 2 ( )
1 − cos(ax 2 + bx + c) 2
 lim = lim
x → (x − )2 x → (x − ) 2

 a(x − ) (x − ) 
2sin 2   a 2 (x − ) 2
 lim  2 
x → (x −  ) 2
(x − ) 2
4
a2
4

a2
 ( − )2
2

210
f (3x) f (2x)
11. Let f : R → R be a positive increasing function with lim = 1 . Then lim =
x → f (x) x → f (x)

2 3
(A) (B) (C) 3 (D) 1 [2010]
3 2
Ans. (D)
Sol. for x > 0, x < 2x < 3x

and f(x) is positive increasing function


 f(x) < f(2x) < f(3x)
f (2x) f (3x)
 1 
f (x) f (x)
f (2x) f (3x)
 lim 1  lim  lim
x → x → f (x) x → f (x)

f (2x) f (2x)
 1  lim  1  lim =1
x → f (x) x → f (x)

 1 − cos{2(x − 2)} 
12. lim   [2011]
x →2  x−2 

(A) equals 2 (B) equals – 2


1
(C) equals (D) does not exist
2
Ans. (D)
1 − cos 2(x − 2) 2 | sin(x − 2) |
Sol. lim = lim
x →2 x−2 x →2 (x − 2)

LHL= – 2 & RHL = 2


1 − cos 2(x − 2)
 lim does note exist
x →2 x−2

( f (x) ) −9
2

13. Let f : R → [0, ) be such that lim f (x) exists and lim = 0 . Then lim f (x)
x →5 x →5 | x −5| x →5

equals [JEE Mains 2011]


(A) 0 (B) 1 (C) 2 (D) 3
Ans. (D)

211
Sol. lim f(x) exists.
x →5

( f (x) ) −9
2

= 0  lim f ((x)) = 9
2
lim
x →5 | x −5| x →5

 lim f (x) = 3 Ans. { f : R → [0, ]}


x →5

(1 − cos 2x)(3 + cos x)


14. lim is equal to [IIT Mains 2013]
x →0 x tan 4x
1 1
(A) 2 (B) – (C) (D) 1
4 2
Ans. (A)
(1 − cos 2x)(3 + cos x)
Sol. lim
x →0 tan 4x
x  4x
4x
2sin 2 x
= lim =2
x →0 x2

sin( cos 2 x)
15. lim is equal to [IIT Mains 2014]
x →0 x2

(A)  (B) (C) 1 (D) – 
2
Ans. (A)
sin( cos 2 x)  0 
Sol. lim  
x →0 x2 0

sin (  −  sin 2 x ) sin ( sin 2 x)  sin 2 x


= lim = lim 
x →0 x2 x →0  sin 2 x x2
=

16. Let f(x) be a polynomial of degree four having extreme values at x = 1 and x = 2. If
 f (x) 
lim 1 + 2  = 3 , then f(2) is equal to [IIT Mains 2015]
x →0  x 
(A) 4 (B) –8 (C) –4 (D) 0
Ans. (D)
Sol. Let f(x) = ax4 + bx3 + cx2 + dx + e
 f (x)   d e 
lim 1 + 2  = 3  lim 1 + ax 2 + bx + c + + 2  = 3
x →0  x  x →0  x x 
 d=e=0& 1+c=3 c=2

212
f(x) have extreme value at x = 1 & x = 2
 f (1) = 0 & f (2) = 0
 4a + 3b + 4 = 0 & 32a + 12b + 8 = 0
1
On solving, a = &b=–2
2
x4
 f(x) = − 2x 3 + 2x 2
2
 f(2) = 8 – 16+ 8 = 0

(1 − cos 2x)(3 + cos x)


17. lim is equal to [IIT Mains 2015]
x →0 x tan 4x
1
(A) (B) 4 (C) 3 (D) 2
2
Ans. (D)
(1 − cos 2x)(3 + cos x) 2sin 2 x(3 + cos x)
Sol. We have lim = lim
x →0 x tan 4x x →0 tan 4x
x  4x
4x
2sin 2 x (3 + cos x) 1
= lim 2
 lim 
x →0 x x →0 4 tan 4x
lim
x →0 4x

4  tan  
= 2  1  lim = 1
4 →0  
= 2.
1
18. Let p = lim+ 1 + tan 2 x
x →0
( ) 2x then log p is equal to [IIT Mains 2016]

1 1
(A) 2 (B) 1 (C) (D)
2 4
Ans. (C)
1
Sol. (
p = lim+ 1 + tan 2 x
x →0
) 2x (1)

tan 2 x
(
lim 1+ tan 2 x −1 1 ) lim
x →0+ 2( x )2
= e x →0+ 2x =e
p = e1/2
1
 log p =
2

213
19. For x  R, f(x) = |log2 – sin x| and g(x) = f(f(x)), then : [JEE(Main)-2016]

(A) g is differentiable at x = 0 and g'(0) = –sin(log 2)

(B) g is not differentiable at x = 0

(C) g'(0) = cos(log 2)

(D) g'(0) = –cos(log 2)

Ans. (C)
Sol. f(x) = |ln 2 – sin x|

f(f(x)) = |ln 2 – sin (|ln 2 – sin x|)|

In the vicinity of x = 0

g(x) = ln 2 – sin (ln 2 – sin x)


 g(x) is differentiable at x = 0
 g(x) = cos (ln 2 – sin x) × cos x
 g(0) = cos (ln 2) Ans.

cot x − cos x
20. lim equals [JEE Mains 2017]
 (  − 2x)3
x→
2

1 1 1 1
(A) (B) (C) (D)
24 16 8 4
Ans. (B)

Sol. x→ +t
2
– tan t + sin t – sin t(1 − cos t)
 lim + = lim +
t →0 (– 2t) 3
t →0 −8t 3
2sin 2 t / 2 1
= lim + = Ans.
t →0 8  t 2 / 4  4 16

21. For each t  R, let [t] be the greatest integer less than or equal to t. Then,
1 2 15  
lim+ x    +   + .... +    [JEE Mains 2018]
x →0
x x  x 
(A) does not exist (in R) (B) is equal to 0
(C) is equal to 15 (D) is equal to 120

214
Ans. (D)
 1 2 15  
Sol. lim+  x   + x   + .... + x   
x →0
 x x  x 
15(16)
= 1 + 2 + 3 + ……+ 15 = = 120 Ans.
2

1 + 1 + y4 − 2
22. lim = [JEE Main 2019 (09-01-2019-Shift-1)]
y→0 y4
1
(A) exists and equals (B) does not exist
4 2
1 1
(C) exists and equals (D) exists and equals
2 2 2 2 ( 2 + 1)
Ans. (A)

1 + 1 + y4 − 2  0 
Sol. lim  
y→0 y4 0

1 + 1 + y4 − 2 1 + 1 + y4 + 2
lim 
y→0 y4 1 + 1 + y4 + 2

1 + y4 − 1 1
= lim 
y→ 0 y4 2 2

1 1 + y4 − 1 1 + y4 + 1
= lim 
2 2 y→ 0 y4 1 + y4 + 1

1 1 + y4 − 1 1 1 1 1
= lim  =  = Ans.
2 2 y→ 0 4 2 4 2
y 1 + y4 + 1 2 2

23. For each x  R, let [x] be the greatest integer less than or equal to x. Then

x ( x  + x ) sin  x 
lim is equal to [JEE Main 2019 (09-01-2019-Shift-2)]
x→0− x
(A) 1 (B) sin1 (C) – sin1 (D) 0

Ans. (C)
(– h) (–1 + h)sin(–1)
Sol. lim
x →0 – h h

215
= lim (−1 + h)sin1
h →0 +
= – sin 1 Ans.

24. For each t  R , let [t] be the greatest integer less than or equal to t.
 
(1− | x | + sin |1 − x |) sin  [1 − x] 
Then, lim+ 2 
x →1 |1 − x | [1 − x]
[JEE Main 2019 (10-01-2019-Shift-1)]
(A) does not exist (B) equals 1 (C) equals –1 (D) equals 0
Ans. (D)
 
((1 − 1 − h) + sin h)  sin  [–h] 
Sol. lim 2 
x →1+ h h[– h]

 –  
( – h + sin h )  sin 

= lim +   2 
x →0 h(–1)

 sin h 
= lim +  − 1 +  = −1 + 1 = 0 Ans.
h →0  h 

25. Let [x] denote the greatest integer less than or equal to x. Then :

tan( sin 2 x) + (| x | − sin(x[x])) 2


lim [JEE Main 2019 (11-01-2019-Shift-1)]
x →0 x2

(A) does not exist (B) equals 0 (C) equals  + 1 (D) equals 

Ans. (A)
tan( sin 2 x) (| x | – sin (x[x])) 2
Sol. lim + lim
x →0 x2 x →0 x2

= l1 + l2 (Let)

tan ( sin 2 x) sin 2 x


 l1 = lim  =
x →0 ( sin 2 x) x2

(| x | – sin (x[x])2
l2 = lim
x →0 x2

216
(x − 0)2
RHL = lim+ =1
x →0 x2

2
 x + sin x 
LHL = lim –   =4
x →0  x 

Limit does not exist


x cot(4x)
26. lim is equal to : [JEE Main 2019 (11-01-2019-Shift-2)]
x →0 sin 2 x cot 2 (2x)

(A) 4 (B) 1 (C) 0 (D) 2

Ans. (B)
x  tan 2 (2x)
Sol. lim
x →0 sin 2 x  tan 4x

x2 tan 2 (2x) 4x 1
lim 2
 2
4  =1
x →0 sin x 4x tan 4x 4

cot 3 x − tan x
27. lim is : [JEE Main 2019 (12-01-2019-Shift-1)]
x → 4 cos ( x +  / 4 )

(A) 8 2 (B) 8 (C) 4 2 (D) 4


Ans. (B)
1 − tan 4 x (1 + tan 2 x) (1 − tan 2 x)
Sol. lim = lim
x → 4    x → 4  
tan x  cos  x + 
3
cos  x + 
 4  4
1 − tan x
= 2 lim (1 + tan x) 
x → 4  
cos  x + 
 4
1 − tan x
= 4 lim ; Apply L-H Rule
x → 4  
cos  x + 
 4

− sec 2 x ( 2) 2
= 4 lim =4 =8
x → 4   1
− sin  x + 
 4

217
 − 2sin −1 x
28. lim is equal to [JEE Main 2019 (12-01-2019-Shift-2)]
x →1− 1− x

2 1 
(A) (B) (C) (D) 
 2 2
Ans. (A)
 − 2 sin −1 x  0 
Sol. lim−  
x →1 1− x 0
Apply L-H Rule
1 1
−  2
= lim − 2 2sin −1 x 1 − x 2 = lim 2 1− x
x →1 −1 x →1−
2sin −1 x (1 − x)(1 + x)
2 1− x

2 2
= =
 2 

sin 2 x
29. lim equals : [JEE Main 2019 (08-04-2019-Shift-1)]
x→0 2 − 1 + cos x

(A) 4 (B) 2 2 (C) 4 2 (D) 2


Ans. (C)
sin 2 x
Sol. lim  ( 2 + 1 + cos x )
x →0 1 − cos x

sin 2 x
= lim  ( 2 + 1 + cos x )
x → 0 1 − cos x

sin 2 x 2 2
= lim 2 2= =4 2
x→ 0 x 1
2sin 2
2
2 4

x4 −1 x3 − k3
30. If lim = lim 2 , then k is : [JEE Main 2019 (10-04-2019-Shift-1)]
x →1 x − 1 x →k x − k 2

4 8 3 3
(A) (B) (C) (D)
3 3 2 8

218
Ans. (B)
 x3 − k3 
x −1
4  
Sol. lim = lim 2 x − k 
x →1 x − 1 x →k  x − k 2 
 
 x−k 
3(k) 2 8
 4 × (1)3 =  k=
2(k) 3

x 2 − ax + b
31. If lim = 5 , then a + b is equal to :
x →1 x −1
[JEE Main 2019 (10-04-2019-Shift-2)]
(A) – 7 (B) – 4 (C) 1 (D) 5
Ans. (A)
x 2 − ax + b
Sol. lim =5
x →1 x −1
x→1+t
(1 + t)2 − a(1 + t) + b
 lim + =5
t →0 t
 1− a + b 
 lim +   + (2 − a) + t = 5
t →0  t 
1–a+b=0&2–a=5a=–3&b=–4
 a + b = – 7 Ans.

32. Let f(x) = 5 – |x –2| and g(x) = |x + 1|, x  R. If f(x) attains maximum value at  and
(x − 1)(x 2 − 5x + 6)
g(x) attains minimum value at , then lim is equal to :
x →− x 2 − 6x + 8
[JEE Main 2019 (12-04-2019-Shift-2)]
(A) 3/2 (B) –3/2 (C) –1/2 (D) 1/2
Ans. (D)
Sol. f(x) = 5 – | x – 2 |  Maximum value occurs at x = 2
g(x) = |x + 1|  Minimum value occurs at x = – 1

(x − 1)(x 2 − 5x + 6)  0 
 lim  
x→ 2 (x 2 − 6x + 8) 0

(x − 1) (x − 2) (x − 3) 1
= lim = Ans.
x→ 2 (x − 2) (x – 4) 2

219
x + 2sin x
33. lim is : [JEE Main 2019 (12-04-2019-Shift-2)]
x →0 x + 2sin x + 1 − sin x − x + 1
2 2

(A) 1 (B) 3 (C) 6 (D) 2


Ans. (D)
x + 2sin x 0
Sol. lim  
x →0 x 2 + 2sin x + 1 − sin 2 x − x + 1  0 
Rationalise

= lim
(x + 2sin x) ( x 2 + 2sin x + 1 + sin 2 x − x + 1 )
x →0 (x 2 + 2sin x + 1) − (sin 2 x − x + 1)

(x + 2sin x)  (2) 0


= lim  
x →0 x 2 + 2sin x – sin 2 x + x  0 
Apply L-H Rule
(1 + 2 cos x) 23
= 2 lim = = 2 Ans.
x →0 2x + 2 cos x – 2sin x cos x + 1 2 + 1

3x + 33− x − 12
34. lim is equal to : [JEE Main 2020 (07-01-2020-Shift-1)]
x →2 3− x/2 − 31− x

Ans. (36)
3x + 33− x − 12  0 
Sol. lim
x →2 3− x/2 − 31− x
 
0

32x –12  3x + 27
= lim
x →2 [3x /2 − 3]

(3x − 9) (3x − 3) (3x/2 − 3) (3x/2 + 3) (3x − 3)


= lim = lim
x →2 (3x/2 − 3) x →2 (3x/2 − 3)

= (3 + 3) (9 – 3) = 36

1
 3x 2 + 2  x 2
35. lim   is equal to : [JEE Main 2020 (08-01-2020-Shift-1)]
x →0  7x 2 + 2 
 

1 1
(A) 2
(B) e2 (C) e (D)
e e
Ans. (A)

220
1
 3x 2 + 2  x 2 
Sol. lim  2  (1 )
x →0  7x + 2 
 
 3x 2 + 2   –4 x 2 
lim 2 −1 1 lim 2  1
x → 0 7x + 2 x → 0 7x + 2
=e   x2 =e   x2

= e–2

x + x 2 + x 3 + ... + x n − n
36. If lim = 820, ( n  N ) then the value of n is equal to :
x →1 x −1
[JEE Main 2020 (02-09-2020-Shift-1)]
Ans. (40)
x + x 2 + x 3 + ... + x n − n  0 
Sol. lim   = 820
x →1 (x − 1) 0

Apply L-H Rule; lim1 + 2x + 3x 2 + ...... + nx n −1 = 820


x →1

 1 +2 + 3 + ……+ n = 820
n(n + 1)
 = 820  n2 + n – 1640 = 0
2
 (n + 41) (n − 40) =  n = 40 Ans. {n  N}

1/ x
  
37. lim  tan  + x   is equal to : [JEE Main 2020 (02-09-2020-Shift-2)]
x →0
 4 
(A) e (B) e2 (C) 2 (D) 1
Ans. (B)
1/ x
  
Sol. lim  tan  + x   (1)
x →0
 4 
 
tan  + x  −1
4 
lim 0
 
x →0
=e x
0
Apply L-H Rule
lim sec2 (  /4 + x)
= ex→0 = e2 Ans.

221
1− x+ | x |
38. Let [t] denote the greatest integer  t. if for some   R − 0,1 , lim
= L,
 − x + x
x →0

then L is equal to: [JEE Main 2020 (03-09-2020-Shift-1)]


1
(A) 0 (B) 2 (C) (D) 1
2
Ans. (B)
1− h + h 1 1
Sol. RHL= lim = lim + =
x → 0+ h  − h + 0 h →0  − h 

1− h + h 1 1
LHL = lim = lim + =
x → 0–h  + h –1 h → 0  + h –1  − 1
For limit to exist, LHL = RHL
1 1
 =   –1= ± 
 −1 
 L = 2 Ans.


1  x2 x2 x2 x 2 

39. If lim  8 1 − cos − cos + cos cos   = 2− k , then the value of k is ...

x →0 x
  2 4 2 4  
[JEE Main 2020 (03-09-2020-Shift-1)]

Ans. (8)
1  x 2  x2 
Sol. lim  1 − cos  1 − cos 
x →0 x 8
 2  4 
2sin 2 x 2 / 4 2sin 2 x 2 / 8 1 1
= lim 2 2
 2 2
 2
 2
x →0 (x / 4) (x / 8) (4) (8)
2 2 1
= = = 2−8  k = 8
16  64 256
1 1
(a + 2x) 3 − (3x) 3
40. lim 1 1
(a  0) is equal to : [JEE Main 2020 (03-09-2020-Shift-2)]
x →a
(3a + x) − (4x)
3 3

4 4 1 1
 2 3  2 3  2   2 3  2   2 3
(A)   (B)   (C)     (D)    
9 3  3  9   9  3 
Ans. (C)
Sol. x→a+t
(a + 2(a + t))1/3 − (3(a + t))1/3
 lim+
t →0 (3a + a + t)1/3 − (4(a + t))1/3

222
 2t 1/3  t 
1/3

(3a)1/3 1 +  −  1 +  
= lim+  3a   a 
t →0 
1/3  t 
1/3
 t 
1/3

(4a) 1 +  − 1 +  
 4a   a 

  2t   t  2 1
 1 +  − 1 +   − 
  
1/3 1/3
 
= lim+     9a   3a   =    9 3 
3 3
t →0  4 
 1 + t  − 1 + t    4   1 − 1 
  12a   3a    12 3 

 3   8 − 12   3   4   2   2 
1/3 1/3 1/3

=   =   =    Ans.
 4   3 − 12   4   9   3   9 

1 − cos (p(x))
41. If  is positive root of the equation, p(x) = x2 – x – 2 = 0, then lim+ is
x → x−2
equal to :
[JEE Main 2020 (05-09-2020-Shift-1)]

1 3 3 1
(A) (B) (C) (D)
2 2 2 2
Ans. (B)
Sol. x2 – x – 2 = 0 (x – 2) (x +1) = 0
 x = 2, – 1,   = 2
 p(x) = x2 – x – 2 = (x – 2) (x +1)
 p(x) 
2 sin  
1 − cos (p(x))  2 
lim+ = lim+
x →2 x−2 x →2 (x − 2)

 (x − 2) (x + 1) 
2 sin  
 2 
 lim+
x→2 x−2

 (h) (3 + h) 
2 sin  
= lim +  2    3+ h  = 3
(3 + h)  
h →0
h  2  2
2
 ( 1+ x 2 + x 4 −1) x 
xe −1 

42. lim   [JEE Main 2020 (05-09-2020-Shift-2)]


x →0
1 + x + x −1
2 4

(A) is equal to e (B) is equal to 1 (C) is equal to 0 (D) does not exist

223
Ans. (B)
1 + x2 + x 4 −1  0 
Sol. Consider, lim  
x →0 x 0
1 + x2 + x4 −1 1+ x2 + x4 +1 x + x3
= lim  = lim =0
x →0 x 1 + x 2 + x 4 + 1 x →0 2
1 + x 2 + x 4 −1
Put =t
x
( 1+ x 2 + x 4 −1 ) / x
e −1  ex − 1 
lim =1  lim = 1 .
x →0  1+ x2 + x4 −1   x →0 x 
 
 x 

n
 1 1
 1 + 2 + ....... + n 
43. lim 1 + 2  is equal to : [JEE Main 2021 (25-02-2021-Shift-1)]
n → n
 
 
1 1
(A) (B) 0 (C) (D) 1
2 e
Ans. (D)
1 1 1 1
+ + + ... +
1 + 1 + 1....1 1  1 1 1 n n n n
Sol.  2 1 + + + ... +  
n2 n  2 3 n n2

1 1  1 1 1
 2 1 + + ... +   2
n n  2 n n

If n →

1  1 1
 2 
1 + + ... +  → 0
n  2 n
n
 1 1
 1 + 2 + ... + n 
So, lim 1 +  ; ( 1 )
n →
 n2 
 

 1 1 
lim  1 + 2 + ... + n 
e n→ n 
 n2 
 

224
1 1  1 + 1 + 1 + ... + 1 
1 + + ... + 
Now, 0  2 n  2 3 n 
n  n 
 
 
n
1 1 n 1 n 1

r =1
 =
r n r =1 r

n

n
1
1 1 2
=
n

0 x
dx =
n
 2 
lim  
e n →  n
= e0 = 1

ax − ( e4x − 1)
44. If lim exists and is equal to b, then the value of a – 2b is ______.
x →0 ax ( e4x − 1)
[JEE Main 2021 (25-02-2021-Shift-2)]
Ans. (5)

Sol.
ax − e4x − 1( )  0
lim  0 
x →0 ax e ( 4x
−1 )

= lim
ax − e4x − 1 ( ) = lim
(
ax − e4x − 1 ) Use lim
e4x − 1
=1
x →0

ax 
(e 4x
−1 ) 4x x →0 ax  4x x →0 4x
4x

Apply L'Hospital Rule


a − 4e4x a−4 
= lim  0 form
x →0 8ax

limit exists only when a – 4 = 0  a = 4


4 − 4e4x
= lim
x →0 32x

1 − e4x  0
= lim  0 
x →0 8x

−e4x  4 1 1
= lim = − b= −
x →0 8 2 2

 1
a − 2b = 4 − 2  −  = 5
 2

225
    
 3 sin  6 + h  − cos  6 + h  
The value of lim 2      is
45. 
h →0
 (
3h 3 cosh − sinh  )
 
[JEE Main 2021 (26-02-2021-Shift-1)]
4 2 3 2
(A) (B) (C) (D)
3 3 4 3
Ans. (A)
 1 3   3 sinh  
 3  cosh+ sinh  −  cosh− 
 2 2   2 2  
Sol. L = lim 2  
h →0
 (
3h 3 )( ) 
 
 
4 sinh 4
= lim L=
h→0 3h 3

aex − b cos x + ce− x


46. If lim = 2 , then a + b + c is equal to _______.
x →0 x sin x
[JEE Main 2021 (16-03-2021-Shift-1)]

Ans. (4)
aex − bcos x + ce− x
Sol. lim =2
x →0 x sin x

 x2   x2   x2 
a 1 + x + + ...  − b 1 − + ....  + c 1 − x + − ...... 
 lim    2!    =2
2! 2!
x →0  x sin x 
 x
 x 
a–b+c=0 ….(1)
a–c=0 ….(2)
a+b+c
& =2
2

 a+b+c = 4

cos −1 (x −  x  )  sin −1 (x −  x  )
2 2

47. The value of lim+ , where [x] denotes the greatest


x →0 x − x3
integer  x is : [JEE Main 2021 (17-03-2021-Shift-1)]
 
(A)  (B) 0 (C) (D)
4 2
Ans. (D)

226
cos−1 x sin−1 x 
Sol. lim+  =
x →0 (1 − x2 ) x 2

[r] + [2r] + ..... + [nr]


48. The value of lim , where r is non-zero real number and [r] denotes
n → n2
the greatest integer less than or equal to r, is equal to :
[JEE Main 2021 (17-03-2021-Shift-2)]
r
(A) (B) r (C) 2r (D) 0
2
Ans. (A)
Sol. We know that
r  [r] < r + 1

and 2r  [2r] < 2r + 1

3r  [3r] < 3r + 1

  
nr  [nr] < nr + 1

r + 2r + ....+ nr  [r] + [2r] +....+ [nr] < (r + 2r +....+ nr) + n


n(n + 1) n(n + 1)
r r +n
2 [r] + [2r] + ..... + [nr] 2
2
 2
 2
n n n

n(n + 1)  r r
Now, nlim =
→ 2  n2 2

n(n + 1)r
+n
2 r
and lim 2
=
n→ n 2

So, by Sandwich Theorem, we can conclude that


[r] + [2r] + ..... + [nr] r
lim =
n→ n2 2

tan (  cos 2  )
49. The value of the limit lim is equal to :
→0 sin ( 2 sin 2  )
[JEE Main 2021 (17-03-2021-Shift-2)]
1 1 1
(A) − (B) − (C) 0 (D)
2 4 4
Ans. (A)

Sol. lim
( (
tan  1 − sin2  ))
→ 0
(
sin 2 sin  2
)

227
= lim
− tan  sin2  ( )
→0
(
sin 2 sin 2
)

 tan  sin2 
= lim − 
( )  2 sin2   1 1
  =−
→ 0   sin2  (
  sin 2 sin2  )  2 2

sin −1 x − tan −1 x
50. If lim is equal to L, then the value of (6L + 1) is
x →0 3x 3
[JEE Main 2021 (18-03-2021-Shift-1)]
1 1
(A) (B) (C) 6 (D) 2
6 2
Ans. (D)
 x3   x3 
x + + ...  −  x − + ... 
lim    =1
3! 3
Sol. 3
x →0 3x 6

So 6L + 1 = 2
 x+2 
51. If the value of lim 2 − cos x cos 2x
x →0
( ) 2 
 x 
is equal to ea, then a is equal to
[JEE Main 2021 (20-07-2021-Shift-1)]
Ans. (3)
 x+2  (1–cos x cos 2x )(x + 2)

( ) 2  lim
Sol. lim 2 − cos x cos 2x  x 
= e x→0 x2
x →0

(1−cos x.cos 2x )(x +2)


lim
2
lim
(1−(1−sin x )(1−2sin x ))(x +2) 2 2

x (1+ cos x cos 2x )


x→0 2
x (1+ cos x cos 2x )
x →0 2

=e =e
(3sin x − 2sin x )( x + 2)
lim
2 4

x (1+ cos x cos 2x )


x →0 2

=e

lim
(
sin 2 x 3− 2sin 2 x (x + 2))
x →0
(
x 2 1+ cos x cos 2x )
=e
3 2
=e 1+1

= e3
a = 3

xex –  log (1 + x) +  x 2 e – x
52. If lim = 10,     R, then value of  +  +  is _____.
x →0 x sin 2 x
[JEE Main 2021 (20-07-2021-Shift-2)]
Ans. (3)

228
 x 2 x3   x 2 x3   x 2 x3 
x  1 + x + + + ....  –   x – + + ....  + x 2 1 – x + – + .... 
Sol. lim  2! 3!   2 3   2! 3!  = 10,
x →0 3
x
  –  = 0,   = 
 3
 + + =0   = –
2 2
    3  – 2 + 9
 – –  = 10  – + = 10  = 10
2 3 2 3 2 6
  = 6,  = 6,  = – 9
So, the value of  +  +  = 3
53. Let f : R → R be a function such that f(2) = 4 and f (2) = 1. Then, the value of
x 2f (2) − 4f (x)
lim is equal to : [JEE Main 2021 (27-07-2021-Shift-1)]
x →2 x−2
(A) 12 (B) 8 (C) 4 (D) 16

Ans. (A)
x 2f (2) − 4f (x) 2xf (2) − 4f (x)
Sol. lim = lim = 2·(2)·f(2) – 4f (2) = 16 – 4 = 12
x →2 x−2 x → 2 1

 x 
54. The value of lim  8  is equal to :
x →0
 1 − sin x – 1 + sin x 
8

[JEE Main 2021 (27-07-2021-Shift-2)]


(A) – 1 (B) 4 (C) 0 (D) – 4

Ans. (D)

Sol. Rationalize denominator three times

x{(1 − sin x)1/8 + (1 + sin x)1/8 }{(1 − sin x)1/4 + (1 + sin x)1/4 }{(1 − sin x)1/2 + (1 + sin x)1/2 }
lim
x →0 (1 − sin x − 1 − sin x)
8x
 lim = −4
x →0 −2sin x

 9 x 
55. lim    is equal to:
 n =1 n ( n + 1) x + 2 ( 2n + 1) x + 4 
x →2 2

[JEE Main 2021 (26-08-2021-Shift-2)]


5 1 9 7
(A) (B) (C) (D)
24 5 44 36
Ans. (C)
9  
x
Sol. lim   
x →2 
n =1  n ( n + 1) x + 2 ( 2n + 1) x + 4 
2

229
9  x 
lim   
x →2 
n =1 ( nx + 2 ) + x ( nx + 2 )
2

 
9  
x
lim   
n =1  ( nx + 2 )( nx + 2 + x ) 
x →2

9  
1 1
lim   − 
x →2
n =1  ( nx + 2 ) ( n + 1) x + 2 
1 1
= −
x + 2 2x + 2
1 1

2x + 2 3x + 2

1 1
+ −
9x + 2 10x + 2
1 1

x + 2 10x + 2
1 1 9
 lim − =
x →2 x + 2 10x + 2 44
( )
− 1 − 2 ( x 2 + bx + c )
2 x 2 + bx + c
e
56. If    are the distinct roots of x + bx + c = 0 , then lim2

( x − )
x → 2

is equal to: [JEE Main 2021 (27-08-2021-Shift-1)]

(A) b2 + 4c (
(B) 2 b 2 + 4c ) (
(C) 2 b 2 − 4c ) (D) b2 − 4c
Ans. (C)
( )
− 1 − 2 ( x 2 + bx + c )
2 x 2 + bx + c
e
Sol. lim
( x − )
x → 2

 2 ( x 2 + bx + c ) 22 ( x 2 + bx + c )2 
1 + + +  − 1 − 2 ( x 2 + bx + c )
 1! 2! 
 lim  
( x − )
x → 2

2 ( x 2 + bx + c )
2

 lim
( x − )
x → 2

2 ( x −  ) ( x − )
2 2

 lim
( x − )
x → 2

 2 ( −  ) = 2 ( b2 − 4c )
2

230
57. If lim
x →
( )
x 2 − x + 1 − ax = b , then the ordered pair (a, b) is:
[JEE Main 2021 (27-08-2021-Shift-2)]
 1  1  1  1
(A)  1, −  (B)  −1,  (C)  −1, −  (D)  1, 
 2  2  2  2

Ans. (A)
Sol. lim
x →
( x 2 − x + 1 − ax = b , )
 lim
x →
( x 2 − x + 1 − ax  ) x 2 − x + 1 + ax
x 2 − x + 1 + ax
=b

 lim
(x 2
− x + 1) − ( ax )
=b
2

x →
x 2 − x + 1 + ax
 Limit exist only if a 2 = 1
a = 1, − 1
−x + 1
 lim =b
x →
x 2 − x + 1 + ax
1
−1 +
 lim x =b
x → 1 1
1− + 2 + a
x x
−1
 =b
1+ a
But a  −1
So, a = 1
−1
b=
2
−1
( a, b ) = 1, 
 2 

sin 2 (  cos 4 x )
58. lim is equal to: [JEE Main 2021 (31-08-2021-Shift-1)]
x →0 x4
(A) 4 (B) 22 (C) 42 (D) 2
Ans. (C)

Sol. lim
(
sin 2  (1 − sin 2 x )
2
)
x →0 x4

= lim
(
sin 2  (1 + sin 4 x − 2sin 2 x ) ) = lim sin (  −  ( 2sin
2 2
x − sin 4 x ) )
x →0 4 x →0 4
x x

231
(
sin 2  ( 2sin 2 x − sin 4 x ) )  sin   ( 2sin 2 x − sin 4 x )   2 ( 2sin 2 x − sin 4 x )2
2

= lim = lim    
x →0   ( 2sin x − sin x ) 
4
x →0 x 2 4
x4
 
4
 sin x 
 ( 2 − sin 2 x ) = 42
2
=  lim 
2

x →0
 x 
tan 3 x − tan x
Let  = lim and  = lim ( cos x ) are
cot x
59. the roots of equation
x→

  x →0
4 cos  x + 
 4
ax + bx − 4 = 0 , then the ordered pair (a, b) is:
2

[JEE Main 2021 (31-08-2021-Shift-2)]


(A) (1, –3) (B) (–1, 3) (C) (1, 3) (D) (–1, –3)
Ans. (C)
Sol.  = lim ( cos x )
cot x

x →0
cos x −1
lim
= e x→0 tan x

− sin x
lim
=e = e0 = 1
x→0 sec2 x

tan 3 x − tan x
 = lim
x→

 
4 cos  x + 
 4
tan x ( tan x + 1)( tan x − 1)
= lim
x→

 
4 cos  x + 
 4
tan x − 1 sec 2 x 2 2
= 2 lim = 2 lim = = −4
x→

  x→

  −1
4 cos  x +  4 − sin  x + 
 4  4
Equation whose roots are α and β is
x 2 + 3x − 4 = 0
a = 1, b = 3

60. Let f ( x ) = x 6 + 2x 4 + x 3 + 2x + 3, x  R Then the natural number n for which


x n f (1) − f ( x )
lim = 44 is [JEE Main 2021 (01-09-2021-Shift-2)]
x →1 x −1

Ans. (7)
9x n − x 6 − 2x 4 − x 3 − 2x − 3
Sol. Given lim = 44
x →1 x −1
 xn −1   x6 −1   x 4 −1   x3 −1   x −1 
 lim9  −  − 2 −  − 2  = 44
x →1
 x − 1   x − 1   x − 1   x − 1   x − 1 
 9n − 6 − 8 − 3 − 2 = 44
n =7

232
EXERCISE–V

x2
a − a2 − x2 −
Q.1 Let L = lim 4 , a > 0. If L is finite, then [JEE 2009, 3]
x →0 x4
1 1
(A) a = 2 (B) a = 1 (C) L = (D) L =
64 32
Ans. (A,C)
a − | a | (1 − x 2 / a 2 )1/2 − x 2 / 4
Sol. L = lim = finite ; a > 0
x →0 x4
 1 (1/ 2) (1/ 2 − 1) 
a − a 1 − (x 2 / a 2 ) + (−x 2 / a 2 ) 2 − ...... − x 2 / 4
 lim  2 2!  = finite
x →0 4
x
 1 1 2  1  4
 − x + 3 x
 lim  2a 4  4  8a  = finite
x →0 x
 1 1
 −  1
 lim  2a 2 4  + 3 = L = finite
x →0 x 8a
 1 1
 − =0 & L= 3
2a 4 8a
1
 a=2&L= Ans.
64
1/x
Q.2 If lim 1 + x ln(1 + b2 )  = 2b sin2 , b > 0 and   (−, ], then the value of  is
x →0
[JEEAdv 2012]
   
(A)  (B)  (C)  (D) 
4 3 6 2
Ans. (D)
1/x
Sol. lim 1 + x ln(1 + b2 )  (1) = 2b sin2
x →0

lim (1+ x ln(1+ b2 )−1) 


1
e x →0 x = 2bsin 2 

 e1n(1+b ) = 2bsin 2 
2

1  1
 1 + b2 = 2b sin2  sin2 = b+  ; b > 0
2 b

 sin2  1

233

 sin2 = sin  = ± 1   = ± Ans.
2

Q.3 Let (a) and (a) be the roots of the equation [JEE 2012]
( 3
)
1 + a −1 x 2 + ( ) (
1 + a −1 x + 6
)
1 + a − 1 = 0 where a > –1
Then lim+ (a) and lim+ (a) are
a →0 a →0
5 1 7 9
(A) – and 1 (B) – and –1 (C) – and 2 (D) – and 3
2 2 2 2
Ans. (B)
Sol. Put 1 + a = t6
as a → 0+ t → 1+
 (t2 – 1)x2 + (t3 – 1)x + (t – 1) = 0
 (t +1)x2 + (t2 + 1 + t)x +1 = 0
Now, as t → 1+, 2x2 + 3x +1 = 0
 (2x +1) (x + 1) = 0
1
 x=− &x=–1
2
1
 Roots are − & – 1 Ans.
2

 x2 + x +1 
Q.4 If lim  − ax − b  = 4, then [JEE 2012]
x →
 x +1 
(A) a = 1, b = 4 (B) a = 1, b = –4 (C) a = 2, b = –3 (D) a = 2, b = 3

Ans. (B)
x 2 + x + 1 − (ax + b) (x + 1)
Sol. lim =4
x → x +1
(x 2 + x + 1) − (ax 2 + (a + b) x + b)
 lim =4
x → x +1
(1 − a)x 2 + (1 − a − b)x + (1 − b)
 lim =4
x → x +1
 1− b 
(1 − a)x + (1 − a − b) +  
 lim  x  =4
x → 1 + 1/ x
 1–a=0&1– a–b=4

234
 a = 1 & b = – 4 Ans.

Q.5 The largest value of the non-negative integer a for which


1− x
 −ax + sin(x − 1) + a  − x
1 1
lim   = is [IIT Adv. 2014]
x →1
 x + sin(x − 1) − 1  4
Ans. (0)
(1− x )
 −ax + sin(x − 1) + a  1
Sol. lim   =
x →1
 x + sin(x − 1) − 1  4
Put x = 1+ h
1+ 1+ h
 − a (1 + h) + sin h + a  1
 lim+   =
h →0  1 + h + sin h − 1  4
1+ 1+ h
 − ah + sin h  1
 lim+   =
h →0  h + sin h  4

 − a +1  − a +1
2
1 1
   =  =   a = 0 or a = 2
 2  4 2 2
– ah + sin h
But; when a = 2, tends to – ve value
h + sin h

 a = 0 Ans.
 ecos(  ) − e 
n
e
Q.6 Let m and n be two positive integers greater than 1. If lim   = −   then
→0   m  2
 
m
the value of is [IIT Adv. 2015]
n
Ans. (2)
 ecos(  ) − e   0 
n

Sol. lim   
→0   m  0 
 
 ecos(  )−1 − 1  cos  n − 1 – e
n

lim e   =
→0  (cos  n − 1)   m
2
 
– 2sin 2 ( n / 2) ( n / 2) 2 −1
lim  =
→0 ( n / 2) 2 m 2

– 2sin 2 ( n / 2) ( n / 2) 2 −1 −1
 lim  = lim  2n −m =
→0 ( / 2)
n 2
 m → 0 2 2

235
 lim  2n −m = 1  2n – m = 0
→0

m
 = 2 Ans.
n
x 2 sin(x)
Q.7 Let    R be such that lim = 1. Then 6( + ) equals [IIT Adv. 2016]
x →0 x − sin x

Ans. (7)
x 2 sin(x)
Sol. lim =1
x →0 x − sin x

sin(x) x  x 2
 lim  =1
x →0 (x) x − sin x

x 3
 lim =1
x →0  x3 
x −  x − + .... 
 3! 
 
 lim =1   – 1 = 0 & =1
x →0   − 1  1 1/ 6
 2 +
 x  6

  =1 &  =1/6

 6( + ) = 7 Ans.

1 − x(1+ |1 − x |)  1 
Q.8 Let f(x) = cos   for x  1. Then [JEE Adv 2017]
|1 − x | 1− x 

(A) lim− f(x) does not exist (B) lim− f(x) = 0


x →1 x →1

(C) lim+ f(x) = 0 (D) lim+ f(x) does not exist


x →1 x →1

Ans. (B,D)
Sol. lim f(x)
x →1+

1 − (1 + h) (1 + h)  −1 
lim  cos  
x →1+ h h  h 
− h 2 − 2h 1
= lim +  cos  
x →0 h h
 [– 1, 1]

236
1
= lim+ (−2 + h) cos   = does not exist
x →0 h
lim f(x)
x →1−

1 − (1 − h) (1 + h) 1
lim  cos  
x →1–h h h
h2 1 1
= lim+ .cos   = lim+ h.cos   = 0
h →0 h  h  h →0 h

Q.9 Let f: → be a function. We say that f has


f (h) − f (0)
PROPERTY 1 : if lim exists and is finite, and
h →0 h

f (h) − f (0)
PROPERTY 2 : if lim exists and is finite.
h →0 h2

Then which of the following option(s) is /are correct?


[JEE (advanced) 2019]
(A) f(x) = x |x| has PROPERTY 2 (B) f (x) = x2/3 PROPERTY 1
(C) f (x) = sin x has PROPERTY 2 (D) f (x) = |x| has PROPERTY 1

Ans. (B,D)
RHL= 1
Sol. (A) f(x) = x|x|
RHL= –1
f (h) − f (0) h|h|
lim = lim
h →0 h2 h →0 h2

= does not exist.


(B) f(x) = x2/3
f (h) − f (0) h 2/3
 lim = lim =0
h →0 |h| h →0 | h |

(C) f(x) = sin x


f (h) − f (0) sin h
 lim 2
= lim = doe not exist
h →0 h h →0 h2

(D) f(x) = |x|


f (h) − f (0) |h|
 lim = lim = lim | h | = 0
h →0 |h| h →0 | h | h →0

237
 k +1  k+2 
n
 sin  n + 2   sin  n + 2  
Q.10 For non - negative inters n, let f (n) = k =0
 k +1 
n
 sin 2  n + 2  
k =0
−1
Assuming cos x takes values in [0, ], which of the following option(s) is / are correct
?
[JEE (advanced) 2019]
3
(A) sin (7 cos-1 f(5)) = 0 (B) f ( 4 ) =
2
1
(C) lim f (n) = (D) If  = tan(cos−1 f (6)), then 2 + 2 −1 = 0
n → 2
Ans. (A,B,D)
 k +1  k+2 
n
 sin  n + 2    sin  n + 2  
k =0
Sol. f (n) =
 k +1 
n
 2sin 2  n + 2  
k =0

n
     2k + 3  
 cos  n + 2  – cos  n + 2  
k =0
= n
  2k + 2  
 1 − cos  
n + 2  
k =0

    2k + 3 
n n
 cos  n + 2  −  cos  n + 2  
k =0 k =0
=
 2k + 2 
n
(n + 1) −  cos  
k =0  n+2 

 n +1 
sin  
    n + 2   cos  n + 3  
(n + 1)  cos  −  
n+2    n+2
sin  
n+2  n +1    
= ;  = − 
 n +1  n+2  n+2
sin  
(n + 1) –  n + 2   cos 
  
sin  
n+2

     
(n + 1)  cos   − cos   + 
=  n + 2   n +2
(n + 1) + 1

     
(n + 1) cos   + cos  
= n+2  n + 2  = cos   
 
(n + 2) n+2

  
 f(n) = cos  
n+2

238
     
(A) sin (7 cos −1 5) = sin  7 cos −1  cos   = sin  7   = 0
  7   7

 3
(B) f (4) = cos   =
6 2

  
(C) lim f (n) = lim cos   = cos 0 = 1
n → n → n+2

    
(D)  = tan ( cos −1 (f (6)) ) = tan  cos −1  cos   = tan
  8  8

 2 tan  / 8
  tan =
4 1 − tan 2  / 8

2
 1−  2 + 2 – 1= 0
1 − 2

Q.11 Let e denote the base of the natural logarithm. The value of the real number a for
1
(1 − x) x − e −1
which the right hand limit lim is equal to a non-zero real number,
x→0 xa
is____ [JEE Advanced 2020]
Ans. (1)
1
(1 − x) x − e−1  0 
Sol. lim+  
x→0 xa 0
Apply L-H Rule
1  x 
(1 − x) x  x − 1 − ln(1 − x) 
 lim+ a −1
 
x→0 a  x  x2 
1
(1 − x)  x − (x − 1) ln(1 − x) 
x
 lim+ 
x→0 a  x a −1  x 2 (x − 1) 

  x 2 x3  
1
 x + (x − 1)  x + +  
(1 − x)  x
 2 3 
 lim+ 
x→0 a  x
a −1
 x −1 

239
 x2  1 1  1 1 
(1 − x)1/ x  +  −  x 3 +  −  x 4 
 lim+  2  2 3 3 4 
a +1
x→0 − a(1 − x)  x

 which is a real number only when a + 1 = 2 → a = 1 Ans.

4 2(sin 3x + sin x)
Q.12 The value of the limit is lim
x→  3x 5x   3x 
2  2sin 2x sin + cos  −  2 + 2 cos 2x + cos 
 2 2   2 

[JEE Advanced 2020]


Ans. (8)
 4 2(sin 3x + sin x) 
Sol. lim  

x→  x 7x 5x  3x 
2  cos − cos + cos −  2 2 cos 2 x + cos  
 2 2 2  2 

 8 2  sin x cos 2 x 
= lim  
x→   x 3x   5x 7x 
2   cos − cos  +  cos − cos  − 2 2 cos 2 x 
 2 2   2 2  

16 2 sin x cos 2 x
= lim
 x x
2 2sin x  sin + 2sin 3x  sin − 2 2 cos 2 x
x→
2 2

16 2 sin x cos 2 x
= lim
 x
x→
2 2sin (sin 2x  cos x) − 2 2 cos 2 x
2

16 2 sin x cos 2 x
= lim
 x
x→
2 8sin sin x  cos 2 x − 2 2 cos 2 x
2

16 2 16  2
= =
−2 2 8−4
1
8
2

= 8 Ans.

240
Maths IIT-JEE ‘Best Approach’ (MCSIR) Limits

BRAHMASTRA
FINAL REVISION MODULE BEFORE EXAMINATION

241
Maths IIT-JEE ‘Best Approach’ (MCSIR) Limits

LIMITS
THINGS TO REMEMBER

Limit of a function f(x) is said to exist as, x→a when


Lim− f (x) = Lim+ f (x) = finite quantity.
x →a x →a

FUNDAMENTAL THEOREMS ON LIMITS

Let Lim f (x) = l & Lim g (x) = m. If l & m exists then :


x →a x →a

(i) Lim f (x) ± g (x) = l ± m (ii) Lim f(x). g(x) = l. m


x →a x →a

f (x)
(iii) Lim = , provided m  0
x →a g (g) m
(iv) Lim k f(x) = k Lim f(x) ; where k is a constant.
x →a x →a

 
(v) Lim f [g(x)] = f  Lim g (x)  = f (m) ; provided f is continuous at x = m.
x →a  x → a 
For example Lim l n (f(x) = ln  Lim f (x)  = l n l (l > 0).
x →a x → a 

STANDARD LIMITS

sin x tan x tan −1 x sin −1 x


(a) Lim = 1 = Lim = Lim = Lim
x →0 x x →0 x x →0 x x →0 x
[Where x is measured in radians]
x
 1
(b) Lim (1 + x)1/x = e = Lim 1 +  note however there Lim (1 – h)n = 0
x →0 x →  x h → 0+
n→

and Lim (1 + h )n → 
h → 0+
n→

(c) If Lim f(x) = 1 and Lim  (x) =  , then ;


x →a x →a

Lim  f (x) 
(x) Lim (x)[f (x) −1]
=e x →a

x →a

(d) If Lim f(x) = A > 0 & Lim  (x) = B (a finite quantity) then ;
x →a x →a

Lim [f(x)] (x) = ez {where z = Lim  (x). ln[f(x)]} = eBlnA = AB


x →a x →a

a −1 x
ex − 1
(e) Lim = ln a (a > 0). In particular Lim =1
x →0 x x →0 x
xn − an
(f) Lim = n a n −1
x →a x−a

SQUEEZE PLAY THEOREM

If f(x)  g(x)  h(x)  x & Limit f(x) = l = Limit h(x) then Limit g(x) = l.
x→a x→a x→a

242
Maths IIT-JEE ‘Best Approach’ (MCSIR) Limits

INDETERMINANT FORMS

0 
, , 0  , 0º, º,  −  and 1
0 

Note :
(i) We cannot plot  on the paper. Infinity () is a symbol & not a number. It does
not obey the laws of elementary algebra.
(ii) += (iii)  ×  =  (iv) (a/) = 0 if a is finite
a
(v) is not defined.
0
(vi) a b = 0 , if & only if a = 0 or b = 0 and a & b are finite.

The following strategies should be born in mind for evaluating the limits

(a) Factorisation
(b) Rationalisation or double rationalisation
(c) Use of trigonometric transformation ;
appropriate substitution and using standard limits
(d) Expansion of function like Binomial expansion, exponential & logarithmic expansion,
expansion of sinx , cosx , tanx should be remembered by heart & are given below :
x ln a x 2 ln 2 a x 3 ln 3 a
(i) a x = 1 + + + + .........a  0
1! 2! 3!
x x 2 x3
(ii) e x = 1 + + + + ............ x  R
1! 2! 3!
x 2 x3 x 4
(iii) ln(1+ x) = x − + − + .........for − 1  x  1
2 3 4
 
(iv) sin x = x − + − + ... x   − , 
3 5 7
x x x
3! 5! 7!  2 2
 
(v) cos x = 1 − + − + ...... x   − , 
2 4 6
x x x
2! 4! 6!  2 2
x 3 2x 5   
(vi) tan x = x + + + ........ x   − , 
3 15  2 2
x3 x5 x7
(vii) tan–1x = x − + − + .......
3 5 7

243
Maths IIT-JEE ‘Best Approach’ (MCSIR) Limits

SOLVED EXAMPLE
xf (2) − 2f (x)
1. Let f(2) = 4 and f (2) = 4 . Then lim is given by [JEE 2002]
x →2 x−2
(A) 2 (B) –2 (C) –4 (D) 3

Ans. (C)
xf (2) − 2f (x) 0
Sol. lim  
x →2 x−2 0
Apply L-H rule

f (2) − 2f '(x)
= lim = f (2) − 2f'(2) = 4 − 8 = −4
x →2 1

1 + 24 + 34 + ......n 4 1 + 23 + 33 + ......n 3
2. lim − lim [JEE 2003]
n → n5 n → n5
1 1 1
(A) (B) (C) Zero (D)
5 30 4

Ans. (A)
1 + 24 + 34 + ......n 4 1 + 23 + 33 + ......n 3
Sol. lim − lim
n → n5 n → n5
 n(n + 1) 
2

1  1   2   
n 
4 4 4

= lim    +   + ... +    − lim  2 


]
n → n  n
   n   n   n → n5
4
1 n r 11 2 1 
= lim    − lim  + 2 + 3 
r =1  n  4n n n 
n → n n →

1 1
x5 1
=  x dx − 0 =
4
=
0
5 0 5

( f (x) ) −9
2

3. Let f : R → [0, ) be such that lim f (x) exists and lim = 0 . Then lim f (x)
x →5 x →5 | x −5| x →5

equals [JEE Mains 2011]


(A) 0 (B) 1 (C) 2 (D) 3

Ans. (D)
Sol. lim f (x) exists
x →5

(f(x))2 − 9
lim = 0  lim(f(x))2 = 9
x →5
x −5 x →5

 lim f (x) = 3 { f : R → [0, )}


x →5

244
Maths IIT-JEE ‘Best Approach’ (MCSIR) Limits

 − 2sin −1 x
4. lim is equal to [JEE Main 2019]
x →1− 1− x
1 2 
(A) (B) (C) (D) 
2  2

Ans. (B)
 – 2 sin−1 x 0
Sol. lim− 0
x →1 1−x  
Apply L – H Rule
1 2

−1
2 2 sin x 1 − x2 2
= lim−  lim−  2/
x →1 1 x →1 −1
2 sin x 1 + x
1 − x(–1)
2

cot 3 x − tan x
5. lim is : [JEE Main 2019]
x → 4 cos ( x +  / 4 )

(A) 4 (B) 4 2 (C) 8 2 (D) 8

Ans. (D)
cot3 x − tan x
Sol. lim
x → /4  
cos  x + 
 4

 2 lim
(1 − tan x ) 2

x → /4  
cos  x + 
 4 

 2 lim
(cos 2
x − sin2 x )
x → /4 cos x.cos ( x +  / 4)
2

 4 lim
( cos 2
x − sin2 x )
x → /4 1
( cos x + sin x )
2
 4 2 lim ( cos x + sin x ) = 8
x → /4

6. For each x  R, let [x] be the greatest integer less than or equal to x. Then
x ( x  + x ) sin  x 
lim is equal to [JEE Main 2019]
x →0− x
(A) – sin1 (B) 0 (C) 1 (D) sin1

Ans. (A)
x [x]+ | x | sin[x]
Sol. lim−
x →0 |x|
lim[x] = −1
x → 0−

x(−1 − x) sin ( −1)


 lim− = − sin 1
x →0 −x

245
Maths IIT-JEE ‘Best Approach’ (MCSIR) Limits

7. Let f(x) = 5 – |x –2| and g(x) = |x + 1|, x  R. If f(x) attains maximum value at  and g(x)
(x − 1)(x 2 − 5x + 6)
attains minimum value at , then lim is equal to :
x →− x 2 − 6x + 8
[JEE Main 2019]
(A) ½ (B) –3/2 (C) 3/2 (D) –1/2

Ans. (A)
Sol. f(x) = 5 - x − 2 & g(x)= x + 1
 =2  = −1

(x − 1)(x 2 − 5x + 6)
lim
x →− x 2 − 6x + 8
(x− 1)(x− 3)(x− 2)
= lim
x →− (x− 4)(x− 2)
(x− 1)(x − 3) 1.(−1) 1
= lim = =
x →2 (x− 4) −2 2

x + x 2 + x 3 + ... + x n − n
8. If lim = 820, ( n  N ) then the value of n is equal to : [JEE Main 2020]
x →1 x −1

Ans. (40)
lim
( x − 1) ( x 2
−1 ) +....+ ( x
n
) =820
−1
Sol. +
x −1
x →1 x −1 x −1
 1 + 2 + 3 + ......+n = 820
  n =820
n(n+ 1)
 =820
2
 n = 40

1− x+ | x |
9. Let [t] denote the greatest integer  t. if for some   R − 0,1 , lim = L , then
 − x + x
x →0

L is equal to: [JEE Main 2020]


1
(A) 0 (B) 2 (C) (D) 1
2

Ans. (B)
1− x − x 1
Sol. LHL: lim− =
x→0  − x −1  −1
1− x + x 1
RHL: lim+ =
h→ 0  − x +1 
for existence of limits
1 1
=
  −1
|| = | − 1|

246
Maths IIT-JEE ‘Best Approach’ (MCSIR) Limits

1
2 = 2 - 2 + 1   =
2
1
L = =2

 1  x2 x2 x2 x 2  
10. If lim  8 1 − cos − cos + cos cos   = 2− k , then the vlaue of k is ...
 
x →0 x 2 4 2 4  
[JEE Main 2020]

Ans. (8)
 1  x2 x2 x2 x 2  
Sol. lim  8 1 − cos − cos + cos cos  
 
x →0 x 2 4 2 4  
2 2
 x2   x2   x2   x2 
 1 − cos   1 − cos    . 
= lim 
2   4   2   4 
.
x→0
 x2 
2
 x2 
2
x8
   
 2   4 
1 1 1 1
= . .  = 2 −k
2 2 64 256
=2-8 = 2-k  k = 8

1 − cos ( p ( x ) )
11. If  is positive root of the equation, p(x) = x2 – x – 2 = 0, then lim+ is
x → x+−4
equal to : [JEE Main 2020]
1 3 3 1
(A) (B) (C) (D)
2 2 2 2

Ans. (B)
Sol. f(x) = x2 –x –2
2
−1 =
1 − cos( x − 2)( x + 1)
lim+
x→2 x + − 4
1 − cos( x − 2)( x + 1)
lim
x→2+ ( x − 2)
1 − cos(h (h + 3))
lim
h →0 h
1 − cos(h (h + 3)) 1 3
lim  (h + 3)2  9 =
h →0 h  (h + 3)
2 2
2 2

247
Maths IIT-JEE ‘Best Approach’ (MCSIR) Limits

 ( 1+ x 2 + x 4 −1)/ x 
xe −1 

12. lim   [JEE Main 2020]


x →0
1 + x + x −1
2 4

(A) is equal to e (B) is equal to 1 (C) is equal to 0 (D) does not exist

Ans. (B)
  ( 1+ x2 + x4 )2 −1  
   
  x( 1+ x + x +1) 
( )
2 4 

x e − 1  1 + x2 + x 4 + 1
 ( 1+ x2 + x4 −1) /x   
x e − 1  
Sol. lim    lim  
x →0
(
1 + x2 + x4 −1 ) x →0
( )
x2 + x4
 x3 + x   x3 + x 
   
 2x   2x 
e 
−1

e −1
 
x lim  2 = lim  2 =1
x →0  x3 + x  x →0  3
x + x
x 2  2
 2   2 

13. The largest value of the non-negative integer a for which


1− x
 −ax + sin(x − 1) + a 1− x 1
lim   = is [IIT Adv. 2014]
x →1
 x + sin(x − 1) − 1  4

Ans. (0)
1+ x
 −a(x − 1) + sin(x − 1)  1
Sol. Lim   =
x → 1  (x − 1) + sin(x − 1)  4

1+ x
 sin(x − 1) 
 −a + 
 x −1  1
Lim   =
x →1
 1 + sin(x − 1)  4

 (x − 1)  
2
 −a + 1  1
  =
 1 + 1  4
(–a + 1)2 = 1
–a + 1 = 1 or –a + 1 = –1
a=0 or a = 2 (rejected)
Max value of a = 0

x 2 sin(x)
14. Let    R be such that lim = 1. Then 6( + ) equals [IIT Adv. 2016]
x →0 x − sin x

248
Maths IIT-JEE ‘Best Approach’ (MCSIR) Limits

Ans. (7)
Sol. It is given that   R such that
x2.sin(x)
lim =1
x →0 x − sin x
therefore,

x2.sin(x)
lim =1
x →0 x − sin x

 sin(x) 
x2.  x
 x 
 lim =1
x →0 x − sin x
x3
  lim =1
x →0  x3 x5 
x −  x − + − ......
 3! 5! 
 

x3
  lim =1
x →0 x3 x5
x (  − 1) + − + ......
3! 5!
For finite limit  = 1 ,
1
3!  = 1   =
6
Then,

6( + ) = 6 1 +  = 6 + 1 = 7
1
 6

1 − x(1+ |1 − x |)  1 
15. Let f(x) = cos   for x  1. Then [JEE Adv 2017]
|1 − x | 1− x 
(A) lim− f(x) does not exist (B) lim− f(x) = 0
x →1 x →1
(C) lim+ f(x) = 0 (D) lim+ f(x) does not exist
x →1 x →1

Ans. (B,D)
1 – x(1+ | 1 – x |)  1 
Sol. f(x) = cos  
|1 – x | 1 – x 
1 – (1 + h)[1 + h] 1
lim− f(x) = lim cos  
h→1 h→0 (h) h
1 – 1 + h2 1
= lim cos  
h→ 0 h h
1
= limh cos  
h→0 h
=0
1 – (1 + h)[1 + h] 1
lim f(x) = lim cos  
h→1+ h→0 (h) h
 −2h – h2  1
= lim   cos  
h→0
 h  h
1
= lim ( −2 − h) cos  
h→0 h

249
Maths IIT-JEE ‘Best Approach’ (MCSIR) Limits

= −2  (Some value oscillating between -1 and 1 )

 does not exist.

16. Let e denote the base of the natural logarithm. The value of the real number a for which
(1 − x)1/ x − e −1
the right hand limit lim+ is equal to a non-zero real number, is____
x →0 xa
[JEE Advanced 2020]

Ans. (1.00)
1
ln(1 − x )
e ( )
1/x
ln 1− x
− e−1 ex − e−1
Sol. L = lim  L = lim+
+
x →0 xa x →0 xa
1 x2 x3   x x2 
 −x − − ... −  + ...
x  2 3  −1 −1 2 3 
e −e e .e   − e−1
L= lim+  L = lim+
x →0 xa x →0 xa
 −  x + x2 .... 
−1  2 3


 
e e − 1
 
L = lim  
x →0+ xa
  x x2 
2  
  +   
  x x 2  2 3  
e−1 1 +  − −  +  .... − 1
  2 3  2!  

  
 
 
 L = lim+  
a
x →0 x
 1 x
2 
 x +  
−1   1 x   2 3 
e   − − ... + ....
 2 3  2!
 
 
= lim+  
a −1
x →0 x
for Non - Zero limit a – 1 = 0  a = 1

250
“If you have a Dream, don’t just sit there.

Gather courage to believe that you can

succeed and leave no stone unturned to

make it a reality.”

You might also like